Download as pdf or txt
Download as pdf or txt
You are on page 1of 483

Physiology exam 2022

1. How does angiotensin II raise blood pressure?

Directly by increasing vasoconstriction and indirectly by stimulating aldosterone release

Only by increasing sodium (and, thus, water) reabsorption in the kidney

Only by causing vasoconstriction

2. During the cardiac cycle, contraction of the right atrium leads to an increase of approximately __________ in the hydrostatic
pressure within the right atrium.

1-2 mm Hg

4-6 mm Hg

7-8 mm Hg

10-12 mm Hg

3. At the end of the atrial diastole

Right ‘ventricle is completely full

Left ventricle is completely empty

Left ventricular pressure is about 35 mmHg

Left atrial pressure is about 20 mmHg

Right ventricular pressure is about 10 mmHg

4. After a heart contraction, all of the following play a role in reducing the cytoplasmic Ca2+ concentration EXCEPT:

Sarcoplasmic/endoplasmic reticulum Ca2+ ATPase (SERCA)

Plasma membrane Na+/Ca2+ exchanger

Plasma membrane Ca2+ ATPase

Sarcoplasmic/endoplasmic reticulum Na+/Ca2+ exchanger

5. Ventricular systole

Begins with the isometric relaxation phase

Lasts about 0.5 s in healthy adults at rest

Lasts longer than atrial systole. Can coincide with atrial systole in complete heart block

6. The purkinje tissue cells in the heart :

conduct impulses faster than some neurons


are larger than ventricular myocardial cells

lead to contraction of the base before the apex of the heart

are responsible for the short duration of the QRS complex

are responsible for the configuration of the QRS complex

7. Anincrease inplasmaosmolarity stimulates release of what hormone from the posterior pituitary?

Renin

Antidiuretic hormone (ADH)

Aldosterone

Angiotensin II

8. The long-term mechanism for regulation of blood pressure involves regulating which of the following?

Vessel diameter

Blood volume

Contractility

Heart rate

9. Which of the following is true of ventricular diastole?

Atrioventricular valves are open

Semilunar valves are open

Atria contract all throughout this phase

10. Peripheral resistance is maximum in:

Aorta

Artery

Arteriole

Vein

11. Both atrioventricular valves , aortic pulnionary valve are all closed in

Atrial systole

isometric ventricular relaxation, Isometric ventricular contraction

Ventricular ejection

Atrial diastole
12. When the bundle of His is completely interrupted,the:

ventricles contract at a rate of 30-40 beats / minutes

atria beat irregularly

QRS complexes vary in shape from beat to beat

P-R interval remains constant from beat to beat

S.A node stops discharging

13. This famous physiologist discovered the circulation of blood:

Ernest Starling

William Harvey

Marcello Malpighi

Andreas Vesalius

14. An acute decrease in arterial blood pressure elicits which of the following compensatory changes?

Decreased firing rate of the carotid sinus nerve

Increased parasympathetic outflow to the heart

Decreased heart rate

Decreased contractility

Decreased mean systemic pressure

15. In human being the duration of cardiac cycle is ___________

0.008 sec

0.5 sec

0.8 sec

8 sec

16. Ventricular pressure is higher than the atrial pressure in all phases of the cardiac cycle Except in:

isometric contraction phase.

atrial systole phase.

maximum ejection phase.

reduced ejection phase

17. If ST-elevation was noted in leads II, III and aVF, what would it suggest?
A septal myocardial infarction

An inferior myocardial infarction

An anterior myocardial infarction

A posterior myocardial infarction

18. Diacrotic notch is due to:

sudden closure of AV valves.

sudden closure of aortic valve.

sudden decrease in aortic pressure.

marked decrease in ventricular pressure

19. A change in blood pressure that decreases the impulses to the cardiovascular center results in increased impulses from
which branch of the autonomic nervous system? What would be the effect on blood pressure (BP)?

Sympathetic; increased BP

Parasympathetic; increased BP

Sympathetic; decreased BP

Parasympathetic; decreased BP

20. The ventricular repolarisation in ECG is best seen in:

"P" wave

"Q" wave

"R" wave

"T" wave

21. The greatest pressure decrease in the circulation occurs across the arterioles because:

They have the greatest surface area

they have the greatest cross-sectional area

the velocity of blood flow through them is the highest

the velocity of blood flow through them is the lowest

they have the greatest resistance

22. What view of the heart do leads V3 and V4 represent?

Septal

Inferior
Anterior

Lateral

23. An electrocardiogram is a graphic illustration of ____________

cardiac conduction system

cardiac cycle

cardiac output

systemic and pulmonary circuits

24. Of the conductive tissues of the heart, only these cells can directly stimulate cardiac myocytes to contract:

Cells of the sinoatrial (SA) node

Cells of the atrioventricular (AV) node

Fibers of the bundle of His

Fibers of the left and right bundle branches

25. Which artery is most likely to be affected in the context of ST elevation being present in leads V3 and V4?

Right coronary artery

Left circumflex coronary artery

All of the above

Left anterior descending coronary artery

26. Starling's law of the heart

Does not operate in the failing heart

Does not operate during exercise

Explains the increase in cardiac output that occurs when venous return is increased

Explains the increase in cardiac output when the sympathetic nerves supplying the heart are stimulated

27. Which of the followings is true?

Starling's law of heart states that increase in force of contraction is directly related to cardiac output

Starling's law of heart states that the force of ventricular contraction is directly related to the end diastolic volume

Both A and B

None of the above

28. What view of the heart do leads II, III and aVF represent?
Lateral

Inferior

Anterior

Septal

29. What is the duration of a normal PR-interval?

0.08 - 0.12 seconds (2-3 small squares)

0.04 - 0.12 seconds (1-3 small squares)

0.12 - 0.2 seconds (3-5 small squares)

0.04 - 0.08 seconds (1-2 small squares)

30. The cardiac events that occur from the beginning of one heart beat to the beginning of the next heart beat are called the
cardiac cycle. Which of the following is/are not true?

The left atrium and right atrium contract nearly simultaneously.

The left ventricle and right ventricle contract nearly simultaneously.

The atria contract before the ventricles.

The ventricles contract before the atria.

31. The SA node is the normal pace maker because :

is the most rapidly discharging part

is the most richly supplied by nerve endings .

located in the atrium .

all of the above

32. The action potential of cardiac muscle differs from that of skeletal muscles in:

it is propagated more slowly .

it is shorter in duration

it has a higher amplitude

it has no plateau

all of the above

33. The aortic arch and the carotid sinus contain nerve endings that monitor blood pressure by the degree of stretch on the
vessel wall. These sensors are called:

Atrial baroreceptors
Arterial baroreceptors

Osmoreceptors

34. During the isovolumic contraction (also referred to as isovolumetric contraction) phase of the cardiac cycle:

The atrioventricular valves and semilunar valves are closed.

The atrioventricular valves and semilunar valves are open.

The atrioventricular valves closed and semilunar valves are open.

The atrioventricular valves open and semilunar valves are closed.

None of the above.

35. Under normal conditions, the capillaries :

contain about 25% of the total blood volume .

have a very rapid flow rate

have a higher pressure than that in the arterioles

are all opened at rest

contain about 5 % of the total blood volume

36. An ECG is performed and reveals a progressively increasing PR interval and dropping of QRS complexes at regular intervals.
Which of the following is the most likely diagnosis?

Second-degree heart block (Mobitz type 2)

Second-degree heart block (Mobitz type 1)

Hyperkalaemia

First-degree heart block

37. An ECG reveals an absence of P-waves and an irregular rhythm. Which of the following is the most likely diagnosis?

Atrial fibrillation

2nd-degree heart block

1st-degree heart block

Ventricular tachycardia

38. The left ventricle has a thicker wall than the right ventricle because :

it is richer in blood supply

it ejects blood through a narrower orifice

it ejects a greater cardiac output .


it ejects blood against a higher pressure

it contracts at a higher rate

39. During a cardiac myocyte contraction, some of the Ca2+ necessary for contraction comes from the extracellular fluid.
During relaxation, this Ca2+ will have to be extruded from the cell. Most of this Ca2+ is removed from the cell due to the activity
of the:

Plasma membrane Na+/Ca2+ exchanger

Plasma membrane Ca2+ ATPase

None

40. Pacemaker potentials of the heart result from:

Na+ influx

K+ influx

Cl− influx

Na+ efflux

41. In a normal heart at rest the LV end-systolic volume is:

10 to 30 ml

50 to 70 mls

120 to 150 ml

80 - 100 ml

42. During a typical cardiac cycle under resting conditions, approximately __________ of the volume of blood present in the
ventricles just before ventricular contraction (referred to as enddiastolic volume), enters the ventricles as a result of atrial
contraction.

10%

20%

50%

80%

43. The plateau phase of the cardiac action potential is due to the:

opening of voltage-gated Ca2+ channels

opening of fast voltage-gated Na+ channels

opening of voltage-gated K+ channels

44. Cardiac output is determined by _______


heart rate

stroke volume

blood flow

heart rate and stroke volume

45. During the cardiac cycle, opening of the atrioventricular valves following ventricular relaxation leads to:

an increase in the hydrostatic pressure within the ventricles

a decrease in the hydrostatic pressure within the atria

an increase in the hydrostatic pressure within the atria

46. The sympathetic nervous system also stimulates the release of epinephrine and norepinephrine from what gland?

Pituitary gland

Thyroid gland

Adrenal gland

47. Isometric ventricular contraction

Begins when semilunar valves open

Begins when semilunar valves open

Lasts for about 0.5 s

Is a phase where all four cardiac values closed. Causes a sharp increase in atrial pressure

48. During the isovolumic relaxation (also referred to isovolumetric relaxation) phase of the cardiac cycle:

The atrioventricular valves and semilunar valves are closed.

The atrioventricular valves and semilunar valves are open.

The atrioventricular valves closed and semilunar valves are open.

The atrioventricular valves open and semilunar valves are closed.

49. The spike phase of the action potential of the sinoatrial node pacemaker cells of the heart is caused by:

Opening of voltage-gated Na+ channels

Opening of voltage-gated Ca2+ channels

Closure of voltage-gated K+ channels

Opening of voltage-gated Cl− channels

Closure of voltage-gated Ca2+ channels


50. The largest total cross-sectional area of blood vessels is found in this region along the vasculature:

Small arteries

Arterioles

Capillaries

Venules

51. Which mechanisms of these acts when blood flow to brain decreased?

Chemoreceptor reflexes.

Baroreceptor reflexes.

CNS ischemic response.

Atrial regulation.

52. What factor makes it easy for chemoreceptors cells to detect changes in O2, CO2, and H+?

The blood volume

The pulse rate

The speed of the blood flow

The venous return effectiveness

53. Increased activity of the vagus nerve would cause the heart rate to

Increase

Decrease

54. About the cardiac conductivity, all the following are true Except :

it is slowest in the A-v node

it is maximal in the purkinje fibers .

it is slowest in the ventricular ms.

it is decreased by vagal stimulation

it occurs through modified cardiac muscle fibers

55. If there were 3 large squares in an R-R interval what would the heart rate be?

100 bpm

90 bpm

80 bpm
70 bpm

56. During hemostasis the reflex Compensation of the blood flow to the Heart is:

Increase blood flow to the Heart but not normal

Decrease blood flow to the Heart but not normal

Blood flow will remain the same

Nothing will change in blood flow

57. In a typical human cardiac cycle, the volume of blood remaining in the left ventricle at the end of ventricular systole is
approximately __________. This is referred to as the end-systolic volume.

10 mL

25 mL

50 mL

100 mL

58. In cardiac muscle cells, approximately what percentage of cytoplasmic Ca2+ required for contraction enters the cardiac
muscle cell from the extracellular fluid?

0%

10%

20%

80%

59. Name the three phases of cardiac cycle in the order ____________

early diastole, mid-to-late diastole, ventricular systole

ventricular systole, mid-to-late diastole, Early diastole

mid-to-late diastole, Ventricular systole, Early diastole

early diastole, Ventricular systole, mid-to-late diastole

60. AV valves are opened in:

isometric contraction phase.

atrial systole phase.

protodiastolic phase.

maximum ejection phase.

61. Which of the following parameters is decreased during moderate exercise:


Arteriovenous O2 difference

Heart rate

Cardiac output

Pulse pressure

Total peripheral resistance (TPR)

62. In an individual at rest, approximately this percentage of total blood volume can be found in all capillaries:

5%

10%

35%

65%

63. The absolute refractory period in the ventricle :

increases during sympathetic stimulation

coincides with the phase of rapid depolarization and the plateau

corresponds with the whole duration of action potential

it is shorter than the ARP of skeletal muscle

64. The atrio-ventricular valves :

have three cusps for each valve

their closure is initiated when the ventricular pressure exceeds atrial pressure

open by contraction of papillary muscles

none of the above

65. About the cardiac muscle all the following are true Except :

It has low electric resistance of the membrane at the intercalated discs

It forms true syncytium

It obeys the all or none law

there is almost a special capillary for each muscle fiber

66. Which of the following is not true for ventricular systole?

The ventricles relax

The ventricles contract


The semi lunar valves close

The atrioventricular valves open

67. All the following are correct except :

the left atrial wall is about the same thickness as the right atrium

the left ventricle is separated from the left atrium by a bicuspid valve

Purkinje system allows the excitation of all ventricular muscle fibers to occur at nearly the same time

the right heart receives oxygenated blood from the four pulmonary veins

68. These cells are known as the primary pacemakers of the heart:

Cells of the sinoatrial (SA) node

Cells of the atrioventricular (AV) node

Fibers of the bundle of His

Fibers of the left and right bundle branches

69. Minimum blood Pressure is in:

Aorta

Arteries

Capillaries

Venules

70. The action potential of the cardiac muscle is characterized by the presence of plateau which:

in the ventricular muscle it reached 100 m sec

in the artial muscle, it reached 300 m sec

it result from a balance between Ca++ influx and K+ efflux

it result from an inflow of Cl- and outflow of bicarbonate ions

71. Which one of these is the effect of angiotensin II ?

Vasoconstriction of afferent artery.

Vasodilation of efferent artery.

Vasoconstriction of efferent artery.

Vasoconstriction of both efferent and afferent.

72. In cardiac muscle cells, approximately what percentage of cytoplasmic Ca2+ required for contraction is released from the
sarcoplasmic reticulum?

0%

10%

20%

80%

73. In a normal cardiac cycle (same as CV01 but we remembered the options as)

RA ejection precedes LA ejection

RV contraction starts before LV contraction

LV ejection starts before RV ejection

Pulmonary valve closes before aortic valve

Aortic valve closes after pulmonary valve in expiration

74. Parasympathetic stimulation of the heart has the following effect(s):

Negative chronotropic effect

Positive chronotropic effect

Negative inotropic effect

Positive inotropic effect

75. What is the normal duration of a QRS complex?

0.16 seconds (4 small squares)

0.04 seconds (1 small square)

0.12 seconds (3 small squares)

0.08 seconds (2 small squares)

76. Left ventricular end-diastolic volume is:

10-30 mls

50-70 mls

30-50 mls

100-130 mls

77. The cardiac tissue with the slowest rhythmicity is the :

S.A node without vagal tone

S A node with vagal tome


S.A node with vagal tome

ventricular muscle

AV node

purkinje fibers

78. Regarding the S-A node :

cells within the S-A node act as heart pace maker because their membrane depolarized to threshold and initiate an action
potential

acetyl choline increases the slope of the pace - maker potential .

sympathetic stimulation decreases the slope of the pace - maker potential .

the pace - maker cells within the S-A node are neurons rather than myocytes

79. During exercise,total peripheral resistance(TPR)decreases because of the effect of:

the sympathetic nervous system on splanchnic arterioles

the parasympathetic nervous system on skeletal muscle arterioles

local metabolites on skeletal muscle arterioles

local metabolites on cerebral arterioles

histamine on skeletal muscle arteriole

80. The cardiac events that occur from the beginning of one heart beat to the beginning of the next heart beat are called the
cardiac cycle. During the cardiac cycle:

The atria contract before the ventricles.

The atria and the ventricles contract simultaneously.

The ventricles contract before the atria.

81. During a typical cardiac cycle under resting conditions, approximately __________ of the volume of blood present in the
ventricles just before ventricular contraction (referred to as enddiastolic volume), enters the ventricles as a result of passive
venous return of blood flowing through the atria and into the ventricles.

10%

20%

50%

80%

82. The SA node is the normal pacemaker because of its :

rate of impulse discharge

location in the atrium


neural control

muscular structure

relative position to the A.V node

83. In a typical ventricular myocyte, the refractory period is approximately:

1 ms

5 ms

10 ms

100 ms

250 ms

84. Which of the following is a common cause of right axis deviation?

Ventricular septal defect

Right ventricular hypertrophy

Atrial septal defect

Left ventricular hypertrophy

85. Sinoatrial node cells are :

found in both atria

innervated by the vagus

unable to generate impulses when completely denervated

connected to AV node by fine bundles of purkinje tissue

stable with regard to their membrane potentials

86. In exercising muscle, the major increase in blood flow is due to:

Sympathetic vasodilatation

Metabolic vasodilatation

Muscle pumping

87. What cells in the kidney monitor blood pressure, releasing renin when blood pressure is low?

Cortical cells

Angiotensinogen cells

Aldosterone cells
Juxtaglomerular cells

88. At which site is systolic blood pressure the highest?

Aorta

Central vein

Pulmonary artery

Right atrium

Renal artery

Renal vein

89. The best site to measure mixed venous pO2 is:

SVC

RA

Pulmonary artery

Pulmonary vein

90. During the isovolumic relaxation (also referred to as isovolumetric relaxation) phase of the cardiac cycle:

The hydrostatic pressure in the left ventricle is higher than the hydrostatic pressure in the left atrium.

The hydrostatic pressure in the left ventricle is lower than the hydrostatic pressure in the left atrium.

The hydrostatic pressure in the left ventricle is higher than the hydrostatic pressure in the aorta.

91. An increase in arteriolar resistance, without a change in any other component of the cardiovascular system, will produce:

A decrease in total peripheral resistance (TPR)

An increase in capillary filtration

An increase in arterial pressure

A decrease in afterload

92. The S.A node :

but not the A.V node receives a parasympathetic nerve supply

is connected by a thin band of purkinje fibers to the A.V node

has a faster rate of diastolic depolarization during vagal activity

has a faster rate of diastolic depolarization at 39C than at 37C

is a secondary cardiac pacemaker


93. A patient is noted to have an abnormally shortened PR-interval on their ECG. Which of the following is the most likely
cause?

Right bundle branch block

Left bundle branch block

Atrioventricular nodal fibrosis

Wolff-Parkinson-White (WPW) syndrome

94. What is the most common cause of left axis deviation?

Defects of the conduction system

Atrial septal defects

Left ventricular hypertrophy

Right ventricular hypertrophy

95. Cells of the sinoatrial node are the primary pacemakers of the heart. In the absence of any input from the autonomic
nervous system, how many action potentials do these cells generate every minute?

None

30 action potentials per minute

70 action potentials per minute

100 action potentials per minute

96. During the cardiac cycle, contraction of the left atrium leads to an increase of approximately __________ in the hydrostatic
pressure within the left atrium.

1-2 mm Hg

4-6 mm Hg

7-8 mm Hg

10-12 mm Hg

97. A change in blood pressure that increases the impulses to the cardiovascular center results in increased impulses from
which branch of the autonomic nervous system? What would be the effect on blood pressure (BP)?

Parasympathetic; increased BP

Parasympathetic; decreased BP

Sympathetic; decreased BP

Sympathetic; increased BP

98. Increased sympathetic nerve activity at the heart would cause:


An increase in heart rate only

An increase in both heart rate and contractility

A decrease in heart rate and contractility

An increase in contractility only

99. If the heart rate is 70 beats per minute, what is the total duration (in milliseconds, ms) of any individual cardiac cycle?

429 ms

583 ms

857 ms

1167 ms

100. In this case above the baroreceptors will not send the inhibitory impulses to the:

Vasomotor center

Vagal center

Vasoconstrictor center

101. Choose the correct sequence of action potential conduction along the conductive tissues of the heart:

SA node → AV node → Bundle of His → Purkinje fibers → Bundle branches

SA node → AV node → Bundle of His → Bundle branches → Purkinje fibers

SA node → AV node → Bundle branches → Bundle of His→ Purkinje fibers

SA node → AV node → Purkinje fibers → Bundle of His → Bundle branches

102. The P wave of the ECG represents _______________

ventricular depolarization

atrial depolarization

ventricular repolarization

atrial systole

103. The volume of blood each ventricle pumps out during a cardiac cycle is about ___________

70 ml

5000 ml

7L

1200 ml
104. What view of the heart do leads V1 and V2 represent?

Inferior

Septal

Anterior

Lateral

105. The fibers of the A-V bundle & its branches :

conduct impulses very slowly

are highly contractile

are modified muscle fibers

are nerve fibers

106. What view of the heart do leads I, aVL, V5 and V6 represent?

Lateral

Septal

Anterior

Inferior

107. Within the cardiovascular system, the highest hydrostatic blood pressure is found in the:

Left ventricle during ventricular systole

Left ventricle during ventricular diastole

Left ventricle during atrial systole

Left ventricle during atrial diastole

108. Cardiac output of the right side of the heart is what percentage of the cardiac output of the left side of the heart:

25%

50%

75%

100%

125%

109. In an individual at rest, approximately this percentage of total blood volume can be found in the venous system:

5%

10%
10%

35%

65%

110. What does ST-elevation suggest?

Bundle branch block

Heart murmur

Myocardial infarction

Heart block - Mobitz type II

111. The tendency for blood flow to be turbulent is increased by?

increased viscosity

increased hematocrit

partial occlusion of a blood vessel

decreased velocity of blood flow

112. The longest phase in cardiac cycle is:

maximum ejection phase.

reduced filling phase.

maximum filling phase.

protodiastolic phase.

113. All are involved in ventricular filling except:

atrial systole phase.

maximum ejection phase.

maximum filling phase.

reduced filling phase.

114. The ‘c’ wave in the JVP corresponds most closely with:

Peak aortic flow

Isovolumetric contraction

Isovolumetric relaxation

Closure of aortic valve

Closure of mitral valve


115. What is the definition of cardiac cycle?

The contraction of the atria

Circulation of the blood in the heart

The contraction and relaxation of the ventricles

It is a sequence of event that occurs during one complete heartbeat

116. In the heart all the following are true Except :

the right ventricle is thicker than the left ventricle

the excitation wave cannot spread directly from the atria to the ventricles

contraction normally begins in the right atrium

the ventricles contract almost stimultaneously

the S.A node is the normal pacemaker

117. Sympathetic stimulation of the heart has the following effect(s):

Negative chronotropic effect

Positive chronotropic effect

Negative inotropic effect

Positive inotropic effect

B and D

A and C

118. Which cannot be measured by spirometry:

Tidal Volume

Residual Volume

Inspiratory Reserve Volume

Expiratory Reserve Volume

119. To which of the following is alveolar PCO2 directly proportional?

Rate of CO2 production and alveolar ventilation.

Rate of CO2 production and rate of O2 consumption.

Alveolar ventilation and rate of O2 consumption.

Alveolar ventilation, rate of O2 consumption, and rate of CO2 production.


120. What type of process occur in combination of Oxygen with Hemoglobin:

Oxidation

Oxygenation

Oxyhemoglobin

Reduction

121. Which of the following is not a function of the lung?

All are functions of the lung

Condensation of water vapor from expired air

Cool down air

Metabolism of substances

122. In the start of Inhalation, the pace maker should be switched on. Where is that stimulator that swiches it on or stimulates it
and what’s it’s name?

apneustic centre; located in lower medulla

apneustic center; located in dorsal medulla

pneumotoxic centre; upper part of pons

pneumotoxic centre; lower part of pons

123. Which of the following concerning average lung volumes and capacities of a person at rest is TRUE?

TLC>VC>TV>FRC

TLC>FRC>VC>TV

TLC>VC>FRC>TV

TLC>FRC>TV>VC

124. If the lung were punctured, which of the following would happen?

The lung would collapse on the side of the puncture.

Both the lung and the chest wall would collapse on the side of the puncture.

The relaxation pressure of the chest wall would increase until it surpassed the atmospheric.

The relaxation pressure of the chest wall would increase, but stop before it reached atmospheric pressure.

125. The Oxygen is transported from lungs to tissues:

all the O2 gas is transported to peripheral tissues in combination with Hb


almost all amount of O2 in combination with Hb and some also as dissolved in the blood

only with Hb, O2 can be transferred from lungs to tissues

Both a and b

126. When the hemoglobin combines with carbon monoxide:

Blood viscocity is increased

It loses the capacity to transport oxygen

RBCs are destroyed

Bleeding does not stop

127. Which of the following will NOT increase the minute ventilation?

An increase in arterial pH.

An increase in arterial partial pressure of carbon dioxide.

Increase in alveolar pressure of carbon dioxide.

Exercise.

Hypoxia.

128. What is Blood brain barrier?

a barrier between blood and brain parts and is highly effective for O2 as it is the need of brain

a barrier of very tightly placed endothelial cells in the brain through which the transfer of O2 becomes very difficult and CO2
passes very easily

a highly lipid unsoluble barrier between brain and blood

a lipid and protein unilayered barrier between brain and blood

129. Which of the following will readily diffuse across the blood brain barrier?

CO2

H+

HCO3-

H2CO2

130. What happens in Emphysema?

The air can not be recoiled due to the lost of elasticity of the alveolar tissue

The patient will see the shortness of breath because of less-inflation of the alveolar tissue

The short breathing is due to destruction of the air passage ways of the bronchial tree
The lobes of the lungs are damaged and it causes the shortness of breath

131. Which of the following is FALSE concerning the relationships of the variables in diffusion of O2 across a membrane?

Doubling the thickness of the membrane would cut the total flow of O2 in half.

Doubling the area of the membrane would double the total flow of O2 .

If you increased the alveolar concentration of O2, you would increase the total flow of O2 across the alveolar membrane.

The lower the diffusion coefficient, the higher the total flow. E. Increasing the arterial concentration of O2 would decrease the
total flow of O2.

132. Repeated cycles of respiration cannot be recorded by using spirometer because:

CO2 accumulates in the spirometer

Fresh air or oxygen can not be provided to the patient

Respirometer can easily record

both a and b

133. Asthma is a occurred due to:

Anemic Hypoxia

Histotoxic hypoxia

Hypoxic Hypoxia

Ischemiac Hypoxia

134. Where is the Pace Maker of Respiratory system located?

In the alveoli

In the lobes of Lungs

Medulla

In the Respiratory Membrane

135. Which is not the delayed effect of Hypoxia?

Vomitting

Depression

Fatigue

Itching

136. Spirometer is used for:


one breath

two breaths

three breaths

from one to five breaths

137. Even in the presence of enough oxygen, which of the following disorder cause Hypoxic hypoxia:

Pneumothorax

Ischemic Heart Disease

Parkinson's disease

Diabetes mellitus

138. What is hypoxia?

Reduced supply of O2 at cellular level

Reduced supply of O2 at tissue level

Reduced supply of O2 to brain

Reduced supply of O2 due to excess of CO2

139. Oxygen therapy has maximum efficacy in:

Stagnant Hypoxia

Hypoxic Hypoxia

Histotoxic Hypoxia

Anemic Hypoxia

140. Which of the following is TRUE if a patient breathes slower than normal with increased tidal volumes?

More resistive work is done.

The total work done decreases.

More elastic work is done.

Compliance is decreased.

141. Which cannot be measured by spirometry:

VC and FRC

FRC and TLC

IC and VC

TLC and IC
Ca d C

142. What is incorrect about Pulmonary function Tests?

They can differentiate the obstructive and restructive diseases

They are used in the assessment of functional status of lungs

Plethysmography is a technique used to measure all the lung volumes and capacities

Spirometery is a technique used in for pulmonary function tests

none of the above

143. When inhaling dust or pungent odor, reflexes occur:

protective

Goering-Breuer;

from chemoreceptors of the vascular zones;

from the baroreceptors of the vascular reflexogenic zones.

144. Increased physiological dead space with

Age

Anaesthesia

Sleeping position

All of the above

145. Which is the correct formula of Henry’s law:

Concentration of dissolved gas = Partial pressure / Solubility coefficient

Concentration of dissolved gas = Solubility coefficient / Partial pressure

Partial pressure =Concentration of dissolved gas / Solubility coefficient

Partial pressure= Solubility coefficient/ Concentration of dissolved gas

146. Which of the following does NOT apply to the alveoli at the base of the lungs?

They are less elastic than the alveoli at the apex.

The pleural pressure is lower.

At FRC they are less inflated than the alveoli at the apex.

They are closed at RV.

They have a greater volume change than alveoli at the apex during inspiration from FRC.
147. Compare the Partial Pressures of the O2 ad CO2 gases both in the arterial and venous systems:

On arterial side O2 has 40mmHg and CO2 has 97mmHg; On venous side O2 has 46mmHg and CO2 has 40mmHg

On arterial side O2 has 60mmHg and CO2 has 90mmHg; On venous side O2 has 52mmHg and CO2 has 48mmHg

On arterial side O2 has 78.5mmHg and CO2 has 29.7mmHg; On venous side O2 has 35.4mmHg and CO2 has 82.3mmHg

On arterial side O2 has 97mmHg and CO2 has 40mmHg; On venous side O2 has 40mmHg and CO2 has 46mmHg

148. From C3,C4&C5 the phrenic nerve originates, and it supplies the diaphragm and from T1-T11 the intercostal nerves are
originated which are the nerve impulse suppliers of the external intercostal muscles. Can you tell among these two, which other
muscle is involved in inhalation?

Scalene

Trapezius

Rhomboid Major

Rhomboid Minor

149. Alveoli in healthy people do not stick together when falling, because:

there are elastic fibers in the wall of the alveoli;

in the alveoli there is a surfactant that reduces surface tension

pleural sheets have the ability to absorb water

they have a water film;

150. Which of the following is FALSE concerning the ventilation and perfusion of different regions of the lung?

Alveoli at the top of the lung have a smaller dynamic compliance.

The Hb moving through the base of the lung is less saturated than that at the apex of the lung.

PAO2 at the apex of the lung is higher than that at the base of the lung.

Regional variation in ventilation-perfusion is more efficient for oxygenating blood than is uniform ventilation-perfusion.

Variation of the ventilation/perfusion ratio in the lungs only becomes significant when lung function begins to degrade.

151. All of the following statements decrease the lung compliance except:

Lung fibrosis

Increased pulmonary venous pressure

long period of time where the lung is unventilated

Emphysema

152. Which of the following is FALSE concerning the production and role of lung surfactant?
It is part of a lipoprotein called dipalmitoyl phosphatidyl-choline.

It is synthesized by alveolar type II cells.

As the alveolar surface area decreases during the compression curve, the surfactant decreases the surface tension at a
constant rate.

When surfactant density is decreased during expansion, surface tension initially rises rapidly, then slows down until it reaches
the starting point.

153. What will happen when the alveolar pressure of oxygen is lowered?

The arterial pressure of oxygen will be increased because of rapid blood flow

The arterial pressure of oxygen will be decreased

There will be no a markable change in the functioniong of heart

154. The pressure in the pleural cavity during normal inspiration is:

6, - 9 mm Hg. v .;

3, - 4 mm Hg. v .;

15, - 20 mm Hg. v .;

1, 0 mm Hg. v.

155. Which of the following is NOT true at Functional Residual Capacity (FRC)?

It is about 75% TLC.

The elastic recoil of the chest wall is outward.

The elastic recoil of the lung is inward.

The relaxation pressure of the lung and chest wall combined is at atmospheric pressure.

156. Oxygen therapy is moderately effective in:

Stagnant hypoxia

Histotoxic Hypoxia

Anemic Hypoxia

Hypoxic Hypoxia

157. Hypoxia turns urine:

Acidic

Alkaline
Neutral

No change

158. The Alveolar pressure is the:

pressure of the air inside the lung alveoli

pressure of the air applied from outside to alveoli

pressure of the fluids inside the alveoli

pressure on the walls of the alveoli by the thickening due to low arterial pressure

159. Which of the following is inactivated in the lung?

Angiotensin II

Angiotensin I

Bradykinin

Vasopressin

Noradrenaline

160. Which of the following in FALSE concerning CO2 uptake?

If PO2 = PCO2, then there will be more total CO2 in the blood.

Oxygenation moves the CO2 uptake curve downward.

The CO2 uptake curve is generated by comparing the total CO2 per unit volume of blood, and the PCO2.

Deoxygenated blood carries less CO2 than oxygenated.

161. The term which is defined as the volume remaining in the lungs after normal exhalation and is calculated by adding the
expiratory reserve volume and the residual volume:

functional residual volume

residual volume

tidal volume

vital capacity

162. Which of the following is FALSE concerning airway resistance?

Up to 50% is in the nose.

The maximum resistance in the bronchial tree occurs at the fourth generation.

In the later generations, the radii are smaller, increasing the total resistance at each successive generation.

Airway resistance can be increased by loss of tissue elasticity and contraction of bronchial smooth muscles
Airway resistance can be increased by loss of tissue elasticity and contraction of bronchial smooth muscles.

163. In Congestive Cardiac Failure, Hypoxia happens because:

Oxygen is not available

Diffusion is not taking place

Blood cannot be pumped from heart

Breathing is obstructed

164. Which muscles are involved in inspiration?

Diaphragm

Diaphragm and external intercostal muscles and abdomial muscles

Diaphragm and External intercostal muscles

Diaphragm and Internal intercostal muscles

165. Which of the following is FALSE concerning the airflow in the lungs?

During inspiration and expiration, the flow in the trachea and larger bronchi is turbulent.

Towards the middle of the bronchial tree, the flow is turbulent at the branches and laminar in between.

Near the end of the bronchial tree, the flow is laminar.

The acini have very small radii which significantly increases the total air flow resistance of the bronchial tree.

166. How many there are total number of secondary bronchi in the human respiratory system?

None

167. Which one work together to ensure that our body cells are well supplied with oxygen and are able to give up CO2:

Lungs

Heart

Blood Vessels

All of these

168. Which is composed of a respiratory bronchiole, alveolar ducts, atria, and alveoli

Bronchial plant
Respiratory Unit

Lobes of Lungs

Respiratory Centre

169. Which of the following represents the pressure difference that acts to distend the lungs?

Alveolar pressure

Airway opening pressure

Transthoracic pressure

Transpulmonary pressure

Esophageal pressure.

170. The value of normal Functional Residual Capacity is:

3800ml

2200ml

4800ml

1200ml

171. What happens in COPD?

There is a chronic dysfunction of the respiratory centre

The passage ways are damaged

The air passage ways are more widened

The respiratory membrane is damaged

172. Residual volume of the lung is:

1200ml

1500 ml

4500 ml

500 ml

173. Which of the following is correct?

VC is increased in Asthama

VC is is decreased in sedentary workers

VC is less in athletes

VC is decreased in Pulmonary edema


C s dec eased u o a y ede a

174. . Which of the following pairs is NOT a pulmonary mechanoreceptor paired to a possible stimulus?

Stretch receptor: inflation

Irritant receptor: inhaled dust

Juxtacapillary receptors: decreases interstitial fluid volume in alveolar walls.

Bronchial C receptors: large inflations.

175. Which of the following is not a function of the respiratory system?

transports O2 to the tissues.

contributes to maintenance of normal acid- base balance.

provides a route for heat and water elimination.

enables speech, singing, and other vocalization.

removes, modifies, activates, or inactivates various materials passing through the pulmonary circulation.

176. Juxtacapillary receptors: decreases interstitial fluid volume in alveolar walls

alveolar sacs

Alveolar ducts

Terminal bronchioles

Respiratory bronchiole

177. Which of the following is INCORRECT concerning the O2 / CO2 movement and processing through the lungs and tissues?

Binding of O2 to Hb changes its configuration so that CO2 and H+ ions are more likely to dissociate.

When CO2 diffused into the alveoli, the PaCO2 is lowered.

Carbonic acid is an intermediate in the reaction combining H+ with HCO3- to form H2O and CO2.

Arterial blood flows to the tissues where H+ ions combine with HCO3- to form H2O and CO2.

178. Oxygen therapy remains useless in:

Histotoxic hypoxia

Anemic Hypoxia

Stagnant Hypoxia

Hypoxic Hypoxia

179. During inspiration, how does alveolar pressure compare to atmospheric pressure?

Al l i h h i
Alveolar pressure is greater than atmospheric.

Alveolar pressure is less than atmospheric.

Alveolar pressure is the same as atmospheric.

Alveolar pressure is one of the few pressures where the reference pressure is not atmospheric.

180. The entire sequence of events involved in the exchange of O2 and CO2 between the basic portion of lungs and the
atmosphere is known as:

Ventilation

External Respiration

Breathing

InternalRespiration

181. In Bronchiectasis:

less Oxygen is diffused in blood

Blood becomes thick

RBCs rupture

Cells are unable to pick oxygen from blood

182. If you blocked the blood supply to an alveolus, which of the following would NOT occur as a result?

The ventilation perfusion ratio would be 0.

The PAO2 would be greater than normal.

The PACO2 would be 0.

All of the above are true.

183. In the start of Exhalation, the pace maker of respiration should be switched off. Where is that inhibitor that switches it off
and what’s it’s name?

pneumotoxic centre; upper part of medulla

apneustic centre; lower part of pons

pneumotoxic centre; upper part of pons

apneustic centre; dorsal part of medulla

184. Reduced availability of oxygen to the tissues is:

Anemia

Acidosis

Hypoxia
Ischemia

185. Which of the following substances is activated by passage through the pulmonary circulation?

Bradykinin

Serotonin

Noradrenaline

Angiotensin 1

186. Which of the following is the primary regulating variable of the central chemoreceptors?

PaO2.

PaCO2

arterial pH

Input from stretch receptors.

187. Spirometry can measure all except:

IC

ERV

FRC

Vital Capacity

TV

188. The example of Hyperventilation is:

breathing after a running

Asthma

short breathing

exhaling less than you inhale

189. Tidal volume in man is:

1200ml

1500ml

4500ml

500ml
190. In a normal person, in 100 ml of blood what is the volume of CO2 in arterial blood and venous blood?

In arterial blood, 52ml/dl; In venous blood, 48ml/dl

In arterial blood, 48ml/dl; In venous blood, 52ml/dl

In arterial blood, 46ml/dl; In venous blood, 54ml/dl

In arterial blood, 54ml/dl; In venous blood, 46ml/dl

191. Which of the following can cause stagnant hypoxia?

COPD

Shock or heart failure

Cyanide poisoning

Carbon monoxide poisoning

192. Which of the following structures serves as a common passageway for both the respiratory and digestive systems?

nose

pharynx

trachea

bronchi

esophagus

193. There are about:

500 million alveoli in the two lungs

400 million alveoli in the two lungs

300 million alveoli in the two lungs

200 million alveoli in the two lungs

194. In the hemoglobin, the haem is present between the:

glycosidic chains

peptide chains

ester chains

all of the above chains of Hb have haem

195. Which activity below is not a respiratory event?

Carbon dioxide is exchanged in the alveoli.


Cells produce nitrogen by their metabolism.

Gas exchange between tissues and the blood.

Oxygen is exchanged in the alveoli.

Pulmonary ventilation.

196. Which of the following is incorrect?

In females, vital capacity is less than in males

Vital capacity is more in standing position and less in lying position

Vital capacity is decreased in bugle players

Vital capacity is slightly more in heavily built persons

197. The group of neurons present in the RETICULAR FORMATION on the postero-dorsal part of MEDULLA OBLONGATA are
mainly concerned with the:

Expiration

Inspiration

Drainage of CO2

Respiration

198. Cardiac disorders in which enough blood is not pumped to transport oxygen, lead to:

Histotoxic Hypoxia

Anemic Hopoxia

Hypoxic Hypoxia

Stagnant Hypoxia

199. Which of the following is not used for studying respiratory abnormalities, measuring:

Physiological Dead Space

Functional Residual Capacity

Ventilation / Perfusion Ratio

None of the above

200. The amount of air inhaled or exhaled in one breath during related, quiet breathing is:

functional residual capacity

residual volume

tidal volume
tidal volume

total lung capacity

201. Hypoxia is associated with which of the following usual symptoms:

Loss of appetite, Nausea, vomiting, dry mouth

Polyphagia, Constipation, blurred vision

Itching, sneezing, lacrimation, cough

None of the above

202. Which of the following is not an effector of respiratory system?

blood circulatory system

diaphragm

intercostals

trapezius

203. In order to determine the lung volumes and capacities, following levels are to be noted in spirogram: I. Normal end
expiratory level II. Normal end inspiratory level III. No. of breaths

I, II and III

I and III

I and II

204. Which of the following pairs is INCORRECT concerning central nervous systems and a factor they respond to by affecting
respiration?

Cerebellum: Mechanoreceptor input

Limbic system: emotional states

Cerebral cortex: voluntary control

Cerebral motor cortex: exercise

205. Hypoxia induces the secretion of which hormone from Kidneys:

Renin

Erythropoietin

Aldosterone

None of the above

206. Surfactant in the alveoli:


reduces the surface tension of the water film;

increases the permeability of the alveoli for gases;

creates an elastic craving of the lungs;

increases the surface tension of the water film

207. Which of the following does NOT happen during inspiration?

The ribs move upward

The diaphragm lifts up

The antero-posterior dimensions of the chest are increased

The tranverse dimensions of the thorax are increased

The scalene and sternocleidomastoid muscles can be recruited for inspiration

208. All the following factors lead to Hypoxia except:

Oxygen tension

Oxygen carrying capacity

Velocity of blood

Blood pressure

209. After ventilation of the alveoli with fresh air, what is the next step in respiratory process?

Diffusion of O2 from alveoli to pulmonary blood

Diffusion of CO2 from pulmonary blood to alveoli

Diffusion of O2 from pulmonary blood to alveoli

Both A and C

210. All of these statements about surfactant are true except:

promote stability of alveoli

reduce surface tension of alveoli

larger alveoli have lower surface tension than small alveoli according to Laplace law

assist in avoiding transudation of fluid into capillary

produced by Type II pneumocytes

211. Which of the following will NOT increase the minute ventilation?

An increase in arterial pH.

An increase in arterial partial pressure of carbon dioxide.


c ease a te a pa t a p essu e o ca bo d o de.

Increase in alveolar pressure of carbon dioxide.

Exercise.

Hypoxia.

212. Which Hypoxia is related to the Hemoglobin dysfunctioning?

Hypoxic Hypoxia

Anemic Hypoxia

Histotoxic Hypoxia

Ischemic Hypoxia

213. Which of the following instruments is used in the picture in studying the respiratory process?

vitalometer

pneumotachometer

spirometer

None of the above

214. Following are the causes of Anemic hypoxia, except:

Decreased number of RBCs

Decreased hemoglobin content

Formation of altered hemoglobin

Neutrophilia

215. There is direct relation in between the metabolic activity and the respiratory process. It means that when the metabolic rate
increases, the respiratory process also increases and when it decreases, the rate of respiration also decreases. Now from the
following, tell that single most important factor between this relation?

O2 level

CO2 level

N2 level

The rate of diffusion

216. Anatomical dead space performs the following functions:

maintains the partial pressure of oxygen;

warms, moisturizes, cleans the air, includes protective reflexes;

regulates tidal volume;


regulates the composition of alveolar air.

217. Which of the following is FALSE concerning diffusion in the lungs?

CO is used to measure diffusing capacity because its uptake is diffusion limited.

Actual diffusion time includes time required for an O2 molecule to diffuse from the alveolus, through the membrane, the
plasma, and into a RBC.

Reaction time is the time it takes the O2 molecule to react with Hb.

The diffusion time is greater than the reaction time of an O2 molecule.

218. Due to the reflex stimulation of cardiac and vaomotor centers, what happens?

Decrease in heart rate

Right to left Shunting of blood

Blood pressure is raised, initially

Heart becomes stronger

219. Which of the following is NOT an effector of respiration?

Heart

Diaphragm

Intercostals

Trapezius.

220. Which of the following statements is true about Trachea in a respiratory system?

It acts as passage of air to secondary bronchi

It acts as air passage way to tertiary and primary bronchi

It passes air to bronchus

It functions for the sound production

221. 50% effectiveness of Oxygen therapy:

Hypoxic Hypoxia

Stagnant Hypoxia

Anemic Hypoxia

Histotoxic Hypoxia

222. We need OXYGEN exactly for:


anaerobic respiration

aerobic respiration

to produce internal body H2O

All of the above

223. For the purpose of drainage, CO2 gas is transferred from tissues to into the capillaries and then back to the right heart(
Right atrium and Right ventricle). But in this process not all the CO2 gas is transferred. Can you tell, why there’s left some
amount of it?

To combine with the coming Oxygen gas

Human body lacks the very high pressures that it requires

The cellular walls do not afford to transfer through much CO2 gas

None of the above

224. Alveolar ventilation in a male with a respiratory rate of 10/min and tidal volume of 600 ml is

1750ml

3000ml

4500ml

6000ml

225. Hypoxia causes:

Alcohol like intoxication of CNS

Social withdrawal

Addiction problems

Heroine intoxicaton

226. Anatomically, What lies in front of the esophagus?

Ribs

Breast

Wind Pipe

Lungs

227. Cyanide poisoning causes:

Stagnant hypoxia

Histotoxic hypoxia

Anemic hypoxia
Anemic hypoxia

Hypoxic Hypoxia

228. When multiple bronchioles are constricted, which disease occurs?

Emphysema

Asthma

Cystic Bronchitis

Chronic Bronchitis

229. Complete absence of oxygen from the tissues is:

Anoxia

Stenosis

Coma

Shock

230. The flowing is true regarding respiratory center:

lies in spinal cord

lies in medulla oblongata

lies in hypothalamus

lies in cerebellum

231. In which situation would the response to hypoxia be limited?

In a patient with obstructed airways.

When hypoxia is accompanied by hypercapnea.

In the hypoxia induced by high altitude.

During hypoventilation.

232. Which of the following statements on human lung ventilation is correct?

Inhalation involves the active contraction of abdominal muscles

Exhalation involves the passive relaxation of abdominal muscles

Exhalation involves the active contraction of the diaphragm

Inhalation is brought about by an increase in intraalveolar pressure

Exhalation is usually passive except during times of intense exercise


233. Lung compliance

is normally 100 ml/cm water

falls if the lung remains unventilated for longer periods

rises if the pulmonary venous pressure is increased

falls as the lung ages

is the area under the pressure volume curve

234. What is located exactly just above the muscle that’s the divider of thorax and abdomen?

Heart

lungs

both a and b

Diaphragm

235. Chemoreceptor reflex due to Hypoxia, increases the Respiratory rate which leads to:

Washing out of oxygen

Washing out of Nitrogen

Washing out of Carbon dioxide

Washing out of Hydrogen

236. How many major functions of the respiration?

Five

Three

Four

All of the above

237. In the respiration, one deals with mixtures of gases, mainly of:

O2 and CO2

O2, N2 and CO2

CO2, O2 and NO

All of the above gases

238. Passive exhalation begins with:

expiratory muscle contraction;

relaxation of the inspiratory muscles;


relaxation of the inspiratory muscles;

air outlet from the lungs;

contraction of the inspiratory muscles.

239. In the human being, normal plasma colloid osmotic pressure is:

30mmHg

20mmHg

40mmHg

10mmHg

240. Partial pressure of O2 is controlled by: i) the rate of absorption of oxygen into the blood ii) the rate of entry of new oxygen
into the lungs iii) atmospheric pressure of O2 iv) atmospheric pressure of CO2

i, ii and iii

ii and iii

i and ii

i, ii, iii and iv

241. Stagnant hypoxia is also called:

Hyperkinetic hypoxia

Hypokinetic hypoxia

Both a and b

None of the above

242. Which one is not the type of Hypoxia?

Hypoxic hypoxia

Anemic hypoxia

Histotoxic hypoxia

Stagnant Hypoxia

None

243. Which of the following statements on lung volumes is correct?

E. R. V. is the total volume that can be expired

E. R. V. + I. R. V. = V. C.

T. V. + I. R. V. is the maximum volume that can be inspired


R. V. represents an emergency supply of air during periods of stress

R. V. represents anatomic dead space

244. Oxygen content describes which type of Hypoxia:

Stagnant Hypoxia

Anemic Hypoxia

Histotoxic Hypoxia

Hypoxic Hypoxia

245. Which of the following can cause stagnant hypoxia?

COPD

Shock or heart failure.

Cyanide poisoning.

Carbon monoxide poisoning.

246. For the purpose of drainage, how the CO2 gas is transferred from interior of the cell to the outside of the cell?

Osmosis

Diffusion

Active transport

Passive transport

247. All of the following shift the oxygen dissociation curve to the right except:

decreased pH

Increased temperature

Carbon Monoxide

Increased 2.3 DPG

248. If the blood moved slower than normal through the alveolar capillaries, which of the following would have an increased
uptake?

Carbon dioxide.

Carbon monoxide.

Oxygen

None of the above.


249. Which of the following could NOT be part of an acinus?

Alveolar sacs

Alveolar ducts

Terminal bronchioles

Respiratory bronchiole

250. Cilia and mucous are found in:

Pharynx

Trachea

Glottis

Epiglottis

251. The anatomic dead space:

varies with minute ventilation

is typically 150ml

will increase COPD

is alveolar minus the pathological dead space

all of the above

252. Which of the following is FALSE concerning the effect of effort on airflow and volume during inspiration and expiration?

During inspiration, greater effort always results in greater flow.

Peak expiratory flow occurs at the beginning of expiration.

At low and moderate lung volumes, the greater the effort above threshold, the greater the airflow in expiration.

Portions of the expiration curve are effort independent.

253. Oxygen cannot diffuse into the blood and leads to Hypoxia in which of the following conditions:

Emphysema

Mysthenia gravis

Asthma

Depression

254. If you increased the left atrial pressure from 5 mmHg to 15 mmHg, what effect would that have on pulmonary circulation?

If you increased the left atrial pressure from 5 mmHg to 15 mmHg what effect would that have on pulmonary circulation?
If you increased the left atrial pressure from 5 mmHg to 15 mmHg, what effect would that have on pulmonary circulation?

It would increase the speed at which blood moves through the pulmonary circulation.

No change.

Blood flow would almost or completely stop.

255. Where does the secondary bronchi supply air to?

alveoli

lobes of lungs

respiratory gas exchange area

lungs

256. High altitude leads to Hypoxia because:

It reduces the number of red blood cells

It increases the tissue resistance

Oxygen tension in inspired air is reduced

Psychological effect

257. Work of the lung in breathing

is increased with larger tidal volumes

is increased with higher flow rates

in inspiration need to overcome the elastic forces and viscous resistance

in expiration need to overcome airway and tissue resistance

all of the above

258. If you blocked the blood supply to an alveolus, which of the following would NOT occur as a result?

The ventilation perfusion ratio would be 0.

The PAO2 would be greater than normal.

The PACO2 would be 0.

All of the above are true.

259. Which of the following is FALSE concerning chemoreceptor input to the respiratory centers?

CSF is a poor buffer and a drop in PCO2 produces a large change in pH initiating a change in respiration.

The body can adjust to chronic hypercapnea by using an active HCO3- transport process in the choroid plexus.

The carotid and aortic bodies detect increases in PaCO2 and pH, and decreases in PaO2 .
75% of ventilatory response is regulated by chemoreceptors in the CSF and 25% by the carotid and aortic bodies.

Central chemoreceptors tend to respond slowly over time, while carotid bodies react quickly to immediate needs.

260. What is the average or normal value of V/P ratio?

1.5

1.0

0.8

0.5

261. Which of the following definitions is FALSE?

O2 content of blood is the actual amount of O2 in one deciliter of blood.

O2 saturation of blood is the ratio of O2 content to its O2 capacity.

The O2 uptake curve of blood is the functional relationship between O2 content and PO2.

The O2 content of blood depends completely on the amount of Hb in the blood.

262. Asthma is an example of:

hypoventilation

Histotoxic hypoxia

Hyper ventilation

Normal ventilation but abnormal circulatory system

263. Which of the following is INCORRECT concerning the efficiency of breathing and the oxygen consumption of the respiratory
muscles?

Efficiency is defined as the ratio of mechanical work done to move air to the amount of metabolic energy used by the
respiratory muscles.

The respiratory system uses less than 3% of the body's total oxygen consumption at rest.

Respiratory muscles are more efficient than large muscle groups.

Emphysema increases the oxygen requirement for respiratory muscles.

Hyperventilation can increase the oxygen consumption of respiratory muscles to 30%.

264. How many there are secondary bronchi in the right lung?

four

three

two
five

265. If the equal pressure point during expiration is in the lobar bronchi, which of the following is TRUE?

Expiratory flow would be effort dependent.

Expiratory flow would be effort independent.

The bronchi beyond the equal pressure point would compress.

This situation would only occur during medium and low lung volumes.

266. What is diffusion?

A planned movement of molecules through the respiratory membrane and adjacent fluids

the random motion of molecules in all directions through the respiratory membrane and adjacent fluids

motion of the blood parts from higher to lower concentrated area through the respiratory membrane

motion of any fluid from lower to higher concentrated area through the respiratory membrane

267. The small air sacs in the lungs that diffuse O2 into the blood stream and take in CO2 from the blood stream:

basic part of respiratory system

bronchial tree

Lungs

Respiratory membrane

268. Which of the following is NOT a form by which CO2 can be transported in the blood?

As bicarbonate

Dissolved in the blood.

Bound to the amino end groups in proteins.

Bound to the imidazole ring of glutamate.

269. Which of these does not affect diffusion rate?

Surface area for diffusion

Concentration gradient

Solubility of gas

molecular weight of gas

none of the above

270. Elect the true statement regarding the control of gastric secretion:
Gastric acid is secreted by parietal cells of the gastric glands in response to hormonal stimulation.

Most of the secretion of acid and pepsinogen occurs during the intestinal phase.

Gastric secretion does not begin until food enters the stomach.

Secretin secreted by the duodenum stimulates gastric secretion

271. The following reflexes inhibit small intestine motility except:

Peritoneo-intestinal

Reno-intestinal

Gastro-ileal.

Somato-intestinal.

272. The transport of glucose across intestinal mucosa is directly dependent Upon the following except:

Na concentration in intestinal lumen.

Plasma insulin.

Inhibitors of energy metabolism

273. About 90% of kidney stones can pass out through urinary system by drinking _______

Water

Juice

Sugar drinks

Milk

274. The following are true statement about absorption from small intestine except:

Hexose are absorped faster than pentose.

D amino acids are absorped better than L amino acids.

Glucose is absorped faster than fructose.

275. Concerning HCl secretion, all the following is true except:

It’s associated with increased PH of the gastric venous blood

It’s stimulated by gastrin, acetylcholine,histamine and norepinephiren

The energy required is derived from ATP breakdown

It involves formation of carbonic acid

There is an active transport of H⁺ from oxyntic cells into the gastric lumen and K⁺ in the opposite direction
276. Yellow color of urine is due to ____________

Urochrome

Urea

Bilirubin

Uric acid

277. The so-called chemoreceptor trigger zone:

Is synonymous with the vomiting center

Is located in the cerebral peduncles, ventral to the Aqueduct of Sylvius

Is sensitive to the action of morphine

May be destroyed experimentally ; the ablation of both chemoreceptor trigger zones abolishes all forms of vomit

278. Absorption of fat soluble vitamins require the following:

Bile.

Bile and chymotrypsine.

Bile and pancreatic amylase.

None of the above.

279. In contrast to Secondary esophageal peristalsis,primary esophageal peristalsis characterized by which of the following
statements?

It does not involve relaxation of the lower esophageal sphincter.

It involves only contraction of esophageal smooth muscle.

It is not influenced by intrinsic nervous system.

It has an oropharyngeal phase.

It involves only contraction of esophageal skeletal muscle

280. 88% filtrate is reabsorbed in:

Distal convulated tubule

Proximal convulated tubule

Loop of Henle

Collecting duct

281. Inflammation of joint due to accumulation of uric acid crystals is called ____________

Gout
Osteoporosis

Osteomalacia

Myasthenia gravis

282. Glomerular capillary pressure varies between:

20 to 30 mm Hg

45 to 70 mm Hg

80 to 100 mm Hg

10 to 15 mm Hg

283. Resting anal pressure is mainly due to :

Tone of external anal sphincter

Tone of the internal sphincter.

Tone of the puborectalis muscle.

284. Which of the following sphincters does NOT prevent reflux of material?

Lower esophageal sphincter

Gastroduodenal sphincter.

Ileocolonic sphincter

Internal anal sphincter.

285. Which of the following statement is wrong?:

The gastric peristalsis includes propulsive,mixing and grinding movements

The cephalic phase of gastric secretion involves no release of gastrin.

Deficiency of the intrinsic factor causes pernicious anaemia.

Large doses of gastrin cause contraction of the pyloric sphincter.

286. Renal columns are also called:

Columns of Bellini

Columns of Bertini

Malpighian columns

Intermedullary rays
287. Glomerular capillaries are lined by epithelial cells attached to basement membrane and has many pores for the process of:

Phagocytosis

Connection

Filtration

Protection

288. Select a single incorrect answer about Swallowing:

Reflex apnea occurs during the pharyngeal phase.

The buccal phase of swallowing is voluntary.

Food is prevented from entering the nose during swallowing by elevation of epiglottis

289. Which of the following is NOT involved in swallowing?

Contracture of the upper esophageal sphincter.

Coordination by the swallowing center in the medulla oblongata.

The approximation of the vocal cords to close the glottis.

The raising of the larynx to close its entrance.

The elevation of the soft palate to close the nasopharynx.

290. Following substances are completely reabsorbed from the renal tubules, except:

Glucose

Vitamens

Amino acids

Minerals

291. Which best describes the motility in the large intestine?

Food clears the LI in about the same time as it takes to clear the small intestine.

Of the parts of the large intestine, chyme spends the least time in the ascending colon.

The ascending colon consists of haustra that aid in storage and dehydration of feces.

Remnants of meals stay in the LI for approximately 6-8 days.

Receptive relaxation allows the transverse colon to accept chyme without significantly increasing the pressure.

292. Renal papilla is:


V shaped area

A Circular area

Present in cortex

Part of ureter

293. Each arcuate artery gives rise to:

Interlobular arteries

Lobar arteries

Segmental arteries

Glomerular arteries

294. Peristaltic contractions of small intestine are characterized by all of the followings except:

They are myogenic

They are due to local axon reflex

They are stimulated by gastro-enteric reflex

295. The followings are true statements about absorption from small intestine except:

Fructose is absorbed by facilitated diffusion.

Glucose is absorbed by Na+ dependent 2ry active transport.

Vit. B12 is absorbed by diffusion

296. If you follow the path that food normally takes through the digestive system, which of the following is in the correct order?

Upper esophageal sphincter, epiglottis, lower esophageal sphincter, pyloric canal, duct of Santorini, duct of Wirsung.

Pharynx, epiglottis, lower esophageal sphincter, pyloric canal, duct of Wirsung, duct of Santorini.

Pharynx, epiglottis, lower esophageal sphincter, pyloric canal, duct of Santorini, duct of Wirsung.

Pharynx, upper esophageal sphincter, pyloric canal, lower esophageal sphincter, duct of Wirsung, duct of Santorini.

297. During which stage of Swallowing is respiration inhibited?

Voluntary stage.

Pharyngeal stage.

Esophageal stage.

Postprandial stage

298. All the following statements are true except:


VIP stimulates intestinal secretion and inhibits gastric acid secretions

Diamox (a carbonic anhydrase inhibitor) increases both gastric HCL formation and the HCO₃⁻ content in the pancreatic juice

Both gastrin and glucagons are secreted from the pyloric antrum and duodenum

Gastrin is secreted from almost the whole GIT mucosa

Glicentin is formed by the intestinal mucosa and has some glucagones activity

299. Mass movement in the colon would be abolished by:

Vagotomy.

Sympathectomy.

Destruction of the intrinsic nerve plexus.

300. Low pressure capillary bed is formed by

Renal glomerular capillaries

Peritubular capillaries

Afferent arterioles

Efferent arterioles

301. The main mechamism of succus entericus secretion is:

Vagal stimulation.

CCK.

Secretin

Local axon reflex.

302. Water, electrolytes and other substances are taken back into the blood from glomerular filterate, in a process called:

Tubular reabsorption

Tubular secretion

Glomerular extraction

Tubular transport

303. What is the function of ADH?

Controls sugar level in blood

Increases water absorption

Decreases water absorption

S h i f l
Synthesis of salt

304. Acidification of the duodenum will:

Decrease pancreatic secretion of bicarbonate.

Increase secretion of gastric acid.

Decrease gastric emptying.

Increase contraction of the gall bladder.

Increase contraction of the sphincter of Oddi.

305. Urea synthesis occurs in ______________

Kidney

Liver

Pancreas

Muscles

306. Macula densa secretes:

Thromboxane A2

Lipase

Pepsin

Erythropoitin

307. Filtration fraction is:

Portion of plasma that becomes filtrate

Portion of filtrate that becomes urine

Portion of urine that is reabsorbed

Portion is plasma, secreted into nephron

308. Fructose is absorped from small intestine by :

Simple diffusion.

Facilitated diffusion.

Na+ dependent 2ry active transport.

309. Renal corpusels are formed of:

Glomerulus
Bowman capsule

None of the above

Both of the above

310. Macula densa is the part of:

Renal hilum

Renal pelvis

Juxtaglomerular apparatus

Ureter

311. The GIT hormone are characterized by all the following except:

They are secreted by APUD system and are divided into 2 families on The basis of their structural similarity.

They are secreted in response to specific physiological Stimuli during digestion.

Their effects are abolished by cutting the nervous connections of GIT.

They affect areas in GIT that may be far away from the sites of their release.

I.V injection of their extracts produce similar effect to those produced by the stimuli that release them.

312. Ingesting antacids with and after a meal so that PH doesn`t decrease below 6 will cause a greater than normal secretion of:

Gastrin.

Secretin.

Pancreatic bicarbonate.

CCK.

Somatostatin

313. All of the following statements concerning pancreatic secretion are true except:

Its pH is about 8

Has high HCO₃⁻ content

It’s secretion is primarily under neural control

Contains digestive enzymes

314. Angiotensin II produced under the effect of Renin, produces which of the following effects?

Increases blood pressure directly by noradrenaline

Increases blood pressure indirectly vasoconstriction

Increases blood pressure directly by vasoconstriction


Increases blood pressure directly by vasoconstriction

Increases blood pressure by vasodilation

315. Hemodialysis is done when the person is suffering from ________

Goiter

Diabetes insipidus

Diabetes mellitus

Uremia

316. The rate at which maximum amount of substance is reabsorbed from the renal tubule:

Transport Maximum

Transport Minimum

Both of the above

None of the above

317. Intestinal proteolysis is caused by the followings except:

Trypsin.

Carboxypeptidase.

Chymotrypsin.

Pepsin

318. Which of the following does NOT contribute to hydrochloric acid secretion in the stomach?

A basolateral bicarbonate/chloride exchanger to maintain intracellular chloride ions also resulting in an “alkaline tide” in the
bloodstream.

Passive flow of chloride and potassium ions through apical channels.

Hydrogen ions supplied by a basolateral hydrogen/sodium pump.

Water passively follows acid into gut to maintain osmotic balance.

The primary contributor to hydrochloric acid secretion is a hydrogen/potassium ATPase that pumps out hydrogen in exchange
for extracellular potassium

319. Normal urinary out put is:

1 to 1.5 liters/day

5 to 10 liters/day

70 liters/day

20 liters/day
320. Tubuloglomerular feedback is the mechanism that regulates GFR through:

Renal tubule

Macula densa

Both

None

321. Under the normal conditions, the bulk of gastric secretions are produces:

Before the food is ingested and while the pleasure of it’s ingestion is anticipated

While the food stays in the stomach.

After the food has entered the duodenum.

During fasting periods.

322. Which of the following is not a part of renal pyramid?

Collecting ducts

Peritubular capillaries

Convoluted tubules

Loops of Henle

323. What happens during swallowing action of the body?

all processes of dilution are under voluntary control.

the vocal folds are relaxed allowing the food to travel down the oesophagus.

the swallowing center are located within the medulla oblongata inhibits medullary respiratory center.

the bolus travels down due to gravity.

324. The chyme entering the small intetstine causes a release of secretin which results in:

Stimulation of pancreatic fluids which there are no enzymes.

A pancreatic fluid that aids in protection against the development of duodenal ulcers.

A pancreatic secretion whose pH is just right for action of the pancreatic enzymes that are eventually released.

Pancreatic fluid secretion of a large volume containing low chloride but high bicarbonate concentration.

All are correct

325. Which of the following has little effect on pancreatic secretion or it's components?

Acetylcholine
Acetylcholine.

Sympathetic stimulation.

Secretin

Cholecystokinin

VIP

326. All the pressures involved in determination of filtration are collectively called:

Starling forces

Filtration forces

Diffusion forces

Colloidal forced

327. Distension of the stomach:

Is associated with a decrease in it’s peristaltic movement.

Leads to reduction of gastric juice secretion.

Decrease the tone of the lower esophageal sphincter.

Causes an acute increase in the intragastric pressure.

Results in an increase in the volume of the stomach with a little rise in the intragastric pressure

328. Both formation and concentration of urine is mainly the function of:

Cortical nephrons

Juxtamedullary nephrons

Both types

None of the above

329. Excretion is the process by which unwanted substances and metabolic wastes are:

Produced in the body

Utilized in the body

Eliminated from the body

Accumulated in the body

330. Choose the best description of the duodenum.

Mucosa with crypts and villi; epithelium with scattered goblet cells; lamina propria with occasional lymph nodules; submucosa
unspecialized.
Mucosa with crypts but no villi; epithelium with scattered goblet cells; lamina propria with occasional lymph nodules;
submucosa unspecialized.

Mucosa with both crypts and villi; epithelium with scattered goblet cells; lamina propria with occasional lymph nodules;
submucosa with mucous glands.

Mucosa with both crypts and villi; epithelium with scattered goblet cells; lamina

331. Digestion of proteins by succus entericus enzymes includes all the following enzymes except:

Aminopeptidase.

Carboxypeptidase.

Nucleases.

Dipeptidase

332. Pancreatic secretion is inhibited by all the followings except:

Somatostatin

Glucagon

Acid in the duodenum

Sympathetic stimulation.

333. Glomerular hydrostatic pressure is present in ________

Tubule of kidney

Bowman’s capsule

Malpighian tubule

Glomerulus

334. Which of the following statement is true?

The intrinsic factors is secreted by the parietal cells of the gastric mucosa.

The gastric juice can’t digest the gastric mucosa because the cell membranes contain a pepsin activator.

The stomach normally absorbs about 30% of the ingested food.

The secretion of the gastric juice is associated with increased H⁺ in the blood.

The gastric juice is essential for the digestion of both fat and protein.

335. Haematuria means _________

RBCs in urine

WBCs in urine

Fats in urine
Urea in urine

336. Rectal mucosa is sensitive to:

Pain

Touch

Temperature

Stretch

337. Polystyrene sulfonate is used in renal failure to ___________

Prevent constipation

Reduce serum level

Correct acidosis

Exchange potassium ions

338. Filtration coefficient:

50 ml/mm Hg

125 ml/mm Hg

6.25 ml/mm Hg

20 ml/mmHg

339. PCT is lined with ______________

Cuboidal epithelium

Columnar epithelium

Simple brush border epithelium

Simple Cuboidal epithelium

340. Which of the following statements is correct?:

The gastric juice contains pepsins,gelatinase,lipase and α-amylase enzymes.

The only essential constituent of the gastric juice is the intrinsic factor.

The hunger contractions disappear after bilateral vagotomy or damage of the feeding center in the hypothalamus.

HCL is essential for both carbohydrate and fat digestion

341. Which of the following is the major factor that protects the duodenal mucosa from the damage by gastric acid?

P ti bi b t ti
Pancreatic bicarbonate secretions.

The endogenous mucosal barrier of the duodenum.

Duodenal bicarbonate secretion,

Hepatic bicarbonate secretion.

Bicarbonate contained in bile.

342. Following are the hormones that regulate tubular reabsorption, except:

Aldosterone

Parathormone

Calcitonin

Testosterone

343. Factor decreasing the sensitivity of tubuloglomerular feedback:

Adenosine

Thromboxane

Hydroxyeicosatetranoic acid

Cyclic AMP

344. Which of the following statements about gastric secretion is incorrect:

Gastric acid secretion can be inhibited by somatostatin

The main phase of gastric secretion is the gastric phase

Gastric secretion increases in response to the presence of food in mouth after the vagi to stomach have been cut

345. How is the MMC controlled?

Ingestion of food ends MMC at all points in the intestine.

IV feeding ends MMC.

Gastrin and CCK terminate MMC in stomach and entire small intestine.

Vagal innervation completely controls MMC.

The MMC increases in speed as it reaches the ileum.

346. Mark the mismatch pair.

Henle’s loop – concentration of urine

DCT – Absorption of glucose

PCT – Absorption of sodium and potassium ions


Bowman’s capsule – Glomerular filtration

347. The following are choleretics except:

Bile salts

Secretin.

Acetylcholine.

Bile pigments

348. About the gastric motility, all the following is true except:

It’s marked in the distal half of the stomach

It's inhibited in painful conditions

it increases by vagal stimulation and decreases by symp. Stimulation

It decreases by the enterogastric reflex and by the secretin and CCK hormones

It increases by over distension of the stomach

349. Deglutition (Swallowing):

Is a complicated act requiring the precise coordination of many muscle group.

Is an automatic function of smooth muscle.

Is associated with a lowering of hard palate to prevent reflux of food into the nasopharynx.

Does not require relaxation of cricopharyngeal muscle.

A and c are correct.

350. Name the condition when concentration of ketone increases in urine.

Turners syndrome

Diabetes mellitus

Sickle cell anemia

Acromegaly

351. Urea synthesis takes place in liver because __________

Enzyme arginase is present in liver only

Kidney is smaller than liver

ADH is found in liver

Urea is present only in liver


352. Vasopressin is called as ___________

Synovial fluid

Neurotransmitter

Antidiuretic hormone

Growth regulating substance

353. The secretion of gastrin by the pyloric antrum ceases:

When the stomach is distended by a full meal

When the gastric mucosa contents have a pH of about 2.0

If histamine is injected.

If the vagi are stimulated

354. In the distal convulated tubules, the sodium reabsorption is stimulated by hormone:

Angiotensin

Erythropoiten

Aldosterone

Parathormone

355. Whole proteins may be absorped from small intestine by :

Pinocytsis.

Na+dependant 2ry active transport.

Facilitated diffusion.

356. Mass movement in the colon after meal is due to the following except:

Gastrocolic reflex

Duodenocolic reflex

Gastrin hormone.

Secretine hormone.

357. Which one is an important constituent of rennin angiotensinogen aldosterone system?

Macular cells

Plasma cells

JGA cells
Erythropoietin

358. Glucose is mainly reabsorbed in _______

Henle’s loop

DCT

PCT

Nephron

359. Regarding pancreatic secretion:

Pancreatic secretion is inhibited by gastrin secreted by the G cells of the antrum

Pancreatic acinar cells contain trypsin

Cholecystokinin inhibits secretion from the exocrine pancreas.

The introduction of acid into the duodenum stimulates pancreatic secretion.

360. Choose the correct listing of the anatomical divisions of the stomach in the direction food normally passes.

Corpus, fundus, antrum.

Fundus, antrum, corpus.

Antrum, corpus, fundus.

Fundus, corpus, antrum.

Corpus, antrum, fundus.

361. Which of the following does NOT regulate gastric acid secretion?

Vagus nerve stimulation.

Cholecystokinin.

Histamine

Gastrin

Acetylcholine

362. Hairpin bend of the loop of Henle has all the following features, except:

Formed by flattened epithelial cells

Has no brush border

Continued as ascending limb

Continued as descending limb


363. Capsular pressure is also called:

Colloidal osmotic pressure

Filtration pressure

Hydrostatic pressure in Bowman's capsule

Glomerular capillary pressure

364. Glomerular filteration is which of the following process?

Microfiltration

Macrofiltration

Ultrafiltration

Autofiltration

365. Following are the substances passively reabsorbed from renal tubule, except:

Chloride

Urea

Ascorbic acid

Water

366. When blood passes through glomerular capillaries, plasma is filtered into bowman capsule. All substances are filtered
except:

Carbohydrates

Lipids

Proteins

Minerals

367. Pentose is absorped from small intestine by:

Simple diffusion.

Facilitated diffusion.

Na+ dependant 2ry active transport.

368. Podocytes are found in ______________

Cortex of nephron

Outer wall of Bowman’s capsule

Inner wall of Bowman’s capsule


Wall of glomerular capillaries

369. The following statements about gastric secretion are correct except:

Gastric secretion increases when a hungry person thinks about food

Gastric secretion helps vit.B₁₂ absorption

Production of HCl depends on activity of carbonic anhydrase

Gastric secretion is associated with increased H⁺ concentrarion in venous blood coming from stomach

370. Which of the following statements correctly describes a function of the parasympathetic innervation of the GI tract.

Norepinephrine is the primary excitatory neurotransmitter.

Excessive parasympathetic activity can lead to a pathological state called paralytic ileas.

Inhibitory efferent Vagal innervation causes sphincters to contract.

Afferent sensory input is received in the nucleus of the solitary tract and processed in the dorsal motor nucleus of the medulla
oblongata.

Glucose concentration, pH, osmolality, movement of material past mechanoreceptors, and level of contractile strength or
stretch are monitored by efferent parasympathetic fibers propria packed with mucous glands; submucosa with numerous
lymph nodules

371. Stimulation of Gastrointestinal Secretion include:

Chemical stimuli.

Tactile stimulation.

A and B.

Distension.

All are correct.

372. Many factors influence the gastric action potential of the stomach. Which of the following is false?

Acetylcholine, gastrin, and cholecystokinin increase amplitude of the plateau phase.

Gastrin increases AP frequency.

The initial contraction coincides with the plateau phase while the trailing contraction coincides with the rising phase of the AP.

Epinephrine and VIP decrease the amplitude of the plateau phase.

It is a single action potential originating in the orad boundary of the distal stomach.

373. Glomerular filtrate collected at Bowman's capsule contains all the substances of plasma except protiens which makes it:

Hypotonic to plasma

Hypertonic to plasma
ype to c to p as a

Isotonic to plasma

None of the above

374. Glomerular filtrate contains _________

Plasma without sugars

Blood without blood cells and proteins

Blood with proteins but without cells

Blood without urea

375. 85% of all nephrons are:

Cortical nephrons

Juxtamedullary nephrons

Collecting nephrons

Mixed nephron

376. Which of the following statement about motor function of stomach is untrue?:

The frequency of gastric peristaltic contraction is about 3/min

Fasting hypoglycemia produces hunger contractions

The enterogastric reflex stimulates gastric emptying

377. In adults, during resting conditions both the kidneys receive 1,300 ml/min blood which is:

50% of cardiac output

10% of cardiac output

26% of cardiac output

70% of cardiac output

378. The condition where urea accumulates in blood is

Anemia

Glycosuria

Uremia

Acidosis

379. Tubular secretion is the process by which the substances are transported:

F bl di l b l
From blood into renal tubules

From renal tubules into blood

From Renal tubules into urine

From urine into blood

380. Theoretically, which of the following deficiencies would make pepsin's role of protein digestion in the stomach more
crucial?

Lipase deficiency.

A defect in the amino acid intestinal transport mechanism.

Enterokinase deficiency.

Cholecystokinin deficiency

381. Among these processes, which one is the function of glomerulus?

Glomerular secretion

Glomerular reabsorption

Glomerular filtration

Glomerular circulation

382. Emptying at ileocecal region is promoted by:

Distenion of cecum

Distention of colon.

Distention of stomach.

Sympathetic reflex.

383. Emptying gastric contents into the small intestine is strictly controlled. Which of the following is most acurately represents
this concept?

High caloric meals empty faster.

Liquid and solid meals empty at the same rate.

Higher acidity in the stomach results in a slower rate of emptying.

Tonicity of the gastric solution doesn't affect the rate of emptying.

384. Secretin:

Is a GIT hormone Secreted from the pylorus.

Stimulates a pancreatic Secretion rich in enzymes.

Acts as powerful cholagogue.


Is secreted as aresult of vagus nerve stimulation.

Stimulates gastric Secretion

385. Each kidney consists of:

1 to 1.3 million nephrons

3 to 5 million nephrons

10 billion nephrons

0.5 million nephrons

386. The absorptive surface area of small intestineis over:

2.8m

250m2

7m

50m2

387. It’s believed that in parietal cells of the gastric mucosa (which secrete hydrochloric acid), an active transport system is
responsible for the transmembrane passage of:

Chloride

Carbon dioxide.

Bicarbonate.

Water

388. Of the following water soluble vitamins, which one CANNOT be absorbed by passive diffusion if the concentration is high
enough?

Vitamin B1

Niacin.

Vitamin B12

Biotin.

389. Method for collection of tubular fluid:

Micropuncture technique

Stop-flow method

Both

None
390. Reabsorption of the substances occur at the following sites:

Proximal convulated tubule

Distal convulated tubule

Loop of henle

All

391. Transport maximum for glucose (TmG) is:

375 mg/min in adult male

300 mg/min in adult female

500 mg/min in adult male and female

Both a and b

392. Emptying at ileocecal region is inhibited by:

Gastroileal reflex.

Gastrin

Colonoileal reflex.

Distention of terminal ilium.

393. Macula densa of the Juxtaglomerular apparatus is sensitive to:

Sodium chloride in the tubular fluid

Sodium Bicarbonate in tubular fluid

Protiens in tubular fluid

Pressure in glomerulus

394. The following are true about defecation except:

It is initiated by distention of rectum with feces.

It is facilitated by micturition reflex.

It is under voluntary control in infant.

It is inhibited by pain.

395. Each kidney has got about 8:

Minor calyx

Major calyx
Pyramids

Arcuate artries

396. Juxtaglomerular hormone secretes which hormone:

Progesterone

Prostaglandin

Testosterone

None of the above

397. Which of the following is NOT a component of saliva?

Alpha-amylase and ligual lipase.

Mucin and muramidase.

Bicarbonate and magnesium.

Protease and peptidase.

398. Intestinal motility help absorption from small intestine by all the following except:

Increase blood and O2 supply.

Increase lymph flow.

Increase osmotic pressure.

Increase contact of food with mucosa

399. Volume of urine is regulated by ___________

Volume of urine is regulated by ___________

Aldosterone

ADH and Aldosterone

Aldosterone and testosterone

400. Large intestinal secretion is stimulated by:

Direct stimulation.

Local myentric reflex.

Para sympathetic stimulation.

All of the above.


401. Of the approximately 9 liters of water that go through the GI tract daily, how much makes it out in the feces?

100 ml

1000 ml

2000 ml

1500 ml

500 ml

402. Following are functions of the kidneys, except:

Uric acid production

Maintenance of water balance

Maintaince of acid base balance

Erythropietic production

403. What is glycosuria?

Low amount of sugar in urine

Average amount of sugar in urine

Low amount of fat in urine

High amount of sugar in urine

404. Which of the following statements is wrong?

Repeated vomiting leads to dehydration,alkalosis, and K⁺ depletion

The rate of gastric emptying is affected by the fat content of food

The enterogenic reflex is a gastro-inhibitory reflex

The enterogenic reflex depends on vagal impulses as well as local enteric and prevertebral ganglionic reflexes

The enterogastric reflex is of value in facilitating carbohydrates digestion

405. Renal glomerular capillaries forms high pressure bed of:

60 to 70 mm of Hg

8 to 10 mm of Hg

100 mm of Hg

5 mm of Hg
406. About the process of Swallowing, all the followings are true except:

As a peristaltic wave passes along the esophagus, the cardiac sphincter, stomach wall and duodenum relax.

The esophageal musculature below the pharynx contains striated (skeletal)muscles which is entirely under control of vagi
nerves.

The most sensitive site for initiation of the pharyngeal phase is the tonsillar pillars.

Food does not enter the trachea mainly due to approximation of the vocal cords.

The voluntary phase includes the period during which food passes through both the buccal cavity and pharynx.

407. As regards the process of vomiting:

Its is controlled by a center in the cerebral cortex

It always begins with nausea

Prolonged vomiting usually produces dehydration associated with acidosis

It’s associated with relaxation of the body and fundus of the stomach

Uraemic toxins stimulates the vomiting center directly

408. Which of the following is not a nitrogenous waste?

Citrulin

Allantoin

Purines

Creatinine

409. Normal GFR is about:

125 ml/min

180 L/day

none of the above

both a and b

410. The type of gradual increase in the osmolarity of the medullary intestitial fluid is called:

Tubular gradient

Renal gradient

Medullary gradient

Urinary gradient

411. Juxtaglomerular apparatus is not formed by:


Macula densa

Extraglomerular mesangial cells

Juxtaglomerular cells

Podocytes

412. Epithelial cells with pedicles on the basement membrane are called:

Podocytes

Astrocytes

Lymphocytes

Phagocytes

413. The GIT is controlled by: Its own intrinsic nervous system(Auerbach,s and Meissner,s plexuses).

The sympathetic N.S.

The parasympathetic N.S.

only B&C.

A,B&C.

414. Juxta glomerular cells of renal cortex synthesize a hormone called __________

ADH

Renin

Oxytocin

Urochrome

415. Principal and intercalated cells form:

Ascending limb

Proximal convoluted tubules

Collecting duct

Glomerulus

416. Gastric secretion is inhibited by all the following except:

Presence of excess H⁺ in the pyloric antrum

Presence of excess protein digestive products in the stomach

Certain emotions e.g. fear and depression


Certain GIT hormones e.g. secretin,CCK and VIP

Presence of fat, hypertonic solutions and excess acid in the duodenum

417. Pressure exerted by plasma protiens:

Colloidal osmotic pressure

Hydrostatic pressure

Net filtertion pressure

Glomerular capillary pressure

418. The parasympathetic supply of the distal colon is:

Pelvic nerve.

Vagus nerve

The lesser splanchnic nerve.

The pudendal nerve.

419. Liquid which collects in Bowman’s capsule is ___________

Water and sulphates

Water and glycogen

Plasma minus blood proteins

Concentrated urine

420. Which of the following is the urinary bladder of embryo?

Allantois

Amnion

Chorion

Yolk sac

421. In micturition _____

Urethra relaxes

Ureters relax

Ureters contract

Urethra contracts

422. the Secretion of gastrin cease(stop):


When the stomach is distended by meal.

When the PH of the gastric content is Decreased below 2.

If the fundic mucosa is anaesthesized.

If the vagi are Stimulate.

If histamine is injected

423. Anal mucosa is sensitive to all of the following except:

Pain

Stretch.

Touch

Temperature

424. Main function of Henle’s loop is ___________

Passage of urine

Filtration of blood

Formation of urine

Conservation of urine

425. Which of the following is NOT a component of bile secretions?

Stercobilin.

Bile pigments

Cholesterol

Lecithin

Bilirubin-glucuronide

426. Part of nephron impermeable to salt is ____________

DCT

Collecting ducts

Ascending limb of loop of Henle

Descending limb of loop of Henle

427. The gastric mucosal barrier is maintained by all the following except:

The nature of the surface membrane of the gastric mucosal cells and the tight junctions between these cells
Certain prostaglandins and the alkaline mucous secretion of the stomach

The mucus gel-like membrane that coats the gastric mucous membrane

Administration of H₂ receptor blocking drugs (e.g. cimetidine)

Excessive amounts of bile salts,asprin or ethanol

428. The following statement about calcium absorption from small intestine are true except:

It occurs by active transport.

It requires vit. D and parathormone.

It occours by passive diffusion from upper small intestine.

It is decreased by excess phosphate and oxalate in food.

429. An example of non threshold substance:

Urea

Uric acid

Creatinine

Phosphate

430. Pepsin, secreted by the gastric mucosa:

Continues to act in the neutral medium of the duodenum.

Is proteolytic on the mucosal surface, in the alkaline medium provided by the mucus secreted by the surface cells of the
stomach.

Is synthesized within the chief cells.

Does not increased in concentration in response to the mental stimuli found in the so’called cephalic phase of gastric
digestion.

431. Which of the following is characteristic of the segmenting movements in the small intestine?

It decreases particle size, which increases the surface area for digestion.

It brings the products of digestion to the mucosal surface for absorption.

It results in the mixing of the luminal contents in front of the propulsive segments (i.e. in the receiving segments)

All of the above

432. Vasa recta supplies to tubules in:

Cortical nephrons

Juxtamedullary nephrons

Mixed nephrons
Mixed nephrons

None of the above

433. Colonic motility is stimulated by the following except:

Gastro-colic reflex.

Colono-colic reflex.

Duodeno-colic reflex.

434. Gall bladder bile is characterized by all the followings except:

It is relatively acidic

Darker in color.

More inorganic constituent.

Higher specific gravity.

435. Fecal continence requires:

Intact innervations of anorectal region

Sensation of rectum and anal mucosa.

Tone of the internal and external anal sphincter

All of the above

436. The following are true statement about fat absorption from small intestine except:

It is helped by bile salts.

Glycerol and short chain FAs are absorbed passively.

Deficiency of fat absorption leads to poor absorption of VIT.B complex.

Chylomicrons are extruded into Lacteal laterals by exocytosis

437. Substance which is not reabsorbed from loop of Henle:

Sodium

Phosphate

Chloride

None of the above

438. All are the structural components of kidney, except:

Inner meulla
Trigone

Renal sinus

Nephron

439. Which pair is correct concerning secretory glands/cells in the stomach and their products?

Cardiac glands -- hydrochloric acid and intrinsic factor.

Parietal cells -- pepsinogen

Chief cells -- mucus

G cells -- protease

D cells -- somatostatin

440. GFR and permeability of Glomerular capillary membrane has one of the following relationship:

Directly propotional

Indirectly proportional

Inversely proportional

Negative feedback

441. ADH makes the urine hypertonic with osmolarity of:

1,200 mOsm/L

2000 mOsm/L

5000 mOsm/L

500 mOsm/L

442. In the presence of which hormone, distal convulated tubule and collecting duct of renal tubule become permeable to water
so that it is reabsorbed?

Aldosterone

Parathormone

Growth hormone

Anti diuretic hormone

443. Factor increasing the sensitivity of tubuloglomerular feedback:

ANP

Prostaglandin I

Nitrous oxide
Thromboxane

444. The secretion of the intrinsic factor occurs in:

Parietal cells of the stomach.

Chief cells of the stomach.

Upper duodenum.

Beta cells of pancreas

Liver.

445. Large intestine secretion is stimulated by:

Direct stimulation.

Local .myentric reflex.

Parasympathetic stimulation.

All the above.

446. Man is unable to digest dietary:

Glycogen.

Dextrin.

Saccharose.

Cellulose.

Glucose

447. Which of the following statements about fat and fat digestion/absorption is true?

Fat soluble vitamins are incorporated into mixed micelles and absorbed directly across the microvillus cell membrane

Triglycerides are resynthesized from fa y acids and monoglyceride in the intestinal epithelial cell and are exported directly into
the portal circulation

Pancreatic lipase functions best at a pH of 3

None of the above

None of the above

448. The external anal sphincter is supplied by:

Pelvic nerve.

Pudendal nerve.

Vagus nerve
449. The enterogastric reflex can be elicted by which of the following?

Distension of the duodenum

Acid chime in the duodenum.

Hyperosmotic chime in the duodenum.

All of the above

450. GFR is directly proportional to:

Tempreature of body

Renal blood flow

Pulse rate

Respiratory rate

451. Substances which do not appear in urine under normal conditions are:

High-threshold substances

Low-threshold substanced

Non threshold substances

Sub threshold substances

452. Removal of the stomach can lead to all of the following except:

Marked digestive disturbances

Megaloblastic anaemia (pernicious anaemia)

A fall in the plasma volume after a heavy meal (due to the dumping syndrome)

Diminished Ca⁺⁺ absorption and weak development of bones

453. Muscles in the propulsive and receiving segments of the GI tract respond differently to food movement through the gut.
Which of the following statements correctly describes activity in the propulsive segment?

The circular and longitudinal muscles are contracted.

The longitudinal muscles are contracted and the circular muscles are relaxed.

Both the longitudinal muscles and circular muscles are relaxed.

The circular muscles are contracted and the longitudinal muscles are relaxed.

454. Which structure acts as the countercurrent exchanger?

Vasa recta
Macula densa

Glomerulus

Bowman's capsule

455. Which is not the type of Aphasia?

Broca Aphasia

Dysphoniac Aphasia

Global aphasia

Wernicke aphasia

456. Retching or gagging (opening up of mouth) points to:

Corneal reflex

Pharyngeal reflex

Cremesteric reflex

Gluteal reflex

457. During Sleep:

Heart rate varies between 45 and 60 beats per minute

Heart rate varies between 90 and 120 beats per minute

Heart rate increases

Heart rate is not changed much

458. Buerger waves ( alpha waves) of EEG have the rhythm per second of:

0-4

4-7

8-13

13-30

459. Nightmares occur due to all, except

Improper food intake

Drug or alcohol withdrawal

Digestive or Nervous disorders

Sleeplessness
460. Which involves the conscious recollection of past experience?

Long term memory

Short term

Explicit memory

Associative Memory

461. Damage to sleep centers in brain leads to:

Hypersomnia

Sleep phobia

Insomnia

Nightmares

462. Ability to recall past experience or information is called:

Cognition

Memory

Coordination

Facilitation

463. The two centers Raphe nucleus and Locus ceruleus of pons which induce the Sleep are located in:

Brainstem

Cerebellum

Left Hemisphere

Right Hemisphere

464. During Sleep:

Formation of urine decreases

Specific gravity decreases

Systolic pressure falls to about 90-110mm Hg

All of these

465. Synchronized waves are:

regular and invariant waves

Irregular and Variant Waves


Regular and Variant Waves

None

466. Which one of the following is Skilled memory?

Explicit memory

Non decorative memory

Long term memory

Short term memory

467. Which is responsible for Higher nervous activity:

extensive outer layer of gray matter

inner layer of white matter

Outer layer of White matter

Extensive Inner layer of gray matter

468. Established conditioned reflex is inhibited by some form of stimulus, points to:

Negative conditioned reflex

Neutral conditioned reflex

External unconditioned reflex

Internal unconditioned reflex

469. Following are the stages of NREM sleep, except:

Drowziness

Light sleep

Dream sleep

Deep sleep

470. Unconditioned reflexes are:

Natural reflexes

Inborn reflexes

None of the above

Both a and b

471. Demonstration of conditioned reflex was first made by:


Pavlov

William Harve

Robert Brown

Karl von Frisch

472. Classical conditioning is based on the work of which person?

Pavlov

Ellis

Freud

Bandura

Bowlby

None of these

473. Desynchronized waves are:

Regular and invariant waves

Irregular and Variant Waves

Regular and Variant Waves

None

474. Learning is:

The process by which new information is acquired

The process by which old information is revised

Not helpful

Very higher in the age of above 90

475. Which learning involves relations between two or more stimuli at a time.

Non-associative learning

Associative learning

Sensitization

habituation

476. Sleep apnea syndrome occurs in all of the following conditions, except:

Obesity
Myxodema

Lesions in brainstem

Smoking

477. Alteration of synaptic transmission between neurons involved, points to:

Recalling of Memory

Storage of Memory

Brain washing

Learning from Memory

478. Following are the types of spinal reflexes, except:

Segmental

Intersegmental

Suprasegmental

Extra segmental

479. Fixed or permanent memory is:

Tertiary memory

Secondary memory

Primary memory

Sensory memory

480. Reflex activity, most of the time is of the following nature:

Aggressive

Offensive

Protective

Depressive

481. Type of learning that involves response of a person to only one type of stimulus:

Associative learning

Non-associative learning

Both of above

None
482. Sometimes which is also referred as negative memory:

Facilitation

Habituation

Short term memory

Long term memory

483. Babinski's sign during deep sleep is:

Negative

Positive

None of the above

Both of the above

484. Threshold for most of the reflexes increases. Pupils are constricted. Light reflex is retained. Eyeballs move up and down.
These occur in:

Bad Learning

Good Learning

Sleep

Muscle Tone increasing

485. Which is not involved in Unconditioned reflex?

learning and memory

associative learning and associative memory

non associative learning and non-associative memory

None of above

486. Which of these is an unconditioned stimulus?

Food

Loud noise

Shock

Pain

None of these

all of these

487. Hypertension even Right heart failure can occur in severe cases of:
Sleep apnea syndrome

Narcolepsy

Cataplexy

Epilepsy

488. During Sleep:

Plasma level reduces

Plasma level increases

Plasma level is not changed

None of these

489. Loss of memory, points to:

Dementia

Amnesia

Brain death

Coma

490. One or more neurons, interposed between center and efferent or afferent nerve fibers are termed as:

Internuncial neurons

Sensory neurons

Motor neurons

Neuroglia

491. A word that is known today may be remembered till tomorrow. But if it is not recalled repeatedly, it may be forgotten on the
third day. It is an example of:

Short term Memory

Long term Memory

Moderate term Memory

Secondary Memory

492. An EEG:

provides indication of intelligence

tends to show waves of smaller amplitude during deep sleep than alert state

show waves with a lower frequency during intense thought than during deep sleep
is bilaterally symmetrical

493. Which one illustrates a reflex arc?

brain-Spinal cord- muscles

muscle-receptors-brain

muscles- spinal cord- brain

receptor-spinal cord-muscles

494. Basis of short term memory:

new synapses and their transmission

formation of new centers

Enlargement of Brain

Weight gain of Brain

495. What response did Watson and Rayner (1920) condition?

Anxiety

Happiness

Anger

Fear

Love

All of these

496. Dementia is common feature of:

Alzheimers disease

Meningioma

Hydrocephalus

Trigeminal neuralgia

497. Which drugs are shown to improve learning and memory:

Caffeine, nicotine, strychnine, Metrazol

epinephrine, adrenaline, Caffeine

amphetamine, nor-epinephrine, physostigmine

All of these
498. Reflex action is controlled by:

Autonomic nervous system

Peripheral Nervous system

Central Nervous System

None of these

499. Facilitation is the process in which Memory storage is:

Enhanced

Facilitated

Inhibited

Finished

500. Constant activity, consolidation and encoding of neuronal circuits into different areas of brain form the basis of:

Short term memory

Long term memory

Both of the above

None

501. Recalling first day of schooling, birthday celebration of previous year, picnic enjoyed last week:

Short term Memory

Long term Memory

Moderate term Memory

Tertiary Memory

502. Which type of reflex is developed with one unconditioned stimulus and one conditioned stimulus?

Secondary conditioned reflex

Primary conditioned reflex

Tertiary conditioned reflex

Negative conditioned reflexes

503. Types of conditioned reflex:

Classical

Instrumental
Permanent

Both a and b

Both b and c

504. There are how many types of Speech Disorder?

III

IV

VI

VII

505. Electrical activity recorded by EEG may have:

synchronized waves

desynchronized waves

Both

None of the above

506. New neuronal circuits are established in which stage of development of SPEECH?

1st Stage

2nd Stage

3rd Stage

4th Stage

507. When atleast one part of reflex arc is formed by the autonomic nerve fiber, the reflex is termed:

Visceral reflex

Somatic reflex

Spinal reflex

Congenital reflex

508. Recognition memory is the other name of:

Short term Memory

Long term Memory

Explict Memory
Implicit Memory

509. Mucous membrane reflex is the type of:

Deep reflexes

Superficial reflexes

Visceral reflexes

Tendon reflexes

510. Example of Polysynaptic reflex is:

Extensor reflex

Intersegmental reflex

Withdrawal reflex

Acquired reflex

511. Lesion of asending reticular activating system (ARAS) leads to:

Permanent somnolence/Coma

Insomnia

Wakefulness

Hypersomnia

512. Serotinin released by Raphe nucleus induses:

REM Sleep

NREM Sleep

Both of above

None

513. Which of the following is Speech center

Broca area

Wernicke area

Upper frontal motor area

Secondary oral center

514. Working memory is the second name of:

Short term Memory


Long term Memory

Medium term Memory

Primary Memory

515. Rapid conjugate movements of eye balls with Increased frequency points to:

Paradoxical sleep

slow wave sleep

Non-REM sleep

Both b and c

516. Loss of memory is a dangerous phenomenon. Which one is loss of memory?

Amnesia

Tremor

non-fixed memory

viral encephalitis

517. Delta waves with low frequency and high amplitude, points to which stage of NREM sleep?

Deep sleep

Drowziness

Light sleep

Medium sleep

518. Speech function is localized to which part of hemisphere in most of the people?

Left hemisphere

Right hemisphere

Central part of both hemispheres

It’s not located in any hemisphere

519. Sweat and Lacrimal secretion increases during:

Sleep

Remembering

learning

Associative Learning
520. Wernicke area is situated in:

upper part of parietal lobe

Upper part of temporal lobe

lower part of Occipital lobe

Inner part of Frontal lobe

521. Sleep requirment for growing children:

5 to 6 hours/day

12 to 14 hours/day

18 to 20 hours/day

7 to 9 hours/day

522. The NREM sleep is divided into how many stages?

II

III

IV

523. On the basis of past experience, which alters the behaviour of a person?

Memory

Learning

Reading

Writing

524. Theta waves are obtained generally in:

Children below age of 5 years

Teen agers

Adults

Aged women

525. Sensory terminal has 2 presynaptic components. One is presynaptic and the other is:

Facilitator

Enhancer
Reducer

Amplifier

526. Which type of reflex is developed with one unconditioned stimulus and two conditioned stimuli?

Secondary conditioned reflex

Tertiary conditioned reflex

Primary Conditioned reflex

Positive Coditioned Reflexes

527. Consolidation of memory is possible by:

Rehersal mechanism

Breathing mechanism

Sleep mechanism

Circulatory mechanism

528. Which is the type of sleep without the movements of eyeballs?

REM or Paradoxical Sleep

NREM or Slow-wave Sleep

Both REM & NREM Sleep

None of these

529. Which instrument is used to record EEG?

Electroencephalograph

Electrocardiograph

Electrogram

Electromusclograph

530. Which one of the following is the types of Sleep:

REM sleep

NREM sleep

REM sleep and NREM sleep

REM, NREM & NTM Sleep

531. Locus ceruleus of lower pons secretes which substance to induce REM sleep?
Serotinin

adrenaline

Nor-adrenaline

Dopamine

532. Wernicke area is involved in:

Speech synthesis

Speech understanding

Activation of peripheral speech apparatus

Activation of Central Speech Apparatus

533. The EEG rhythm having the lowest frequency is:

Alpha

Beta

Delta

Theta

534. Raphe nucleus is situated in:

Lower pons and medulla

Cerebellum

Cerebral cortex

Midbrain

535. Memory storage is attenuated in the process of:

Facilitation

Sensatization

Habituation

Retention

536. What forms the basis of higher mental functions?

congenital reflexes

Memory

Sleep
Conditional reflexes

537. Depending upon duration, memory is classified into all, except:

Sensory

Primary

Secondry

Tertiary

538. Speech is an integrated and a well-coordinated motor phenomenon. Which parts are involved in the mechanism of speech?

Cortical areas

Subcortical areas

both

Peripheral areas

539. Which is the natural periodic state of rest for mind and body with closed eyes characterized by partial or complete loss of
consciousness:

Memorizing Older information

Sleep

Learning

Body Rest

540. What is punishment?

Any consequence where something pleasurable is added

A classical conditioning technique

A tool for preventing extinction

Any consequence which makes behavior likely to reoccur in the future

Another form of negative reinforcement

Any consequence which makes behavior unlikely to reoccur in the future

541. In the diagnosis of neuro logical disorders and sleep disorders, is useful:

Electroencephalogram

Electrocardiogram

Electrogram

Electroneurogram
Electrobrainogram

542. How many there are stages in the development of SPEECH?

II stages

III stages

IV stages

V stages

VI stages

543. In Electrocardiograph how many electrodes are used?

Three Electrodes

Two Electrodes

Four Electrodes

Five Electrodes

544. It is the storage of information in brain for a longer period. The information could be recalled after hours, days, months or
years. It is:

Long term memory

Short term memory

Secondary memory

Primary memory

545. Non-declarative memory is the other name of:

Short term Memory

Long term Memory

Explict Memory

Implicit Memory

546. Activation of Locus ceruleus of pons results in:

Rapid Eye Movement Sleep

Non-Rapid Eye Movement Sleep

Both REM & NREM Sleep

None of these
547. What is the Role of Broca Area?

Speech understanding

Speech synthesis

Activation of peripheral speech apparatus

Activation of Central Speech Apparatus

548. Which of these is an unconditioned response?

Sweating

Blink

Salivation

Startle response

None of these

All of these

549. Epilepsy is divided into how many categories:

III

II

IV

550. In what memory storage is enhanced and involves increase in synaptic transmission and increased postsynaptic activity.

Habituation

Facilitation

Fast learning

None of these

551. Which of the following is not a type of Speech Disorder?

Aphasia

Dysarthria

Dysphonia

Stammering

None of these
552. Alpha rhythm includes frequency waves:

15-60 per second

1-5 per second

8-12 per second

22-26 per second

553. Nightmares are seen in:

REM sleep

NREM stage II

NREM stage III

NREM stage IV

554. Which of the following can be the cause of Aphasia?

Severe blow to head

Lumps on vocal cords

Paralysis of vocal cords

Hypthyroidism

555. Which type of reflexes inhibit the Conditioned reflexes?

Secondary Conditioned Reflexes

Tertiary Conditioned Reflexes

Negative Conditioned Reflexes

Positive Coditioned Reflexes

556. After searching and finding telephone number in the directory, we remember the number for a short while. It is an example
of:

Primary memory

Secondary memory

Long term memory

Short term memory

557. In vertebrates, simple reflex action is:

monosynaptic

polysynaptic
bisynaptic

trisynaptic

558. Stretch reflex is the best example of:

Polysynaptic reflex

Monosynaptic reflex

Both of the above

None of the above

559. Activation of Raphe Nucleus results in:

Rapid Eye Movement Sleep

Non-Rapid Eye Movement Sleep

Both REM & NREM Sleep

None of these

560. Which is the type of sleep associated with rapid conjugate movements of the eyeballs, which occurs frequently?

REM Sleep

NREM Sleep

Both NREM & REM Sleep

None of these

561. Sudden attack of uncontrolable sleep:

Coma

Shock

Stupor

Narcolepsy

562. EEG with spike and dome pattern is characteristic of epilepsy:

Jacksonian

Grandmal

Petitmal

Temporal lobe
563. Which of these is an example of conditioned reflex?

Sneezing

Yawning

withdrawl of hand on touching a hot plate

watering of mouth at the smell of food

564. Damage of speech centers impairs the expression and understanding of spoken words. It also affects:

reading

writing

both reading and writing

Reading, Writing, Vision

565. Pathalogical reflexes are classified according to:

Congenital basis

Response Basis

Clinical basis

Synapse basis

566. During the development of which type of reflex, animal is trained to perform a task in order to get some reward?

Operant Conditioned Reflexes

Classical Conditioned Reflexes

Unconditioned Reflexes

All types of reflexes

567. Which is the category of the Epilepsy:

Generalized Epilepsy

Localized Epilepsy

Both

None of these

568. Reflexes that protect body against gravitational force:

Flexor reflexes
Extensor reflexes

Aquired reflexes

Protective reflexes

569. Plantar reflex points to:

Stroking the thigh

Stroking dorsum of foot

Stroking the sole

Stroking the abdominal wall

570. Cataplexy points to:

Sudden outburst of emotions

Uncontrolable sleep

Nightmares

Nocturnal enuresis

571. Skilled memory is the other name of:

Short term Memory

Long term Memory

Explict Memory

Implicit Memory

572. Recalling of events of weeks, months, years or sometimes lifetime. It is related to:

Special type of Long term memory

Long term Memory

Moderate term Memory

Primary Memory

573. What is Sensitization?

A process by which person gets used to something, to which a person is constantly exposed.

A process by which the body is made to become more sensitive to a stimulus

A process by which old information is revised

None of these
574. Habituation is also reffered to as:

Positive memory

Negative memory

Neutral memory

Distant memory

575. Which is not the type of Learning:

Associative learning

Non-associative learning

Habituation

Sensitization

None of these

576. Followings are the components of Reflex arc, except:

Sensory nerve

End organ

Effector organ

Dura matter

577. Which functions are related to the higher nervous activity

Higher Intellectual functions

Higher Brain Functions

Higher Cortical Functions

All of these

578. During sleep following changes occur, except:

Rate and force of respiration is decreased

Salivary secretion is decreased

Formation of urine is decreased

Sweat secretion is decreased

579. Source of EEG:

A potential of pyramidal cells

A i l f li ll
A potential of ganglion cells

EPSP and IPSP of cortical cells which behave like dipoles

After potentials of parietalc cortex

580. Recollection of past experience for a very short period, on the basis of which an action is executed. It is related to:

Short term Memory

Long term Memory

Moderate term Memory

Primary Memory

581. Which is not the stage of Non-Rapid Eye Movement Sleep

Stage of Drowsiness

Stage of Light Sleep

Stage of Medium Sleep

Stage of Deep Sleep

None of these

582. Conditioned reflex forms the basis of

all higher intellectual functions

all higher nervous activity

all higher cortical functions

all higher brain functions

All of these

583. Explicit memory, points to:

Forgetfullness

Subjective recalling

Conscious recollection of past experience

Objective recalling

584. How many types there are of Sleep?

II

III

IV
V

585. The expression of thoughts by production of articulate sound, bearing a definite meaning is a process?

Skilled Speaking

Speech

Skilled presentation

Central Speech

586. What association is formed in secondary conditioning?

Association between two unconditioned responses

Association between extinction and recovery

Association between the first conditioned stimulus and a second conditioned stimulus in the absence of the unconditioned
stimulus

Association between unconditioned stimuli and conditioned responses

Association between two conditioned responses without conditioned stimuli

None of these

587. Bulbar or medullary reflexes are named because:

They have their centers in cerebral cortex

They are non protective

They have their centers in medulla oblongata

They are destructive in nature

588. Inhibition of Conditioned reflexes is of how many types?

589. Somnabulism:

Getting up from bed during sleep

Walking during sleep

Associated with psychoneurosis in adults

All of above
All of above

590. Short term memory, points to:

seconds to minutes

Months to years

Hours to days

Weeks to months

591. Following are the pupillary reflexes, except

Light reflex

Accomodation reflex

Ciliospinal reflex

Oculocardiac reflex

592. Memory depends on:

duration

non-associative learning

way of learning

associative learning

593. Which neurotransmitter plays role in Molecular basis of Facilitation:

Adrenaline

Dopamine

Serotonin

Neostigmine

594. Which of the following is controlled by autonomous nervous system?

Papillary reflex

Swallowing food

Knee-jerk response

peristalsis of intestine

595. Long term memory, points to:

Weeks to months
Years

Life time

All of the above

596. Implicit memory, points to:

Skilled memory

Non skilled memory

Crude memory

Raw memory

597. Beta rhythm includes frequency waves:

15-60 per second

1-5 per second

8-12 per second

15-28 per second

598. The process involved in facilitation of memory is called:

Memory sensitization

Negative memory

Positve Memory

Habituation

599. Mention the role of motor area?

Speech synthesis

Speech understanding

Activation of peripheral speech apparatus

Activation of Central Speech Apparatus

600. Desynchronized waves on EEG points to:

REM Sleep

Non-REM sleep

Both a and b

None of the above


601. Reflex developed with one conditioned and one unconditioned stimuli:

Primary conditioned reflex

Secondry conditioned reflex

Tertiary conditioned reflex

Negative conditioned reflex

602. Anatomical basis of Memory is:

Synapses

Basal ganglia

lateral ventricle

Corona radiata

603. What are the types of Inhibition of Conditioned reflexes

External inhibition & Internal inhibition

Primary, Secondary & Tertiary Inhibition

Associative and Non- associative inhibition

None of the above

604. The process involved in facilitation of memory is called:

Memory sensitization

Memory polarization

Memory inhibition

Memory decentralization

605. Hippocampus and Papez circuit (closed circuit between hippocampus, thalamus, hypothalamus and corpus striatum) are
the main sites for:

Memory encoding

Memory strengthening

Memory sensitization

None

606. Tone in all the muscles of body except ocular muscles decreases very much during sleep. It is called as:

Sleep Paralysis

Dehydrated Sleep
Tired Sleep

Normal Sleep

607. Reflex action in a vertebrate is an essential display induced by a:

sensory nerve

motor nerve

autonomic response

sympathic nerve

608. Which is characterized by combined features of Broca aphasia and Wernicke aphasia:

Global aphasia

Nominal aphasia

Wernicke aphasia

Broca Aphasia

609. Which of these is not a reflex reaction?

Salivation

Secretion of sweat

Flexion due to needle prick

Blinking of eyes due to strong light

610. When an electric shock is given to the leg of a pathed frog, it:

shows reflex response by contraction of leg muscles

blinks its eyes

shows no response

put out its tongue

611. Delta waves in EEG are seen in:

Deep sleep

REM sleep

awake with eyes open

awake with eyes closed


612. Acquired reflexes are developed mostly after:

Conditioning

Watching

Trauma

Disease

613. Which type of reflexes excite the Conditioned reflexes?

Primary Conditioned Reflexes

Secondary Conditioned Reflexes

Positive Coditioned Reflexes

Negative Conditioned Reflexes

614. Retaining sensory signals in sensory areas of brain, for a very short period of few seconds after the actual sensory
experience, i.e. few hundred milliseconds:

Sensory memory

Secondary memory

Long term memory

Short term memory

615. What type of memory can be recalled after hours, days, months or years?

working memory

Secondary memory

Sensory memory

remote memory

616. Delta rhythm includes frequency waves:

15-60 per second

1-5 per second

8-12 per second

15-28 per second

617. There are how many types of Learning?

3
2

618. Negative conditioned reflex is of the following types:

External

Internal

Both a and b

None of the above

619. The main difference between REM sleep and wakefulness is:

EEG desynchronization

Rapid eye movements

Decreased muscle tone

Penile erection

620. In normal person EEG has:

Three frequency bands

Five frequency bands

Nine frequency bands

Seven frequency bands

621. In which disease, EEG patterns are not altered:

Influenza

Epilepsy

Disorders of midbrain

Subdural Hematoma

622. Which of the following statements about fat and fat digestion/absorption is true?

Fat soluble vitamins are incorporated into mixed micelles and absorbed directly across the microvillus cell membrane

Triglycerides are resynthesized from fatty acids and monoglyceride in the intestinal epithelial cell and are exported directly into
the portal circulation

Pancreatic lipase functions best at a pH of 3

None of the above


None of the above

623. Which ONE of the following is not involved in the regulation of glomerular filtration rate (GFR)?

Juxtaglomerular apparatus

Arterial pressure

Efferent arteriolar tone

Na content in distal tubule

624. Inhibition of the myenteric plexus leads to which of the following?

Increased Secretion of Secretin from the duodenum.

Decreased gut motility.

Hyperacidity in the stomach.

Diarrhea.

625. Every mucosa consists of:

A simple epithelium

A surface epithelium together with associated glands

A surface epithelium layer, with or without glands with an underlying layer of loose connective tissue.

A surface epithelium layer, with or without lands together with an under lying connective tissue and a layer of smooth muscle.

626. The hormone that inhibit gastric secretion and motility is...

gastrin

histamine

secretin

cholecytokinin

627. Proteins are primarily digested to and absorbed as...

amino acids

monosaccharides

nucleic acids

fatty acids & glycerol

628. Increased GFR is caused by

Increased cardiac output


Afferent arteriolar vasoconstriction

Efferent arteriolar vasodilatation

Increased chloride delivery to the macula densa

629. Which of the following does not enter the duodenal lumen?

chyme from stomach

bile salts

disaccharidases

pancreatic enzymes

630. Which is the primary absorptive organ of the digestive system?

small intestine

stomach

colon

pancreas

631. The role of Chief cell is to:

Secretes hydrogen ions and chloride ions into lumen and are responsible for maintaining the acidic pH

Secrete pepsin which digests proteins.

Secretes Gastrin which stimulates gastric acid secretion

Secretes pepsinogen.

632. Renal blood flow:

Greater per unit mass than cerebral blood flow

Is greater in the medulla compared to the cortex

Is closely related to tubular sodium reabsorption

Only sympathetically mediated

633. Chief cells of gastric mucosa secrete...

trypsin

bicarbonate ions

pepsinogen

HCl
634. It is known that secretin:

It is a large protein hormone synthesized by the pancreas,together with pancreozymin.

Is a small polypeptide synthesized by the intestinal mucosa .

Neutralizes directly the acid chyme that passes through the pylorus.

Has an optimal activity at a PH equal to 8.4.

635. The minimum pH that the urine can create is

3.0

3.5

4.0

4.5

636. Which of the following factors will not influence the rate of gastric emptying?

fat in the duodenum

acid in the duodenum

caffeine in the duodenum

distension of the duodenum

637. Water handling by kidney (% reabsorption)

93%

94%

99%

99.4%

638. Which of the following is an active proteolytic enzyme?

pepsinogen

secretin

procarboxypeptidase

none of the above

639. The layer of tissue beneath the mucosa is called the:

Muscularis externa

Circular Muscle

S
Serosa

Submucosa

640. What occurs during swallowing?

All the processes of Deglutition are under voluntary control.

The vocal folds are relaxed allowing the food to travel down the oesophagus

The swallowing centre located within the medulla oblongata inhibits themedullary respiratory centre

The food bolus travels down the oesophagus due to gravity

641. Distension of the stomach...

causes glucose absorption

results in salivation

triggers gastrocolic reflex

inhibits gastric emptying

642. Which of these is most completely re-absorbed in the kidneys

Albumin

Glucose

Calcium

Potassium

643. In a patient with significant hypovolaemia and increased osmolality:

Decreased sodium reabsorption at distal tubule & collecting duct

Increased water reabsorption at ascending loop of henle

Increased water permeability of collecting duct

None of the above

644. Bile salts...

emulsify proteins

are formed from cholesterol

are acidic in nature

are needed for carbohydrate digestion

645. The formula for GFR is:


GFR = Kf (HPG - HPB + OPG - OPB)

GFR = Kf (HPG - HPB - OPG + OPB)

GFR = Kf (HPG + HPB - OPG + OPB)

GFR = Kf (HPG + HPB - OPG - OPB)

646. The pancreas arises from which embryonic germ layer?

Mesoderm

Ectoderm

Mesendoderm

Endoderm

647. Which of the following results in decreased K+ excretion?

Prolonged vomiting

Metabolic acidosis

Normal saline infusion

Renal failure

648. It is known that gastrin:

It is a large protein molecule, somewhat similar in size to pepsin.

Is not secreted by empty stomach when peristaltic movements may be quite forceful.

Reaches the secretory cells of the fundus of the stomach through the blood and not through the lumen.

Promotes the secretion of pepsin, but not that of HCL.

649. Cimetidine and a calcium-containing antacid preparation administered in an appropriate dosage regimen for treating
peptic ulcer in that:

Only the calcium-containing antacid may increase gastric acid secretion.

Only cimetidine is likely to cause an increase in urinary pH.

Only cimetidine is likely to produce a laxative effect.

Only cimetidine can prevent the rise in gastric pH after the consumption of a meal

650. The clearance (or 'renal regulation') of which ONE of the following is NOT regulated by a hormone:

Sodium

Potassium

Calcium
Sulphate

651. Lactose intolerance is the inability to digest lactose, due to insufficient amounts of the enzyme lactase. Where is lactase
usually found?

Brush border of duodenum

Lumen of the small intestine

Lumen of the stomach

Brush border of the jejunum

652. What path do bile salts take?

Gall bladder, stomach, duodenum, juejunum, ileum, tissues

Liver, gall bladder, duodenum, jejunum, tissues

Gall bladder, stomach, duodenum, tissues, liver

Liver, gall bladder, duodenum, jejunum, ileum, liver

653. HCl secretion by the stomach...

is triggered by chewing

denatures proteins

activates enzymes that digest proteins

all of these +

654. Mucosa surface cell secretes mucous which provides a physical barrier between ______________ and __________.

Mucosa and Epithelium

Lymph Vessel and lamina Propria

Mucosa and Lumen

Lumen and Epithelium

655. Choose the best description of the duodenum.

Mucosa with crypts and villi; epithelium with sca ered goblet cells; laminapropria with occasional lymph nodules; submucosa
unspecialized.

Mucosa with crypts but no villi; epithelium with sca ered goblet cells; laminapropria with occasional lymph nodules;
submucosa unspecialized.

Mucosa with both crypts and villi; epithelium with sca ered goblet cells; laminapropria with occasional lymph nodules;
submucosa with mucous glands.

Mucosa with both crypts and villi; epithelium with sca ered goblet cells; laminapropria packed with mucous glands;
submucosa with numerous lymph nodules
656. Which of the following organs secrete mucus?

salivary glands

colon

stomach

all of the above

657. Some absorption occurs in the large intestine, but this is mostly limited to...

amino acids

water & electrolytes

fatty acids

buffers and mucous secreted by the large intestinal mucosa

658. Increased stomach pH is the consequence of which of the following?

Damage to the mucous cells

Increased G cell activity

Inhibition of parietal cells

Overexpression by chief cells

659. Resistance to renal blood flow is chiefly determined by:

Renal artery

Afferent & efferent arterioles

Interlobular & arcuate arteries

Peritubular capillaries

660. During swallowing...

all processes are consciously controlled

the swallowing center located in the medulla oblongata inhibits the medullary respiratory center

the vocal folds are relaxed

the food bolus is forced down the esophagus by gravity

661. During which stage of Swallowing is respiration inhibited?

Voluntary stage.

Pharyngeal stage
Pharyngeal stage.

Esophageal stage.

Postprandial stage.

662. Regarding glucose handling in the kidney

Reuptake is passive

Tm is the same for all nephrons

D-glucose more rapidly absorbed than L-glucose

Reabsorption is inversely proportional to lipid solubility

663. Stomach acid is neutralized...

in the duodenum

by secretions secreted by antrum

in the duodenum with trypsin

with trypsin

664. Intrinsic factor is...

secreted by parietal cells and is responsible for vitamin B12 absorption

not necessary for normal body function

abundant in pernicious anemia

an aminopeptidase

665. The salivary secretion:

Is stimulated by most GIT hormones specially gastrin.

Is essential for complete digestion of starch.

Increases more by sweet than by bitter substances.

Markedly Increases by parasympathetic stimulatation.

666. For an organ lying within a body cavity, the tissue which is farthest away from the inner core or lumen of the organ is called
the:

Muscularis externa

Serosa

Mucosa

Myenteric Plexus
667. Amylase secreted by the salivary glands into the oral cavity starts the digestion of:

Proteins

Lipids

Starch

Amino acids

668. Regarding pancreatic secretion:

Pancreatic secretion is inhibited by gastrin secreted by the G cells of the antrum

Pancreatic acinar cells contain trypsin

Cholecystokinin inhibits secretion from the exocrine pancreas.

The introduction of acid into the duodenum stimulates pancreatic secretion.

669. The permeability of glomerular capillaries:

Equals that of other capillaries

Is much less than that of other capillaries

Is equal for cationic and anionic molecules of equal size

Is about 50 times as great as that of a skeletal muscle capillary

670. What prevents food from entering the nasal passages during swallowing?

elevation of uvula

contraction of pharyngeal muscles

apposition of vocal folds

elevation of epiglottis

671. The intrinsic nerve plexuses...

are located in the mucosal region

network of neurons that act as "gut" brain

are made up of two plexuses

two of the above

672. Secretin IS released by:

Acid in the duodenum.

Acid in the urine.


S cells in the duodenal mucosa.

Distension of the colon.

673. Bile is produced by the liver continuously. (Which of the following is true?)

bile functions to catalyze digestive processes in a manner similar to enzymes

bile salts emulsify large droplets of triglycerides

bile salts reduce the surface area of droplets of triglycerides

95% of bile salts are lost in the feces

674. Which tissue layer provides for primary digestive motility?

mucosa

submucosa

mesentery

muscularis mucosa

675. Concerning the gastrin hormone:

It is Secreted at the pyloric antrum and reaches the fundus tohrough the gastric lumen.

It Promotes the secretion of pepsin, but not HCL.

Its Secretion Stimulated by secretin and GIP.

It is structurally similar to CCK .

676. Which of the following is absorbed by the gastric mucosa?

glucose

caffeine

aspirin

HCl

677. Increase in GFR occurs with

Increased sympathetic stimulation

Decreased renal blood flow

Hypoproteinaemia

Ureteric obstruction

678. Which has the greatest renal clearance?


PAH

Glucose

Urea

Water

679. The serosa is...

abdominal cavity containing the stomach inner lining of the digestive tract

outer connective tissue covering of the digestive tract

layer of smooth muscle of the digestive tract

layer of smooth muscle of the respiratory tract

680. The pharyngeal phase of Swallowing:

Can be voluntairly inhibited.

Involves closure of larynx by the vocal cord.

Is associated with lowering of the soft palate.

Is accompanied by deep inspiration.

681. Apatient with trigeminal lesion would have the greatest difficulty with which of the following?

Swallowing

Chewing

Receptive relaxation of the upper esophageal sphincter

Secondary prestalsis in the esophagus

682. Which of the following is involved in the regulation of glomerular filtration rate (GFR)?

Juxtaglomerular apparatus

Afferent arteriolar tone

Efferent arteriolar tone

All of the above

683. Salivation can become a conditioned reflex .This suggests that:

Pleasant taste sensation are not related to the reflex .

Only salivatory nuclei in the brainstem need to be excited by taste sensation without participation of suprasegmental
influences.

The cerebral cortex partially controls salivation.


salivation could be completely interrupted in a decorticate animal whose tongue is mechanically stimulated.

684. The ileocecal sphincter...

prevents acid from entering the esophagus

prevents bacterial growth in the stomach

prevents bacterial infection of small intestine

inhibits gastric emptying

685. Which tissue layer provides for primary digestive motility?

Mucosa

Submucosa

Mesentery (a fold of tissue that a aches organs to the body wall)

Muscularis Mucosa

686. The role of parietal cells is to:

Secretes hydrogen ions and chloride ions into lumen and are responsible for maintaining the acidic pH

Alkaline mucous for shielding the epithelium from hydrochloric acid

Secretes histamine which stimulates gastric acid secretion

Secretes gastric lipase

687. Mastication is important because:

Mastication is important because:

Allows the salivary enzymes to act for a longer time.

Increase the surface area of the food particles.

All of the above.

688. In which region of the GI tract is the longitudinal muscle of the muscularis arranged into distinct longitudinal bundles?

Duodenum

Jejunum

Ileum

Colon

689. Select the true statement regarding the control of gastric secretion:

Gastric acid is secreted by parietal cells of the gastric glands in response to hormonal stimulation.
yp g g p

Most of the secretion of acid and pepsinogen occurs during the intestinal phase.

Most of the secretion of acid and pepsinogen occurs during the intestinal phase.

Secretin secreted by the duodenum stimulates gastric secretion.

690. Kidneys produce:

Erythropoietin

ADH

Angiotensin II

ANP

691. Arterial pressure

Sympathetic nervous system

Sodium flow past macula densa

Afferent arteriolar vasodilatation

Arterial pressure

692. Peristaltic antral contractions...

occur at the rate of 3 per minute

are responsible for gastric emptying

are responsible for mixing food and gastric enzymes within the antrum

all of these

693. Renal blood flow is dependent on:

Juxtaglomerular apparatus

[Na+] at macula densa

Afferent vasodilatation

Arterial pressure

694. Submucosal glands occur in the submucosa of the ________ and they also occur in the _________.

Oesophagus and ileum

Stomach and Duodenum

Stomach and ileum

Oesophagus and Duodenum


695. Significant tubular reabsorption occurs with:

Phosphate

Creatinine

Urea

All of the above

696. Angiotensin II causes:

Increases proximal tubular reabsorption of Na & H2O & increases secretion of K+

Increases distal tubular reabsorption of Na & H2O & decreases secretion of K+

Decreases distal tubular reabsorption of Na & H2O

Increases excretion of Na & H2O

697. High osmolarity of renal medullary interstitium is due to:

secretion of H2O into ascending loop of Henle

diffusion of H2O into ascending loop of Henle

active transport of Na from vasa recta

passive reabsorption of urea from collecting duct

698. About the GIP,all the following are true except:

It is secreted from the duodenal mucosa.

It has have feedback effect on gastric Secretion and motility.

Its release is Stimulated by presence of excess fat.

It inhibit both gastric Secretion and motility.

699. Which is not a brush border enzyme?

lipase

enteropeptidase

sucrase

aminopeptidase

700. The Secretin hormone:

Is secreted by the pancrease.

Is released by the pyloric mucosa.


Contracts the gall bladder wall.

Increases the pancreatic Secretion of water and HCO3-.

701. In most regions of the GI tract, smooth muscle fibers of the muscularis are arranged into:

Circular muscle (inner) followed by longitudinal muscle (outer)

Longitudinal muscle (inner) followed by Circular muscle (outer)

Circular and Longitudinal muscle whose relative position varies at different regions

Circular muscle (inner) followed by Longitudinal muscle and Serosa (outer)

702. Glomerulotubular balance:

Involves afferent arteriole feedback loop

Involves efferent arteriole feedback loop

Juxtaglomerular complex

Ability to increase tubular absorption in response to an increase in filtered load

703. The effect of PTH on the kidney is to:

Increase Ca excretion and decrease phosphate excretion

Decrease Ca excretion and increase phosphate excretion

Decrease Ca excretion and decrease phosphate excretion

None of the above

704. Intrinsic factor is required for the intestinal absorption of _________ in the upper ileum:

Vitamin D

Amino Acid

Vitamin B12

None of the above

705. Water filtration by the kidney:

Is 180 l/hr

Is 125 ml/min

Up to 90% is reabsorbed

Most drugs have MW less than 600 and are freely filtered
706. Glomerular filtration rate (GFR):

Is independent of the size of the capillary bed

Depends only on the hydrostatic and osmotic pressure differences across the capillary

Is determined by the same forces governing filtration across all other capillaries

Depends only on the permeability of the capillary

707. Lipids...

are broken down by pepsin

are absorbed into lacteals

become part of micelles in the duodenum

two of the above [B&C]

708. Regarding water reabsorption in the collecting tubules:

depends on aldosterone levels

collecting tubules able to reabsorb 60-70% of water

depends on renin levels

loops of henle are ONLY located in the renal medulla

709. The products of the liver and pancreas participate in what type of digestion?

Both mechanical

Chemical and mechanical, respectively

Both chemical

Mechanical and chemical, respectively

710. SGLT1 is a symporter that can be found in the small intestine. Which macromolecule does it transport?

Monosaccharides

Nucleic acid

Fatty acids

Amino acids

711. Bile acts on...

distention in the stomach

acid in the stomach


fat in the stomach

fat in the duodenum

712. The Secretion of gastrin cease(stop):

When the stomach is distended by meal.

When the PH of the gastric content is Decreased below 2.

If the fundic mucosa is anaesthesized.

If the vagi are Stimulate. (E)If histamine is injected.

713. The presentation of a bolus of solid food to the mouth:

stimulates taste buds.

Is usually followed by Mastication.

Reflexively stimulates The salivary glands.

All are correct.

714. Folds in the mucosa of the stomach are called _________ which allows the surface area to _________.

Crypt, Increase

Rugae, Increase

Rugae, Decrease

Crypt, Decrease

715. Simple tubular glands within the mucosa of the small or large intestine are called intestinal:

Villi

Peyer’s Patch

Crypt

Pits

716. Pressure diuresis:

Due to decreased reabsorption of Na+ & water in peritubular capillaries

Regulated by macula densa

Increase ADH

Increase angiotensin
717. The ascending limb of the Loop of Henle is:

Impermeable to Na+

Involved in active transport of K+ into the lumen

Involved in active transport of Cl- out of lumen

Involved in active transport of Na+ into lumen

718. Where does the production of bile occur?

Gallbladder

Liver

Pancreas

Hepatic Ducts

719. Venous blood flows into the liver via...

hepatic vein

hepatic portal system

sinusoids

canaliculi

720. The major chemical digestive activity that takes place in the stomach is...

breakdown of starch

breakdown of proteins

digestion of fats

neutralization of acid by buffers & mucous

721. Glycosuria is most likely to occur with:

increased GFR and increased blood glucose level

decreased GFR and increased blood glucose level

decreased GFR and decreased blood glucose level

increased GFR and decreased blood glucose level

722. Which of the following is true about pancreatic secretions?

CCK causes release of bicarbonate

secretin causes release of enzymes


secretin causes release of enzymes

gastrin stimulates release of enzymes

CCK stimulates release of enzymes while secretin stimulates release of bicarbonate

723. The major factor that Stimulates the release of Secretin into the blood stream is:

An acid PH of the chyme entering the duodenum.

The para sympathetic StimulI.

Peptones in the gastric chyme that enter the duodenum.

A stomach full of digested contents.

724. Gastric emptying is primarily controlled by...

the neuronal signals emanating from the medullary swallowing center

the volume of chyme in the stomach

conditions/factors in the duodenum

the degree of fluidity of the chyme in the stomach

725. The volume of air remaining in lungs after normal expiration called:

Inspiratory capacity

Vital capacity

Functional residual capacity

Total lung capacity

726. Which of the following is NOT a form by which CO2 can be transported in the blood?

As bicarbonate

Dissolved in the blood.

Bound to the amino end groups in proteins.

Bound to the imidazole ring of glutamate.

727. If the FIO2 is .21, the FEO2 is .16, the VT is 0.5 L, and the frequency of breathing is 12. What is the VO2? The equations are
VO2=VI * (FIO2 - FEO2) and VI = VT * f.

3.0 L/min

0.75 L/min

-0.75 L/min

0.3 L/min
728. An asthma sufferer finds she has to breathe at twice her normal rate. How does that affect her dynamic compliance?

It stays the same.

It decreases.

It increases.

Static compliance, not dynamic, is the variable affected by asthma.

729. Which of the following is FALSE concerning the airflow in the lungs?

During inspiration and expiration, the flow in the trachea and larger bronchi is turbulent.

Towards the middle of the bronchial tree, the flow is turbulent at the branches and laminar in between

Near the end of the bronchial tree, the flow is laminar.

The acini have very small radii which significantly increases the total air flow resistance of the bronchial tree.

730. To breathe in

external intercostals are contracted and the diaphragm moves superiorly

external intercostals relax and diaphragm moves inferiorly

transversus thoracis, internal intercostals and rectus abdominals are all necessary(d) the transversus thoracis and diaphragms
must meet

external intercostals relax and diaphragm dont moves

731. Which of the following is INCORRECT concerning the O2 / CO2 movement and processing through the lungs and tissues?

Binding of O2 to Hb changes its configuration so that CO2 and H+ ions are more likely to dissociate.

When CO2 diffused into the alveoli, the PaCO2 is lowered.

Carbonic acid is an intermediate in the reaction combining H+ with HCO3- to form H2O and CO2.

Arterial blood flows to the tissues where H+ ions combine with HCO3- to form H2O and CO2.

732. Which of the following will NOT increase the minute ventilation?

An increase in arterial pH.

An increase in arterial partial pressure of carbon dioxide.

Increase in alveolar pressure of carbon dioxide.

Exercise

Hypoxia

733. The volume of air present in lungs after a deep (maximal) inspiration called:

Inspiratory capacity
Vital capacity

Functional residual capacity

Total lung capacity

734. Which of the following is NOT a function of the lungs?

Metabolism

Serves as a reservoir of blood for the left ventricle

It is a filter to protect the systemic vasculature

Facilitates the exchange of O2 and CO2 between air and blood.

All of the above are true.

735. What is name of the phase during which air enters the lungs from atmosphere?

Expiration

Inspiration

Aspiration

Respiration

736. Which of the following is FALSE concerning diffusion in the lungs?

CO is used to measure diffusing capacity because its uptake is diffusion limited.

Actual diffusion time includes time required for an O2 molecule to diffuse from the alveolus, through the membrane, the
plasma, and into a RBC.

Reaction time is the time it takes the O2 molecule to react with Hb.

The diffusion time is greater than the reaction time of an O2 molecule.

737. Which of the following does NOT happen during inspiration?

The ribs move upward

The diaphragm lifts up

The antero-posterior dimensions of the chest are increased

The tranverse dimensions of the thorax are increased

The scalene and sternocleidomastoid muscles can be recruited for inspiration

738. A 140 lb woman would have approximately how much dead space in her lungs?

140 ml.

70 ml.
70 ml.

280 ml.

35 ml.

739. Esophagus opens only during

peaking

exhaling

inhaling

swallowing

740. What is maximum volume of air that is inspired after normal expiration?

Inspiratory capacity

Vital capacity

Functional residual capacity

Total lung capacity

741. The site of gas exchange is:

primary bronchus

trachea

alveoli

cilia

742. Parasympathetic innervation of the lungs via the vagus nerve:

dilates the bronchioles and uses ACh

constricts the bronchioles and uses ACh

constricts the bronchioles and used adrenaline

dilates the bronchioles and uses adrenaline

743. Which one of the following statement is NOT correct regarding trachea?

It usually lies posterior to the muscular esophagus

It splits into the right and left bronchi to supply air to the lungs

Opening to the trachea is covered by epiglottis

Tracheal rings are C-shaped


744. Which of the following can cause stagnant hypoxia?

COPD

Shock or heart failure.

Cyanide poisoning.

Carbon monoxide poisoning.

745. Which of the following is NOT Components of tracheobronchial tree?

primary bronchi

tertiary bronchi

fetal bronchiole

terminal bronchiole

respiratory bronchioles

746. The delivery of oxygen to muscle depends on

blood flow and oxygen content

ACh

stimulation of the vagus nerve

Noradrenaline

747. Which of the following is the primary regulating variable of the central chemoreceptors?

PaO2

PaCO2

arterial pH

Input from stretch receptors

748. Calculate PAO2 for a person at sea level for R = 0.82 and PACO2 = 40 Torr.

110 Torr.

95 Torr

80 Torr

101 Torr

749. Exchange of oxygen and carbon dioxide between alveolar air and blood is governed by _______.

diffusion
blood pressure

gravity

active transport

750. Which of the following concerning average lung volumes and capacities of a person at rest is TRUE?

TLC>VC>TV>FRC

TLC>FRC>VC>TV

TLC>VC>FRC>TV

TLC>FRC>TV>VC

751. Which of the following is FALSE concerning the ventilation and perfusion of different regions of the lung?

Alveoli at the top of the lung have a smaller dynamic compliance

The Hb moving through the base of the lung is less saturated than that at the apex of the lung

PAO2 at the apex of the lung is higher than that at the base of the lung

Regional variation in ventilation-perfusion is more efficient for oxygenating blood than is uniform ventilation-perfusion

Variation of the ventilation/perfusion ratio in the lungs only becomes significant when lung function begins to degrade

752. According to the Law of Laplace, air should flow from the smaller alveoli to the larger, collapsing them. In the lungs, several
factors counter that tendency, and stabilize the alveolar structures. Which of the following is NOT one of them?

Surfactant lowers surface tension to a greater degree when it is on a smaller surface area, allowing the smaller alveoli to stay
open.

Mechanical stability is given by surrounding alveoli.

Transpulmonary pressure is lower for smaller alveoli, allowing them to stabilize in comparison to the bigger ones.

Surface tension at the gas-liquid interface increases as alveolar surface area increases.

753. If you blocked the blood supply to an alveolus, which of the following would NOT occur as a result?

The ventilation perfusion ratio would be 0.

The PAO2 would be greater than normal.

The PACO2 would be 0.

All of the above are true.

754. During expiration the diaphragm becomes

Dome-shaped

Flattened

Normal
Normal

Oblique

755. Which of the following represents the pressure difference that acts to distend the lungs?

Alveolar pressure

Airway opening pressure

Transthoracic pressure

Transpulmonary pressure

Esophageal pressure

756. Which of the following is FALSE concerning the closing volume for the lung?

Comes between Phase 3 and Phase 4 on the single breath N2 washout curve.

Marks the point where the alveoli at the apex close.

Marks a sudden increase in nitrogen concentration in the expelled breath.

Marks when the overinflated, poorly ventilated alveoli at the apex expel their air with high N2 concentrations.

757. Which of the following spirometry measurements has the greatest sensitivity for detecting early air flow obstruction?

FVC

FEV1

FFE

FEF25-75

758. Parasympathetic innervation of the lungs via the vagus nerve:

dilates the bronchioles and uses ACh

constricts the bronchioles and uses ACh

constricts the bronchioles and used adrenaline

dilates the bronchioles and uses adrenaline

759. How many oxygen molecules bound to hemoglobin to give 50% saturation?

7
760. Which of the following is NOT a normal occurance with increasing age?

Vital capacity of the lung decreases

Residual volume increases

Functional residual capacity increases

Inspiratory capacity decreases

Expiratory reserve volume increases

761. Which of the following is NOT a non-respiratory function of respiratory tract?

Olfaction

Vocalization

Anticoagulant function

Immunological defense

Erythropoietic function

762. Which of the following is FALSE concerning chemoreceptor input to the respiratory centers?

CSF is a poor buffer and a drop in PCO2 produces a large change in pH initiating a change in respiration.

The body can adjust to chronic hypercapnea by using an active HCO3- transport process in the choroid plexus.

The carotid and aortic bodies detect increases in PaCO2 and pH, and decreases in PaO2

75% of ventilatory response is regulated by chemoreceptors in the CSF and 25% by the carotid and aortic bodies.

Central chemoreceptors tend to respond slowly over time, while carotid bodies react quickly to immediate needs.

763. What is Normal Respiratory Rate in adults?

12 to 16/minute

30 to 60/minute

20 to 40/minute

10 to 11/minute

764. Each of the alveolar sac is surrounded by

capillaries

veins

arteries

primarily lymphatic ducts


765. Which of the following pairs is INCORRECT concerning central nervous systems and a factor they respond to by affecting
respiration?

Cerebellum: Mechanoreceptor input

Limbic system: emotional states

Cerebral cortex: voluntary control

Cerebral motor cortex: exercise

766. Which of the following is FALSE concerning the relationships of the variables in diffusion of O2 across a membrane?

Doubling the thickness of the membrane would cut the total flow of O2 in half

Doubling the area of the membrane would double the total flow of O2

If you increased the alveolar concentration of O2, you would increase the total flow of O2 across the alveolar membrane

The lower the diffusion coefficient, the higher the total flow

Increasing the arterial concentration of O2 would decrease the total flow of O2

767. If the blood moved slower than normal through the alveolar capillaries, which of the following would have an increased
uptake?

Carbon dioxide.

Carbon monoxide.

Oxygen

None of the above.

768. Sympathetic innervation to the lungs:

dilates the bronchioles and uses ACh

constricts the bronchioles and uses ACh

constricts the bronchioles and uses adrenaline

dilates the bronchioles and uses adrenaline

769. If you increased the left atrial pressure from 5 mmHg to 15 mmHg, what effect would that have on pulmonary circulation?

It would force blood the opposite direction

It would increase the speed at which blood moves through the pulmonary circulation

No change

Blood flow would almost or completely stop


770. Which of the following is the first branching of the bronchial tree that has gas exchanging capabilities?

Terminal bronchioles

Respiratory bronchioles

Alveoli

Segmental bronchi

Alveolar ducts

771. If the equal pressure point during expiration is in the lobar bronchi, which of the following is TRUE?

Expiratory flow would be effort dependent.

Expiratory flow would be effort independent.

The bronchi beyond the equal pressure point would compress.

This situation would only occur during medium and low lung volumes.

772. Which of the following definitions is FALSE?

O2 content of blood is the actual amount of O2 in one deciliter of blood

O2 saturation of blood is the ratio of O2 content to its O2 capacity

The O2 uptake curve of blood is the functional relationship between O2 content and PO2

The O2 content of blood depends completely on the amount of Hb in the blood

773. During inspiration, how does alveolar pressure compare to atmospheric pressure?

Alveolar pressure is greater than atmospheric

Alveolar pressure is less than atmospheric

Alveolar pressure is the same as atmospheric

Alveolar pressure is one of the few pressures where the reference pressure is not atmospheric

774. Air can enter the body and travel to the lungs ...

through the mouth and the nose

through the oesophagus and gullet

through the windpipe and the pores

through the nose and the nervous system

775. Breathing rhythm is generated by


the pontine respiratory centres

the ventral respiratory group

the dorsal respiratory group

the alveoli

776. Which of the following is in the correct path of CO2 from the tissue to the atmosphere?

Reaction with H2O to make H2CO3, dissociation to H+ and HCO3-, H+ combines with imidazole side chain of hemoglobin,
carried back to lungs as HHb+ and HCO3-, reverse reaction forms CO2.

O2 is metabolized to CO2, reaction with H2O to make H2CO3, H2CO3 combines with imidazole side chain of hemoglobin,
H2CO3Hb+ is carried back to the lungs, reverse reaction forms CO2.

Reaction with H2O to make H2CO3, dissociation to H+ and HCO3-, HCO3- combines with imidazole side chain of hemoglobin,
carried back to the lungs as HCO3-Hb+ and H+, reverse reaction forms CO2.

O2 is metabolized to CO2, reaction with H2O to make H2CO3, dissociation to H+ and HCO3-, carried back to lungs in this form,
reverse reaction forms CO2.

777. Which of the following statements about Hb is FALSE?

A higher P50 than normal means that the O2 binds less tightly to Hb.

An increase in 2,3-DPG shifts the O2 uptake curve to the left.

An increase in PCO2 causes a right shift of the O2 uptake curve.

An decrease in pH increases P50.

An increase in temperature shifts the O2 uptake curve to the right.

778. How do you calculate how much inspired air actually ventilates the alveoli during one minute?

Subtract the volume of dead space from the tidal volume.

Subtract both the dead space volume that was already in the lungs plus the dead space of the inspired air that won't reach the
alveoli from the tidal volume.

Subtract the volume of dead space from the tidal volume and multiply it by the number of breaths per minute.

It is equal to the tidal volume times the frequency of breathing.

779. Which of the following is NOT true concerning respiratory distress syndrome in premature infants?

Their ability to synthesize DPPC is limited.

Higher pressures are required to ventilate the lungs.

Lung compliance is low.

Positive pressure respirators are often used to assist them in breathing.

Alveoli tend to overexpand and sometimes burst at the end of inspiration.


780. If the lung were punctured, which of the following would happen?

The lung would collapse on the side of the puncture.

Both the lung and the chest wall would collapse on the side of the puncture.

The relaxation pressure of the chest wall would increase until it surpassed the atmospheric.

The relaxation pressure of the chest wall would increase, but stop before it reached atmospheric pressure.

781. In which situation would the response to hypoxia be limited?

In a patient with obstructed airways.

When hypoxia is accompanied by hypercapnea.

In the hypoxia induced by high altitude.

During hypoventilation.

782. Acidosis will...

make you breathe more

make you breath less

make your breathing uneven

make your kidneys produce more adrenaline

783. Which of the following is NOT an effector of respiration?

heart

diaphragm

intercostals

trapezius

784. What part of the lung has an area of zero blood flow?

Apical Portion

Midportion

Lower Portion

Inner Portion

785. Which of the following is TRUE if a patient breathes slower than normal with increased tidal volumes?

More resistive work is done.

The total work done decreases.


More elastic work is done.

Compliance is decreased.

786. What is NOT Respiratory Protective Reflex?

Cough reflex

Sneezing reflex

Brachioradialis reflex

Swallowing reflex

787. If a patient had a progressive lung disease that required an ever increasing pressure to fill the same volume of lung, how
would the lung's compliance be affected?

It would increase it.

It would stay the same.

It would decrease it.

These variables do not affect lung compliance.

788. Which of the following is FALSE concerning airway resistance?

Up to 50% is in the nose.

The maximum resistance in the bronchial tree occurs at the fourth generation.

In the later generations, the radii are smaller, increasing the total resistance at each successive generation.

Airway resistance can be increased by loss of tissue elasticity and contraction of bronchial smooth muscles.

789. Which of the following pairs is NOT a pulmonary mechanoreceptor paired to a possible stimulus?

Stretch receptor: inflation

Irritant receptor: inhaled dust

Juxtacapillary receptors: decreases interstitial fluid volume in alveolar walls.

Bronchial C receptors: large inflations.

790. Which of the following in FALSE concerning CO2 uptake?

If PO2 = PCO2, then there will be more total CO2 in the blood.

Oxygenation moves the CO2 uptake curve downward.

The CO2 uptake curve is generated by comparing the total CO2 per unit volume of blood, and the PCO2.

Deoxygenated blood carries less CO2 than oxygenated.


791. To which of the following is alveolar PCO2 directly proportional?

Rate of CO2 production and alveolar ventilation.

Rate of CO2 production and rate of O2 consumption.

Alveolar ventilation and rate of O2 consumption.

Alveolar ventilation, rate of O2 consumption, and rate of CO2 production.

792. Dissociation curve shifts to the right when

CO2 concentration decreases

CO2 concentration increases

O2 concentration decrease

CL− concentration increase

793. The maximum volume of air that can be expelled out forcefully after a deep (maximal) inspiration called:

Inspiratory capacity

Vital capacity

Functional residual capacity

Total lung capacity

794. In what situation would the gas exchange ratio be decreased compared to the respiratory quotient?

During slowed breathing.

Holding your breath.

During hyperventilation.

Impossible. The two are always equivalent.

795. Which of the following does NOT apply to the alveoli at the base of the lungs?

They are less elastic than the alveoli at the apex.

The pleural pressure is lower.

At FRC they are less inflated than the alveoli at the apex.

They are closed at RV.

They have a greater volume change than alveoli at the apex during inspiration from FRC.

796. Which of the following could NOT be part of an acinus?


alveolar sacs

alveolar ducts

terminal bronchioles

respiratory bronchiole

797. Which of the following is NOT true at FRC?

It is about 75% TLC.

The elastic recoil of the chest wall is outward.

The elastic recoil of the lung is inward.

The relaxation pressure of the lung and chest wall combined is at atmospheric pressure.

798. Which of the following is INCORRECT concerning the efficiency of breathing and the oxygen consumption of the respiratory
muscles?

Efficiency is defined as the ratio of mechanical work done to move air to the amount of metabolic energy used by the
respiratory muscles.

The respiratory system uses less than 3% of the body's total oxygen consumption at rest

Respiratory muscles are more efficient than large muscle groups.

Emphysema increases the oxygen requirement for respiratory muscles.

Hyperventilation can increase the oxygen consumption of respiratory muscles to 30%.

799. Which of the following is NOT a function of dead space?

Warms expired air to body temperature.

Saturates inspired air with water vapor.

Removes bacteria and other particulate matter.

Conducts the warmed air to the respiratory membranes.

800. If a patient's blood carries 10 grams of Hb per deciliter, what is the O2 carrying capacity of his blood?

18 milliliters per deciliter.

18 milliliters per deciliter.

10 milliliters per deciliter.

13 millliliters per deciliter.

801. Which of the following is FALSE concerning the production and role of lung surfactant?

It is part of a lipoprotein called dipalmitoyl phosphatidyl-choline.

It is synthesized by alveolar type II cells.


y y yp

As the alveolar surface area decreases during the compression curve, the surfactant decreases the surface tension at a
constant rate.

When surfactant density is decreased during expansion, surface tension initially rises rapidly, then slows down until it reaches
the starting point.

802. Which of the following controls the normal breathing process?

Amino acids

Cholesterol

Ventral respiratory group

Dorsal respiratory group

803. All of the following are static lung volumes, except:

Tidal volume

Inspiratory reserve volume

Expiratory reserve volume

Residual volume.

Dynamic Volume

804. Which of the following is FALSE concerning the effect of effort on airflow and volume during inspiration and expiration?

During inspiration, greater effort always results in greater flow.

Peak expiratory flow occurs at the beginning of expiration.

At low and moderate lung volumes, the greater the effort above threshold, the greater the airflow in expiration.

Portions of the expiration curve are effort independent.

805. Which of the following increases renal cortical blood flow?

Prostaglandin E2

Norepinephrine

Angiotensin II

Vasopressin

806. The pressure in the ventricles reaches a maximum value during:

isometric contraction;

rapid ejection;

reduced ejection;
protodiastoles.

807. Which of the following zones in the adrenal cortex does not normally express 17 alpha hydroxylase?

Zona glomerulosa

Zona fasciculata

Zona reticularis

none of the above

808. Aldosterone antagonist diuretics work primarily on the:

cortical collecting duct

distal convoluted tubule

medullary collecting duct

proximal convoluted tubule

809. The systemic circulation begins in:

pulmonary trunk

the aorta

vena cava

pulmonary veins.

810. Hereditary spherocytosis occurs due to mutations in genes coding for:

Spectrin and ankyrin

Na-K ATPase

Glucose 6 phosphate dehydrogenase

Pyruvate kinase

811. The heart continues to beat even after all nerves to it are sectioned. This property is called:

Excitability

Conductivity

Automaticity

Contractility

812. What is NOT a function of blood

filtration
protection

regulation

transportation

813. Indicate which of the listed humoral factors has a positive effect on the myocardium:

acetylcholine;

bradykinin;

metabolites;

adrenaline.

814. When action potentials are generated in series faster than the relaxation period of a muscle, _______ results.

eccentric contraction

latency

summation

twitch

815. Which tissues do not require insulin for glucose uptake? (check all correct answers)

Red blood cells

Most parts of the CNS

Adipose tissue

a and b answers are correct

816. Which of the following statements regarding temperature receptors is FALSE?

There are more cold receptors in the skin than warm receptors.

Temperature sensory organs are naked sensory nerve endings responding to temperature changes

Temperatures sensory organs senses subcutaneous temperature, not skin surface temperature

Cold receptors respond to temperature range IO-38°C & warm receptors respond to temperature range 30-4SoC

817. When is salivary amylase most active?

Churning in Stomach

Lumen in Duodenum

Mastication in Mouth

Storage in Stomach
818. The highest degree of automatism in:

sinoatrial node;

atrioventricular node;

the bundle of his.

819. ESR is increased in:

Anemia

Hypofibrinogenemia

Spherocytosis

Polycythemia

820. Short-term memory information processing usually causes changes in the

brainstem.

medulla.

hypothalamus.

hippocampus.

821. In the capillaries the nature of blood flow:

laminar;

turbulent;

chained;

intermittent.

822. A Na⁺K⁺ Pump will transport ____ Na⁺molecules____ the cell & ___ K⁺___ the cell.

2, into, 3, out of

2, out of, 3, into

3, into, 2, out of

3, out of, 2, into

823. The “All or None” law in the heart shows:

the dependence of the force of contraction on the force of irritation;

independence of the force of contraction from the force of irritation;

independence of the strength of contractions from the frequency of irritations;


the relationship between the force of contraction and the length of the muscle fibers.

824. The amount of hemoglobin contained in normocytes is approximately:

20 pg

25 pg

30 pg

35 pg

825. The extrinsic pathway is inhibited by:

Tissue factor

Thromboplastin

Tissue factor pathway inhibitor (TFPI)

Contact factor

826. The term ‘neurohormone’ is applied to:

Oxytocin and vasopressin

NO and CO

Glycine and glutamate

FSH and LH

827. Which of the following is matched incorrectly?

Microfilaments: actin, myosin

Intermediate filaments: vimentin, keratin

Microtubules: clathrin

Cytoskeleton: spectrin, ankyrin

828. The blood-testis barrier is formed by tight junctions between:

Leydig cells

Sertoli cells

Spermatids

Primary spermatogonia

829. Plasma potassium concentration is chiefly regulated by:

aldosterone
aldosterone

vasopressin (ADH)

renin

ANP

830. Which of the following hormones has intrinsic lactogenic activity?

TSH

MSH

GH

Dopamine

831. The process of transition of water and substances dissolved in it from the tubules of the nephron into the blood is called:

synthesis;

secretion;

reabsorption;

filtering.

832. Blood pressure in the bloodstream is the least in:

arterioles;

venules;

capillaries;

large vein.

833. Cortical representation of somatic sensation lies in:

Central fissure

Pre-central gyrus

Post-central gyrus

Posterior parietal cortex

834. Heart decompensation is a violation of:

excitability;

automatism;

conductivity;

contractility.
835. The ‘gold standard’ for estimation of glomerular filtration rate is the clearance of

inulin

creatinine

urea

mannitol

glucose

836. A phonocardiogram is:

record of body vibrations during the work of the heart;

recording the total bio currents of the heart in the plane of the lead;

recording the total bio currents of the heart on the lead line;

recording sound manifestations of the heart.

837. The major humoral mediator of gall bladder contraction in response fo a fat meal is:

CCK

gastrin

secretin

somatostatin

838. The term “capacitance vessels” is applied to:

pulmonary capillaries

thoroughfare channels

shunts

veins and venules

839. The functions of the pleural cavity does not apply:

reduction of friction during movements of the lungs;

participation in the biomechanics of respiration;

protective, restrictive;

excretion of volatile substances.

840. Enterocytes are replenished by miotically active undifferentiated cells located in:

Brunner’s glands
u e s g a ds

Crypts of Lieberkuhn

Peyer’s patches

gut associated lymphoid tissue

841. Which of the following substances can be used as a marker for the ECF compartment?

Nonmetabolizable sugars

Glucose

Radio-iodinated albumin

D2O

842. Insulin dependent glucose uptake into skeletal muscle and adipose tissue is mainly mediated by:

GLUT 1

GLUT 2

GLUT 3

GLUT 4

843. The most abundant protein in blood is:

Albumin

Hemoglobin

Fibrinogen

Beta-1 globulin

844. Inhibition of which of the following enzymes has the greatest effect on digaestion?

Chymotrypsin

Enterokinase

Pancreatic amylase

Ptyalin

845. Which one of the following is not a vasodilator metabolite?

Adenosine

Potassium ions

Endothelin-1

Hydrogen ions
ADP

846. Normally, instillation of acid into the duodenum to reduce pH in its lumen to 4:

stimulates gastrin release

increases output of an enzyme rich pancreatic juice

increases activity of Brunner’s glands

relaxes the pyrolic sphincter

847. Water handling by kidney (% reabsorption)

93%

94%

99%

99.4%

99.9%

848. Which is not present in high concentrations at the active zone of synapses?

Calcium channels

Catecholamine vesicles

Neuropeptide vesicles

Synaptobrevin

849. Osmotic blood pressure is power:

The interaction of shaped elements with each other;

Thee interaction of blood cells with the wall of blood vessels;

Ensuring the movement of water molecules through a semipermeable membrane;

Providing the movement of blood.

850. The principal regulator of plasma osmolality is:

plasma [Na]

antidiuretic hormone

aldosterone

angiotensin II
851. Which of the following hormones does not act through G-protein coupled receptors?

Dopamine

Epinephrine

Angiotensin II

Retinoic acid

852. Insulin secretion from beta cells of pancreas in response to a glucose load is mediated by:

GLUT 1

GLUT 2

GLUT 3

GLUT 4

853. Which of the following type of motor activity in the small intestine is stimulated by local irritation of the intestinal mucosa?

Haustration

Peristaltic rush

Secondary peristalsis

Segmentation movement

854. Glomerulotubular balance

Involves afferent arteriole feedback loop - No

Involves efferent arteriole feedback loop - No

Juxtaglomerular complex - No

Ability to increase tubular absorption in response to an increase in filtered load

855. Which of the following characterizes carbohydrate digestion?

It begins when food comes in contact with gastric juice

It begins when the food comes in contact with saliva

It begins with when food comes in contact with pancreatic secretions

It ends when starch has been converted to maltose

856. Which of the following is an example of passive transport?

Calcium efflux by calcium pump


Na-Ca exchanger

Potassium efflux through potassium leak channels

Calcium sequestration in sarcoplasmic reticulum

857. The major humoral mediator of meal-stimulated enzyme secretion is:

secretin

CCK

GIP

gastrin

858. A sarcomere is the area between:

A bands

H bands

I bands

Z disks

859. Complex receptors have:

A single nerve ending

free nerve endings

many nerve endings

nerve endings in connective tissue

860. The microcirculation system includes:

arterioles, capillaries, venules, arteriolo-venular fistulaes;

aorta, arteries, arterioles;

anastomoses, venules and veins;

arteries and veins.

861. Respiratory arrest occurs when:

damage to the medulla oblongata;

transection of the brain stem in the region of the diencephalon;

transection of the brain stem above the pons;

transection of the brain stem below the pons.


862. The permeability of glomerular capillaries:

Equals that of other capillaries

Is much less than that of other capillaries

Is equal for cationic and anionic molecules of equal size

Approaches 100% for neutral molecules of 8mm diameter

Is about 50 times as great as that of a skeletal muscle capillary

863. Intrinsic heart rate can be determined by:

vagotomy

administration of atropine

beta adrenergic receptor blockade

IV administration of atropine and atenolol

864. The action of salivary amylase is continued when it enters the stomach

FALSE

Lingual lipase

Proline rich proteins

TRUE

865. In the nephron, glucose reabsorption occurs mainly in the:

proximal tubule

loop of Henle

distal convoluted tubule

collecting duct

866. What fraction of total blood volume is present in the capillaries at any given time?

5%

20%

15%

1%

867. In clinically significiant diarrhea, typically, the most abundant anion in stool is:

acetate
bicarbonate

butyrate

chloride

868. The process of transition of water and the capillaries dissolved in it from the blood into the Bowman-Shumlyansky nephron
capsule is called:

synthesis;

secretion;

reabsorption;

filtering.

869. Postganglionic parasympathetic neurons innervating the gastrointestinal smooth muscle are located in:

myentric plexus

submucosal plexus

paravertebral ganglia

prevertebral ganglia

870. Most of the circulating T4 is bound to:

Prealbumin

Albumin

Thyroxine-binding globulin

871. Disturbances of excitability in the heart manifest themselves:

blockade;

extrasystole;

decompensation;

tachycardia.

872. How many subunits do G proteins have?

One

Two

Three

Four
873. The long phase of absolute refractoriness ensures:

rhythmic contractions of the heart;

consecutive atrial and ventricular contractions;

impossibility of tetanic contractions;

synchronous contraction of myocardial fibers.

874. Name the ventricular function:

the expulsion of blood in the blood circulation;

suction of blood to the ventricles;

hydrodynamic shock for the atria;

unilateral movement of blood through the heart.

875. The amount of blood pressure depends on:

strength of heart rate;

peripheral resistance;

blood viscosity;

all of the above.

876. Growth hormone stimulates the secretion of:

Somatostatin

Lactotroph

Corticotroph

Thyrotroph

877. If dead space is one third of the tidal volume and arterial pCO2 is 45 mmHg, what is the mixed expired pCO2?

20 mmHg

25 mmHg

30 mmHg

45 mmHg

60 mmHg

878. What fraction of total body potassium is present in plasma?


0.4%

8.2%

12%

88%

879. Which of the following is shown by arrow in figure below?

SA node

AV node

Bundle of His

Purkinje fibers

880. Glucocorticoids decrease the number of circulating:

Eosinophils

RBC

Platelets

none of the above

881. An insufficient supply of oxygen to tissues is called:

hypoxia;

hypoxemia

hyperbaria;

caisson disease.

882. Quantitively, the most important enzyme in digestion of fat is:

lingual lipase

gastric lipase

pancreatic lipase

lipoprotein lipase

883. Factors affecting the metabolism in the capillaries include everything, except:

hydrostatic pressure;

oncotic pressure;

vascular permeability;

atmospheric pressure.
at osp e c p essu e.

884. A diabetic woman who is pregnant and who has been taking recombinant insulin during the pregnancy delivered a baby
weighing 5 kg. This is most closely related to:

Passage of maternal insulin into the fetus

Elevated levels of fetal insulin

Uteroplacental insufficiency

Hyperglycemia in the mother

885. Gas exchange occurs in:

large bronchi;

the alveoli;

medium bronchi;

trachea.

886. Which of the following is the most sensitive test of pancreatic B-cell insulin secretory reserve?

Oral glucose tolerance test

Fasting plasma glucose

Urine glucose excretion

Random blood glucose

887. The method of registration of arterial pulses is called:

manometry;

phlebography;

sphygmography;

spirography.

888. The role of the pericardium:

softens friction;

forms valves;

generates pulses;

provides a reduction of chambers in the heart.

889. Juxtamedullary nephrons constitute about ______ % of the total nephrons.

10
15

25

40

890. In the management of a child with diabetic ketoacidosis and a plasma pH of 6.9, insulin infusion is typically continued even
after plasma glucose is reduced to 250 mg/dL, and glucose is coadministered with insulin. This is done in order

allow:

Muscle glycogen to be replenished

Plasma potassium to be normalized

Plasma osmolality to be reduced rapidly

Complete oxidation of ketone bodies

891. An action potential can be generated when...

The value of the potential is above threshold

The value of the potential is below threshold

The value of the potential is equal to zero

All of the value

892. With acute acclimitisation to altitude:

Hypoventilation

Decreased cardiac output

Pulmonary oedema

Polycythaemia

Increase in 2,3 DPG

893. Which of the following hormone is normally released by the stomach into the systemic circulation?

Vasoactive intestinal polypeptide

Gastrin

Pepsinogen

Secretin

894. Normally, most of the water in the GI lumen is absorbed from:

stomach

duodenum
jejunum

colon

895. Which of the following cells undergo meiotic division?

Primordial germ cells

Primary spermatocytes

Secondary spermatocytes

Spermatids

896. Which of the followings is an excitotoxin?

glutamate

glycine

acetylcholine

substance p

897. Normally, the rate-limited step in the metabolism of bilirubin is:

uptake by ligandin

conjugation with glucuronic acid

secretion into the bile

none of the above is correct

898. Respiratory control chemoreceptors are sensitive to:

hypocapnia, alkalosis;

hypercapnia, acidosis, hypoxemia;

hyperoxia, alkalosis;

somatostatin, ADH.

899. What percentage of osmolality of plasma in a healthy, well hydrated individual is attributable to sodium and its
accompanying anions?

30 %

50 %

70 %

90 %
900. A decrease in extracellular K+:

Cause a similar effect in nerve cells as an increase in extracellular Ca+

Causes a similar effect in nerve cells as a decrease in extracellular Na

Has little effect in nerve cell membrane potential

May decrease nerve cell action potential size

901. The substance secreted from D cells is responsible for:

HCl activation.

protection against HCl.

stomach contraction.

stomach relaxation.

902. Peripheral chemoreceptors:

In the carotid sinus

Have glomus cells

Low A-V difference

Innervated by glossopharyngeal nerve RE13

903. Deep, laboured gasping breathing seen in severe acidosis is known as:

apneustic breathing

Biot's respiration

Cheyne-stokes respiration

Kussmaul breathing

904. The second heart tone occurs during:

asynchronous reduction;

isometric contraction;

isometric relaxation;

protodiastoles.

905. The linear velocity of blood flow in the aorta is:

5 m / s;

0.5 m / s.
0,5mm / s .;

0.3 m / s.

906. Which ion is both secreted and absorbed by the nephron?

Na

Cl

Ca

907. In terms of the action potential generation within a neuron, which of the following structures is not involved in the
generation and propagation of an action potential?

Rough endoplasmic reticulum

Dendrites

Axon

Cytoplasm

908. Passive exhalation occurs due to:

contraction of the external intercostal muscles and diaphragm;

relaxation of the external intercostal muscles and diaphragm;

contraction of the abdominal muscles;

contraction of the internal intercostal muscles.

909. The action potential of a neuron (influx):

Declines in amplitude as it moves along the axon

Is initiated by efflux of Na

Is not associated with any net movement of Na+ & K+ across the cell membrane

Is terminated by efflux of K

910. The duration of the cardiac cycle depends on:

blood flow to the heart;

the excitability of a typical myocardium;

heart rate;

911. In whichsensory system does excitation of receptor by an adequate stimulus result in hyperpolarization of receptor cells?

visual pathway
visual pathway

auditory pathway

taste pathway

olfactory pathway

912. Sodium-glucose cotransport in the intestine and kidney is an example of:

Primary active transport

Secondary active transport

Facilitated diffusion

Passive transport

913. Bile salts are essential for absorption of fat because they:

solubilize dietary lipids in micelles

increase surface tension

are hydrophobic

contain cholesterol

914. Hypoxic hypoxia occurs when:

reducing the oxygen capacity of the blood;

a decrease in pO2 in atmospheric air;

changes in the functions of respiratory tissue enzymes;

reducing the number of red blood cells.

915. With regard to development of gonads, which of the following is most likely with a 46XX zygote that contains a functional
SRY gene?

Female internal and external genitalia develop

Both ovaries and testes develop

Only ovaries develop

Only testes develop

916. Which of the following plasma proteins are protease inhibitors?

α1 antitrypsin, Antithrombin III

Transferrin

C-reactive protein

f
all of the above

917. The minimum time for transmission across one synapse is:

0.5 ms

1 ms

1.5 ms

2 ms

918. Which one of the following inhibits ADH secretion?

Hyperosmolality

Exercise

Angiotensin II

Ethanol

919. In the kidneys are secreted:

glucose and amino acids;

vitamins and water;

proteins and salts;

potassium ions and medicinal substances.

920. The most potent stimulus for release of secretin from the duodenum is:

a reduction in duodenal lumen pH to <4.5

peptides

fatty acids with > 8 carbons

carbohydrates

921. Atrial systole lasts:

0.1 s;

0.9 s .;

1 s;

0.2 s.

922. Regarding glucose handling in the kidney

Reuptake is passive
Tm is the same for all nephrons

D-glucose more rapidly absorbed than L-glucose

Reabsorption is inversely proportional to lipid solubility

923. The active blood reaction (pH) is normally equal to:

8.0;

7.34;

1.02;

5.0.

924. The hormone of energy storage (hormone of plenty) is:

Growth hormone

Thyroxine

Insulin

Glucagon

925. Veins are vessels that:

carry blood from the heart;

carry blood to the heart;

carry blood from the ventricles to the arterioles;

carry blood from the capillaries to the atria.

926. Isotonic solution is a solution containing:

A certain number of red blood cells;

The amount of salts corresponding to the blood plasma;

The amount of nutrients corresponding to the blood plasma;

A certain amount of plasma proteins.

927. In a healthy adult male weighing 70 kg, the total volume of fluid present in the transcellular compartment does not
normally exceed:

1 liter

3 liters

5 liters

7 liters
928. Which subunit of G proteins has intrinsic GTPase activity?

Alpha

Beta

Gamma

Delta

929. Calcitonin inhibits the activity of:

Osteoblasts

Osteoclasts

Parafollicular cells

Thyroid

930. Midbrain responses include the following EXCEPT:

Vestibulo-ocular reflex (righting reflex)

Pupillary reaction

Mass reflex

Vestibular placing reaction

931. The submandibular gland secretes

Lingual lipase

Lysoszyme

Proline rich proteins

Salivary aylase

932. In the tubules of the nephrons are completely reabsorbed:

amino acids and glucose;

urea and creatinine;

high molecular substances.

933. Regarding monosynaptic reflexes:

Intrafusal fibres are at right angles to extrafusal fibres

Spindles themselves have no motor supply

The 2 types of sensory endings in spindles are Ib & II


The neurotransmitter at the central synapse is glutamate

934. What is the maximum of volume that can be inspired at rest after a normal inspiration?

expiratory reserve volume

functional residual capacity

inspiratory reserve volume

residual volume

935. Blood functions do not include:

Trophic;

Protective;

Hormone synthesis;

Respiratory.

936. Which ONE of the following is not involved in the regulation of glomerular filtration rate (GFR)?

Juxtaglomerular apparatus

Arterial pressure

Efferent arteriolar tone

Na content in distal tubule

Afferent arteriolar tone

937. Select all correct answers. Uptake of potassium into cells is enhanced by which of the following hormones / mechanisms?

Thyroid hormones

Insulin

Beta-adrenergic receptor activation

all of the above

938. A blood pH of 7.25 would indicate:

acidosis

alkalosis

normal pH

939. Specify a substance of renal origin, which increases blood pressure:

ammonia;
ammonia;

urokinase;

renin;

urea.

940. In the loops of Henle nephrons, the concentration and dilution of urine is achieved by:

reabsorption of sodium in the descending knee and water in the ascending knee;

reabsorption of glucose and amino acids;

reabsorption of water in the descending and sodium in the ascending part of the loop;

the secretion of water and urea in both parts of the loop.

941. Regarding synapses:

IPSP can be produced by closing K channels

Large vesicles contain ACh

Na+ influx triggers fusion/exocytosis of vesicles

Neurotransmitters migrate down the axon by fast axoplasmic transport

942. Two liters of 0.9% NaCl is administered to a 12-year old boy with moderate isotonic dehydration. What is the expected
change in ICF volume after NaCl administration?

No change

Increase by 0.5 liter

Increase by 2 liters

Decrease by 0.5 liter

943. Which of the following is incorrect with regard to emotion?

Cingulate cortex is associated with volitional behaviour and decision making

Hypothalamus is associated with primitive, short-lived emotion that is terminated by medial hypothalamic nuclei

Nucleus accumbens is associated with reward and repeated behaviour in increased dopamine release

Septum dampens socialising tendencies and emotions

944. Which is the ‘concentrating segment’ of the nephron?

Descending limb of loop of Henle

Thin ascending limb of loop of Henle

Thick ascending limb of loop of Henle


Collecting ducts

945. Which of the following is / are active transport processes? (Select all correct answers).

Extrusion of calcium into ECF; Transcytosis (vesicular transport)

Efflux of K during repolarization

Entry of chloride into neurons

all of the above

946. Which of the following substances should not be used to measure GFR?

Inulin

Creatinine

Phenol red

947. Osmotic fragility of red blood cells is decreasedin:

Sickle cell anemia and microcytic hypochromic anemia

Hereditary spherocytosis

Macrocytic anemia

all of the above

948. Vessels are narrowed under the action of:

acetylcholine;

adrenaline;

metabolites;

bradykinin.

949. Which of the following inhibits gastric acid secretion by patietal cells in stomach?

Prostaglandin E2

Gastrin

Acetylcholine

histamine

950. The lipolytic effect of catechol amines is mediated by:

ß1 adrenergic receptors

ß2 adrenergic receptors
ß3 adrenergic receptors

ß1 & ß3 adrenergic receptors

951. Veins and venules are related to:

buffer or tensile vessels;

exchange vessels;

resistance vessels;

capacitive vessels.

952. Gas exchange between alveoli and blood is carried out in:

arteries of the pulmonary circulation;

veins of the pulmonary circulation;

the capillaries of the great circle of blood circulation;

the capillaries of the pulmonary circulation.

953. Testicular feminization syndrome is caused by mutations in genes coding for:

Androgen-binding proteins

Testosterone 5α reductase

Androgen receptor

Inhibin

954. Alveolar dead space is increased with:

Pleural effusion

CCF

Pneumothorax

Hypotension

None of the above

955. Which of the following mechanisms allows complete absorption of glucose from the intestinal lumen into the cytosol of the
enterocyte?

simple diffusion through GLUT-5

Na-glucose cotransporter-1

na channels in enterocyte membrane

paracellular uptake via gap junctions


956. Which part of heart conductive system is this?

SA node

AV node

Bundle of His

Purkinje fibers

957. The effect of PTH on the kidney is to:

Increase Ca excretion and increase phosphate excretion

Increase Ca excretion and decrease phosphate excretion

Decrease Ca excretion and increase phosphate excretion

Decrease Ca excretion and decrease phosphate excretion

None of the above

958. The T wave on an ECG describes:

atrial depolarization;

atrioventricular delay;

depolarization of the interventricular septum;

ventricular repolarization.

959. The pulmonary circulation ends:

pulmonary veins

the aorta

pulmonary trunk

vena cava

960. The most abundant cell type in the islets of Langerhans is:

A cells

B cells

D cells

F cells

961. Which of the following has the most powerful stimulatory effect on salivary secretion under normal conditions?

Gritty material
y

Sour taste stimulus

Sympathetic stimulation

Trigeminal stimulation

962. What percentage of HCO3 is reabsorbed in the kidney?

0.01

50

75

99.9

963. Emiocytosis requires an increase in the intracellular concentration of:

Na

Ca

Cl

964. Insulin sensitivity is not reduced in:

Glucagon-secreting tumors

Tumors of adrenal medulla

Tumors of adrenal cortex

Hypopituitarism

965. The compensatory pause is:

conduction disturbance;

decrease in heart rate;

the loss of one of the next reduction;

change in the force of contractions.

966. Which of the following statements regarding regulation of cell cycle is incorrect?

This serves to regulate transition of the cell from one phase of the cell cycle to the next.

At restriction points, cyclins always promote transition from G0 to S phase; Defects in DNA synthesis normally allow
progression from G2 to M phase.

Deficiency of p53 allows progression to M phase.

ll f th b
all of the above

967. Normal breath begins with a reduction:

internal intercostal muscles and muscles of the shoulder girdle;

muscles of the chest and back;

external intercostal muscles and diaphragm;

neck muscles and internal intercostal muscles.

968. Which of the following CNS locations would not contain myelinated axons?

Reticular formation

Internal Capsule

Corpus callosum

Arcuate fasciculus

969. The amount of mineral substances in the blood plasma is:

8 - 10%;

6 - 8%;

0.8 - 1%;

0.1 - 0.3%.

970. Which of the following clotting factors is not vitamin K dependent?

Factor II

Factor V

Factor VII

Factor IX

971. What is the outermost layer of the general GIT lining?

Circular Smooth Muscle

Longitudinal Smooth Muscle

Muscularis Mucosa

Submucosa

972. In humans, the most abundant adrenocortical hormone is:

Cortisol
Corticosterone

Deoxy corticosterone

DHEA sulfate

973. Which of the following statements about cholinergic transmission is false?

ACh is re-taken up into the presynaptic knob unchanged

Action is halted by hydrolysis of Ach

Atropine blocks nicotinic receptors

Muscarinic receptors are mainly found in the peripheral nervous system

974. Glucose and amino acids are reabsorbed from filtrate in the:

collecting duct

loop of Henle

proximal tubule

distal tubule

975. Which of the following neurotransmitters has both excitatory and inhibitory effects?

glycine

GABA

aspartate

glutamate

976. Capacitive vessels perform the following function:

regulate blood flow in the capillaries

cause blood pressure

smooth the pulsation of blood flow

deposit blood.

977. Which is not a vasoconstrictor?

Angiotensin II

Endothelin

Nitric Oxide

None of them
978. Growth hormone secretion is increased by all of the following except:

Pharmacologic doses of hydrocortisone

Hypoglycemia

Exercise

Protein meal

979. The release of androgens from the adrenal cortex is stimulated mainly by:

LH

FSH

ACTH

GnRH

980. Clinically, in an unselected population of individuals, the most sensitive test to detect a disorder of thyroid gland function is
measurement of:

TSH

Free T3

Free T4

Total T4

981. Choose the correct statement about Cerebrospinal fluid (CSF):

Volume is 50 ml

Produced by choroidal blood vessels and ependymal cells

Production 150 ml a day

None of the above is correct

982. The two drugs that are commonly used to induce diabetes in rats are:

Streptozotocin

Alloxan

a and b

Capsaicin

983. The stomach wall is made up of 2 mucous cells [surface + neck], parietal cells, chief cells, and G cells. B] Which one drives
stomach action via secretion and motility?

Chief
G

Mucous

Parietal

984. Kidney reabsorption is:

the process of transition of water and substances dissolved in it from the blood into the Bowman-Shumlyansky capsule;

the process of transition of water and substances dissolved in it from the blood into the renal tubules;

the process of transition of water and substances dissolved in it from the blood into the loop;

the process of transition of water and substances dissolved in it from the tubules into the blood.

985. Which of the following is not a characteristic of REM sleep?

slow waves

hypotonia

dreaming

PGO spikes

986. The maximum pressure rise in the ventricle occurs during:

ejection

isovolumetric contraction

protodiastole

diastole

987. Which is the most important cholagogue?

secretin

CCK

gastrin

GIP

988. K+ exits neuron via voltage gated K+ channels during:

depolarization

repolarisation

rest

none of the above


989. Plasma proteins do not include:

Albumin;

Gobulins;

Fibrinogen;

Hemoglobin.

990. At any time, the greatest fraction of blood volume is present in the:

heart

arteries

veins

capillaries

991. The main cardiovascular (vital) center is located at:

the spinal cord;

the midbrain;

the medulla;

cerebral cortex.

992. In the anterior lobe of the human pituitary, prepro-opiomelanocortin is processed mainly to:

Alpha-MSH

ACTH

Beta-endorphin

Beta-MSH

993. Indicate myocardial function:

softens friction when the heart is working;

protects shaped elements from destruction;

forms flap valves;

provides the pumping function of the heart.

994. Most excitatory neurotransmission in the brain is believed to be mediated by:

glutamate

glycine
GABA

GnRH

995. Internal resistance in the bloodstream depends on:

blood viscosity;

oncotic blood pressure;

blood pH;

electrolyte content.

996. The juxtaglomerular apparatus of the kidney secretes into the blood:

aldosterone;

sodium uretic factor;

angiotensin;

renin.

997. Vasoconstriction, not true...

Afferently decreases filtration pressure

Efferently always increases blood pressure of glomerulus

Moderate constriction efferently increases blood flow

None of them

998. In which of the following food substances is chewing essential for digestion?

Cheese

Eggs

Fruits and vegetables

Meat

999. A first-order pacemaker is:

bundle of His;

Purkinje fibers;

sinoatrial node;

atrioventricular node.

1000. Which of the following is not secreted by parietal cells?


Gastroferritin

HCl

Intrinsic factor

Pepsinogen

1001. Complete fusion of labioscrotal swellings does not occur despite exposure to high concentrations of androgens beyond
which point in intrauterine life?

3 weeks

6 weeks

9 weeks

12 weeks

1002. What is a resting membrane potential (RMP)?

A combination of two equilibrium potentials guided by the permeability of those two ions

Energy, expressed in millivolts, of a concentration gradient of an ion

Voltage difference across the cell membrane when the cell is at rest

Something that influences the generation of an action potential

1003. Which of the following is not a function of the lung?

All are functions of the lung

Condensation of water vapor from expired air

Cool down air

Metabolism of substances

1004. Which of the following strategies would most likely be therapeutic in an individual with achalasia?

injection of tetanus toxin into the lower esophageal sphincter

surgical division of the lower esophageal sphincter

administration of a muscarinic receptor agonist

administration of a nitric oxide synthase inhibitor

1005. What fraction of filtered water is reabsorbed in the loop of Henle?

15%

25%

35%
5%

1006. The types of contraction that normally occur only in colon is:

peristaltis

antiperistaltis

segmentation

all of the above

1007. Which of the following retinal neurons generate "all or none" action potentials?

Ganglion cell

Bipolar cell

Horizontal cell

Amacrine cell

1008. Pain reflex is accompanied by:

weakening of the heart;

a decrease in heart rate;

strengthening the work of the heart;

the absence of changes in the activity of the heart.

1009. Which of the following normally stimulates the release of digestive enzymes from the ancinar cells of the pancreas?

Cholecystokinin

Gastrin

Histamine

Secretin

1010. Select all correct answers. Somatostatin inhibits the secretion of:

Insulin

Glucagon

Growth hormone

all of the above

1011. Arterial pulse is:

fluctuations in blood pressure;


oscillations of artery walls associated with the work of the heart;

periodic fluctuations in blood flow velocity in the arteries;

automatic monounitary cells in arteries.

1012. Hypoglycemia does not stimulate the secretion of:

Epinephrine

Cortisol

Growth hormone

Insulin

1013. Which of the following statements is TRUE?

Visceral pain tends to be poorly localized & is often referred to distal sites.

. Visceral pain can initiate reflex contraction of nearby skeletal muscle

Ischemic muscle tends to be persistently painful until reperfusion occurs, due to accumulation of metabolic products such as
potassium.

All of the above.

1014. Name the factor that does not affect the glomerular filtration rate:

the number of active nephrons;

the composition of blood cells;

the filtration surface of the glomerular capillaries;

blood volume passing through the kidneys per unit of time.

1015. Which layer lies immediately deep to the muscular mucosa?

Circular Smooth Muscle

Lamina Propria

Longitudinal Smooth Muscle

Submucosa

1016. Increased GFR caused by

Increased cardiac output

Afferent arteriolar vasoconstriction

Efferent arteriolar vasodilatation

Increased chloride delivery to the macula densa


Increased chloride delivery to the macula densa

1017. The surface of the airways is lined with:

ciliated epithelium;

endothelium;

alveolocytes;

stratified keratinized epithelium.

1018. Membrane potential:

Is mainly caused by leaking Na+/K+ channels

Is negative inside in relation to the outside

Is not contributed to by the Na/K pump

Is only found in nervous tissue

1019. Atrioventricular valves close in case:

pressure in the left ventricle is greater than in the right;

pressure in the atria is greater than the pressure in the ventricles;

pressure in the ventricles is greater than the pressure in the atria;

pressure in the atria and ventricles is the same.

1020. The main factors affecting the filtration rate in nephrons are:

hydrostatic pressure, oncotic pressure of blood, intrarenal pressure;

osmotic pressure, blood pH;

only intrarenal pressure and oncotic blood pressure;

partial pressure of gases.

1021. The reticular activating system:

Has depressed conduction during anaesthesia

Has no input from the cranial nerves

Is a simple collection of parallel nerve fibres

Is electrically isolated from the cerebral cortex

1022. Which of the following statements regarding lesions in the motor control pathway is incorrect?

Babinski sign is a localising sign to anterior corticospinal tract lesion


In a right-handed subject, left motor cortical lesion leads to both left & right-hand motor dysfunction

In a right-handed subject, right motor cortical lesion leads to left hand motor dysfunction

In small focal ischaemic lesions in the motor cortex, adjacent undamaged areas may learn to take over the responsibility of
the damaged areas, thus restoring motor function

1023. The first heart sound:

diastolic;

is not heard by a stethoscope;

deaf, long, low;

short, high, clear.

1024. In the mountains, the magnitude of the atmospheric pressure and the gas composition of the air change as follows:

atmospheric pressure and% gas content in the atmosphere decrease;

atmospheric pressure increases and% composition of gases does not change;

atmospheric pressure decreases and the% composition of gases does not change;

atmospheric pressure and% composition of gases increase.

1025. The size of the action potential is decreased by:

Decreasing intracellular Ca

Decreasing the external Na

Decreasing the internal K

Increasing external Na

1026. The stretch reflex:

Involves extrafusal fibres stimulating annulospiral & flowerspray sensory fibres

Is classically initiated by tapping on the quadriceps muscle

Involves impulses being conducted from the muscle spindle to the motor cortex

Is initiated by stretch of the muscle

1027. What happens during depolarization?

Membrane potential (Em) goes to equilibrium potential for potassium ions (Ek)

Membrane potential (Em) goes to equilibrium potential for sodium ions (ENa)

Potassium permeability increases

Sodium permeability increases


1028. Absorption is complete when a substance moves into the GI epithelial cells

Epithelial cells are polarized

FALSE

Hemidesmosomes (focal adhesions) attach intermediate filaments in the cell to the extacellular matrix

TRUE

1029. A substance injected intravenously was found to be distributed through 35% of total body water. Which of the following is
most likely?

It did not pass through blood capillaries.

It entered neurons and cerebral ventricles.

It did not enter cells.

It was excluded from ‘third spaces’.

1030. The major trophic hormone for Sertoli cells and the major stimulator of aromatase activity of Sertoli cells is:

Inhibin

LH

FSH

Melatonin

1031. What is the primary ionic basis of the prepotential in the SA node?

Ca influx through transient T Ca channels

Inwardly directed long-lasting Ca current

Outward Na current

Potassium efflux through leak channels

1032. Surfactant:

Causes hysteresis (Or: Is the ONLY cause of hysteresis) – No, any elastic substance shows hysteresis

Is produced by type 1 pneumocytes

Is commonly deficient in term neonates

Production is slow

Increases pulmonary compliance

1033. O2 and CO2 are transported by:

red blood cells;


leukocytes;

platelets;

plasma proteins.

1034. Which compartment does the term “sucrose space” refer to?

Extracellular fluid (ECF)

Interstitial fluid (ISF)

Intracellular fluid (ICF)

Plasma

1035. Which of the following conditions increases the risk of fasting hypoglycemia?

Adrenocortical insufficiency

Hyperglucagonemia

Severe hypothyroidism

Acromegaly

1036. Digestion of which of the following occurs almost entirely in the small intestine?

Fat

Fruits and vegetables

Protein

Starch

1037. Which of the following statements regarding the actions of gastrointestinal hormones is incorrect?

secretin inhibits gastric emptying

gastrin stimulates histamine release in the stomach

somatostatin reduces blood flow to the GIT

GIP inhibits the release of insulin from pancreas

1038. Which of the following is shown by arrow in figure below?

SA node

AV node

Bundle of His

Purkinje fibers
1039. The three glycoprotein hormones each containing 2 subunits (an alpha and a beta subunit) produced by the pituitary are:

FSH, LH and TRH

TRH, ACTH and LH

TSH, LH and FSH

GH, TSH and LH

1040. Which of the following ions is not transported across the cell membrane by a primary active transport mechanism?

Na

Cl

Ca

1041. Intrinsic factor is essential for absorption of what substance in the terminal ileum?

Bile Salts

Ca²⁺

Vitamin B12

Vitamin C

1042. Which of the following statements regarding pain sensation transmission is FALSE?

Fast or I st pain is transmitted by A8 fiber.

Pain perception alone do not require a functional cerebral cortex.

Pain impulses are modified at the dorsal horn by interneurons & collateral touch fibers.

Principal neurotransmitter of pain is histamine

1043. The fetal zone of the adrenal cortex in the predominantly secretes:

Progesterone

Androstenedione

DHEA

DHEA sulfate

1044. Factors (not) affecting renal blood flow/GFR:

Sympathetic nervous system

Sodium flow past macula densa


Afferent arteriolar vasodilatation

Arterial pressure

Efferent arteriolar vasoconstriction

1045. Which pattern of motility hastens the transit of chime in the small intestine in the digestive state?

mass action contraction

migrating motor complex

peristaltis

segmentation contraction

1046. Alkalosis is:

Shift the blood reaction in the acidic side;

Shift of blood reaction to the alkaline side;

Change in osmotic pressure;

Change of oncotic pressure.

1047. Urinary output is:

The amount of fluid entering bowman's capsule in ml/min

The amount of urine entering the bladder in ml/min

The amount of urine voided over a 24 hour period

The concentration of a solute in the urine per unit time

1048. The composition of the primary urine does not include:

urea;

red blood cells;

vitamins;

amino acids.

1049. Anatomical dead space in adults is:

7 - 8 ml;

18 to 20 ml;

30 - 40 ml;

140 to 150 ml;


1050. The major humoral regulator of the concentration of sodium in plasma is:

aldosterone

vasopressin (ADH)

renin

angiotensin II

1051. The enzyme that controls smooth muscle contraction is:

Ca2+ kinase

Ca2+ATPase

myosin light chain kinase

myosin light chain phosphatase

1052. CO is formed as an end product of:

Heme metabolism

Arginine metabolism

Oxidation of acetoacetate

none of the above

1053. Which part of heart conductive system is this

SA node

AV node

Bundle of His

Purkinje fibers

1054. Normally, there is a net secretion of which ion in the colon?

sodium

potassium

chloride

calcium

1055. Select all correct answers. Platelet aggregation is stimulated by:

Thromboxane A2 and serotonin

Fibrinogen
ADP

Thrombin

1056. Aorta and large arteries are:

vessels of elastic type;

vessels of muscular type;

muscle and elastic vessels;

without vessels.

1057. The process of transfer of substances from the blood capillaries into the nephron canaliculi is called:

synthesis;

secretion;

reabsorption;

filtering.

1058. Which of the followings statements regarding visual reception is INCORRECT?

Rods & cones are hidden beneath an array of neurons in the retina.

Impulses from rods & cones converge onto a network of neurons & ultimately to the optic nerve.

Pigment epithelium absorbs light rays to prevent reflection.

Optic disk is situated at the posterior pole of the globe.

1059. Iodine is concentrated in thyroid follicular epithelial cells by:

Primary active transport

Secondary active transport

Simple diffusion

Facilitated diffusion

1060. Renal blood flow:

Is 600-650ml/min per kidney

Is directly measured by infusing PAH

Is increased by sympathetic tone

1061. Which of the following layers is the most superficial?

Inner circular layer of muscularis externa


Muscularis mucosae

Outer longitudinal layer of muscularis externa

Submucosa

1062. Diurnal variation in eosinophil count is due to:

Cortisol

Insulin

Thyroid hormones

Thrombopoietin

1063. The volume of food in the stomach has which of the following effects on stomach emptying?

A the volume if food in the stomach increases, the rate of emptying decreases

A the volume of food in the stomach decreases, the rate of emptying decreases

A the volume of food in the stomach increases, the rate of emptying increases

The volume of food in the stomach has little if any effect of the rate if stomach emptying

1064. Select all correct answers. Red blood cell antigens A and B are also present in:

Saliva

Semen

Amniotic fluid

All correct

1065. Select uncorrect answers. Which condition(s) is / are characterized by an increase in both bleeding time and clotting
time?

Afibrinogenemia

Hypoprothrombinemia

Hemophilia A

Von Willebrand’s disease

1066. Which of the following does not break down proteins?

Carboxypeptidase

Chymotrypsin

Pepsin

None of them
1067. The normal arterio-venous difference for CO2 is:

2ml/100ml

4ml/100ml

6ml/100ml

10ml/100ml

1068. Which of the following is absorbed in the intestine independent of sodium?

glucose

amino acids

galactose

fructose

1069. The P-wave on an ECG describes:

depolarization of the ventricles;

atrial depolarization;

ventricular repolarization;

atrial repolarization.

1070. Which of the following describes how bicarbonate ions are absorbed from the small intestine?

Directly via facilitated diffusion

Directly via simple diffusion

Indirectly in the form of carbon dioxide and water

Indirectly via active transport

1071. The composition of the histohematic barrier includes:

Only the nucleus of the cell;

Cell mitochondria only;

The membrane of mitochondria and inclusions;

Cell membrane and vascular wall.

1072. Which of the following processes does not exhibit 'saturation kinetics'?

Facilitated diffusion

Na+-Ca2+ exchanger
Simple diffusion

Na+coupled active transport

1073. Which of the following solutions is hypertonic?

0.9 % NaCl

5% dextrose

20% mannitol

Distilled water

1074. Hormone aldosterone is released into the blood when:

reducing the concentration of sodium in the plasma;

an increase in sodium concentration in plasma;

increasing the concentration of H ions in the plasma and lowering the pH;

the accumulation of lactic acid and the development of acidosis.

1075. The most potent vasoconstrictor is:

endothelin 1

angiotensin II

norepinephrine

vasopressin

1076. Most of the iron in the body is present in:

Hemoglobin

Myoglobin

Ferritin

Transferrin

1077. Indicate violation of myocardial contractility:

tachycardia;

sinus arrhythmia;

decompensation;

extrasystole.
1078. The protein that binds extracorpuscular hemoglobin is:

Hemin

Haptoglobin

Hemopexin

Haptopexin

1079. Plasma proteins do not perform the function:

Protective;

Trophic;

Gas transport;

Plastic.

1080. Ions are pumped out of the _____ limb of the loop of Henle

ascending

descending

both of the above

none of the above

1081. Inhibitory postsynaptic potentials (IPSP) may be produced by:

closure of na channels

closure of Ca channels

opening of k channels

all of the above

1082. Which of the following best characterizes the secretions of the large intestine?

Consist mostly of trapped bacteria from the crypts of Lieberkuhn

They are mostly mucus

They contain enzymes for the final digestion of food

They contain large quantities of bicarbonate ions

1083. The first reflex response to an increase in arterial pressure produced by intravenous injection of phenylephrine is:

increase in RR interval

increase in heart rate


decrease in cardiac output

1084. Which of the following types of cells work as scavenger cells in CNS?

microglia

oligodendroglia

ependymal cells

astrocytes

1085. In a healthy adult weighing 70 kg, plasma volume was estimated to be 3000 ml. His hematocrit was 40%. His blood
volume is approximately:

5000 ml

5200 ml

5400 ml

5600 ml

1086. Paneth cells are responsible for the secretion of what?

Antimicrobial peptides.

Bicarbonate.

Enzymes.

Secretin.

1087. The expiratory and inspiratory parts of the respiratory center are located in:

the medulla;

the midbrain;

the hypothalamus;

the spinal cord.

1088. The high rate of excitation through Purkinje fibers provides:

successive contractions of the atria and ventricles;

successive contractions of the right and left ventricles;

asynchronous contraction of ventricular myocardial fibers;

synchronous contraction of the ventricular myocardium fibers.

1089. Which of the following effects is not produced by stimulation of μ opioid receptor?
analgesia

constipation

miosis

diuresis

sedation

1090. Selective section of the pituitary stalk in an experimental animal usually increases the secretion of which of the following
hormones?

Growth hormone

FSH

LH

Prolactin

1091. Angiotensin II causes:

Increases proximal tubular reabsorption of Na & H2O & increases secretion of K

Increases distal tubular reabsorption of Na & H2O & decreases secretion of K

Decreases distal tubular reabsorption of Na & H2O

Increases excretion of Na & H2O

1092. The QRS complex on an ECG describes:

atrial depolarization;

atrioventricular delay;

ventricular depolarization;

ventricular repolarization.

1093. Which receptor does not span the cell membrane seven times?

Rhodopsin

Beta-adrenergic receptor

Insulin receptor

M2 receptor

1094. Volumetric flow rate:

maximum in the hollow veins;

maximum in the aorta;


is the same in all vessels;

changes in the capillaries.

1095. NaHCO3⁻ in the duodenum comes from:

acini.

epithelial ducts.

paneth cells.

the pancreas.

1096. Which of the following statements regarding reticular activating system is false?

RAS is found in the anterior pontine region

RAS is important for regulation of alertness & consciousness

RAS projects to cortex & thalamic nuclei

RAS is responsible for regulation of HR, BP & respiratory pattern

1097. Parasympathetic innervation to the heart is via the:

Phrenic nerve

Splanchnic nerve

Vagus nerve

Glossopharyngeal nerve

1098. Which of the following is true of the gastroenteric reflex?

It increases peristaltic activity of the stomach

It leads to defecation

It leads to increased peristaltic activity in the small intestine

It stimulates contractions of the colon

1099. Body mass index is calculated as:

Weight in pounds by height in meters

Weight in kg by height in meters

Weight in kg divided by square of height in meter squared

Weight in kg divided by body surface area


1100. In terms of the sensory nerve endings located in the skin, which of the following fibrres would not contain fast myelinated
fibres?

C-type free nerve endings

A-delta free nerve endings

Meissner corpuscles

Merkel's disks

1101. Which of the following is incorrect with respect to taste?

CN VII, IX and X synapse on the VPM nucleus of the thalamus

Each area of the tongue is associated with a particular taste, and receptors are specific for their binding ligands

Na+ is involved in the signal cascade associated with the taste "salt"

Taste buds are epithelial cells that are electrically excitable and synapse with neurons

1102. Duration of knee jerk reflex response is in the order of:

10 ms

20 ms

30 ms

50 ms

1103. Which of the following are correct for Gastrin?

Activates pepsin from pepsinogen.

Causes acid release.

Involved in alcohol absorption within the stomach.

None of the above

1104. Aldosterone is secreted by the:

adrenal medulla

hypothalmus

zona glomerulosa

all of the above

1105. Which of the following sleep disorder is associated with REM sleep?

Nocturnal enuresis
Narcolepsy

Night tremors

Sleep walking

1106. The following fluids are not part of the internal environment:

Blood;

Lymph;

Extracellular fluid;

Digestive juices.

1107. Which muscles are involved in inspiration?

Diaphragm

Diaphragm and external intercostal muscles and abdomial muscles

Diaphragm and External intercostal muscles

Diaphragm and Internal intercostal muscles

1108. Which has the greatest renal clearance?

PAH

Glucose Urea – No, 53% reabsorbed

Water

Inulin

1109. The volume of distribution of intravenously administered sucrose in a healthy well hydrated 70-kg man is about:

3.5 liters

10.5 liters

14 liters

28 liters

1110. In a healthy adult, a decrease in blood pressure is noted when:

physical activity;

emotional arousal;

jumping into cold water;

hypodynamia and hyperthermia.


1111. The total pause is:

atrial systole time;

atrial and ventricular diastole;

opening of semilunar valves;

expulsion of blood.

1112. The destruction of the spinal cord above 3-4 cervical segments causes respiratory arrest due to:

respiratory muscle paralysis;

automatic violation of the inspiratory center;

damage to the vagus nerves;

hypercapnia.

1113. Physiologically, the most important choleretic is:

bile salts

CCK

secretin

gastrin

1114. With reference to the action potential:

Decreasing the external [K+] decreases the RMP

Decreasing the external K+ concentration has little effect on the RMP

Increasing the external [Na+] decreases the RMP

None of he above are true

1115. The greatest increase in (physiological) dead space would be expected with:

Pulmonary embolism

Atelectasis (or: collapse of one lung)

Pneumothorax

Bronchoconstriction

Obesity

1116. Which of the following regarding the blood-testis barrier is incorrect?

It develops before spermatogenesis.


Leydig cells are part of the blood-testis barrier.

Disruption of the barrier makes spermatogonia prone to immunological attack.

none of the above

1117. In humans, the hormone that is mainly secreted by the adrenal medulla is:

Epinephrine

Norepinephrine

Dopamine

Adrenomedullin

1118. In which of the following segments in the splanchnic circulation is pressure least in healthy individual at rest?

hepatic arteriole|

hepatic sinusoid

hepatic vein

terminal branches of portal vein in the liver

1119. Stomach motility is increased in:

parasympathetic stimulation.

sympathetic stimulation.

during sleep

none of the above

1120. The flow of air into the alveoli during inhalation occurs due to:

the difference between blood pressure and the force of the elastic thrust of the lungs;

elastic thrust of the lungs;

the difference between atmospheric and intraalveolar pressure;

the difference between the partial pressure of O2 and CO2 in the alveolar air.

1121. ICF volume decreases when dehydration is:

Isotonic

Hypertonic

Hypotonic

all of the above


1122. Sodium transport through the brush border of the intestinal epithelial cells occurs by which of the following mechanisms?

Active transport

Co-transport with hydrogen ions

Facilitated diffusion

Passive diffusion

1123. Most of the T3 in the liver is formed from circulating T4 by the action of:

Type 1 deiodinase (D1)

Type 2 deiodinase (D2)

Type 3 deiodinase (D3)

1124. Which of the following is true?

arteries have a higher pressure than veins

veins and arteries have the same pressure

veins have a higher pressure than arteries

all of the above

1125. The action of the renin-angiotensin system is aimed at:

dilation of blood vessels and increased filtration in the kidneys;

vasoconstriction and reduction of filtration in the kidneys;

synthesis of erythropoietin and activation of blood formation;

the breakdown of hemoglobin and the formation of urobilin.

1126. Distribution of ventilation & perfusion:

Gradient of change in ventilation is greater than that for perfusion

Ventilation increases as go up the lung

Perfusion increases as go up the lung

V:Q ratio at apex is greater than at base

None of the above

1127. Semilunar valves are located in:

the mouth of the aorta;

the mouth of the hollow veins;


the mouth of the pulmonary veins;

between the atria and ventricles.

1128. Normal in a healthy heart, automatism manifests itself in:

typical myocardium;

atrioventricular node;

sinoatrial node;

bundle of His.

1129. Vasoconstricting humoral factors include:

acetylcholine;

bradykinin;

metabolites;

norepinephrine.

1130. Acidic histamine is secreted by:

Chief cells

Enterochromaffin cells

G cells

Parietal Cells

1131. In arterioles the nature of blood flow:

laminar;

turbulent;

chained;

intermittent.

1132. With regard to glomerular filtration:

Autoregulation maintains flow

Afferent arteriole driving force

Is equal for cationic & anionic molecules

1133. The amount of air inhaled or exhaled in one breath during related, quiet breathing is:

functional residual capacity


residual volume

tidal volume

total lung capacity

1134. The hormone that is synthesized only in the zona glomerulosa is:

DHEA

Aldosterone

Corticosterone

Deoxy corticosterone

1135. Which of the following glands does not secrete the mucous component of saliva?

Buccal

Parotid

Sublingual

Submandibular

1136. In humans, what percentage of nephrons has long loops of Henle?

15

25

85

1137. Which of the following hormones is not a glycoprotein?

TRH

FSH

LH

hCG

1138. Which of the following structures is not involved in the reflex arc?

White rami comitantes

Nociceptors

Dorsal root ganglion

Interneurons in the spinal cord


1139. How many transmembrane domains do G-protein coupled receptors have?

12

1140. Neurotransmitter synthesis normally occurs in which part of the cell?

Cytosol

Golgi Apparatus

Nucleus

Rough Endoplasmic Reticulum

1141. Which of the following modes of transport across the cell membrane is quantitatively more important for flux of ions?

Diffusion

Filtration

Vesicular transport

all of the above

1142. In the adult, most of the circulating erythropoietin originates from:

Interstitial cells (fibroblasts) surrounding peritubular capillaries in the renal cortex

Perivenous hepatocytes

Kupffer cells of liver

all o the above

1143. Specify endocardial function:

softens friction when the heart is working;

regulates blood flow through the heart vessels;

provides pumping activity of the atria and ventricles;

forms valves.

1144. The extrinsic pathway is triggered by the release of:

Factor VII

Tissue factor
Tissue factor pathway inhibitor

Contact factor

1145. Increased muscle spindle sensitivity to stretch secondary to increased Ay efferent discharge can occur with:

Jendrassiks maneuver

Hypertonia

Skin stimulation on the ipsilateral skin

All of the above

1146. The hormone that is essential for intrauterine development of male external genitalia is:

Testosterone

Dihydrotestosterone

Mullerian regression factor

SOX factor

1147. Stop breathing is called:

eipic;

hyper;

apnea;

tachypnoe.

1148. The “penis-at-12” syndrome occurs due to a deficiency of:

17α hydroxylase

Testosterone 5α reductase

11β hydroxysteroid dehydrogenase

CYP 450scc

1149. Regarding cholinergic stimulation:

Causes amylase secretion from salivary glands

Has no effect on renal arterioles

Relaxes bronchial smooth muscle

Relaxes the gallbladder

1150. The commonest cell type in the anterior pituitary is:


Somatotroph

Lactotroph

Corticotroph

Thyrotroph

1151. The amount of proteins in the blood plasma is equal to:

2–3%;

8 - 10%;

6 - 8%;

10 - 15%.

1152. The heart continues to beat even after all nerves to it are sectioned. This property is called:

excitability

conductivity

automaticity

contractility

1153. Na-K-2Cl transporter in the apical membrane of the thick ascending limb of the loop of Henle is an example of:

Primary active transport

Secondary active transport

Passive transport

Counter transport

1154. Acinus is:

upper airway;

the system of small bronchi;

respiratory bronchioles, alveolar passages and alveoli;

a collection of alveoli and all airways.

1155. Cytosolic calcium concentration in unexcited cells is about:

2.5 mM

10 mM

150 μM
100 nM

1156. Filtration in the kidneys is:

the process of transition of water and substances dissolved in it from the blood into the Bowman-Shumlyansky capsule;

the process of transition of water and substances dissolved in it from the blood into the loop of Henle;

the process of transition of water and substances dissolved in it from the tubules into the blood;

the process of transition of water and substances dissolved in it from the tubules into the collecting tubules.

1157. In an individual with a blood hemoglobin concentration of 10 g/dL and a hematocrit of 40, MCHC is approximately:

20 g/dL

25 g/dL

30 g/dL

35 g/dL

1158. During the cardiac cycle, aortic valve closes at the end of:

isovolumetric systole

rapid ejection

diastasis

protodiastole

1159. Which of the following is not a function of astrocytes?

Remove dead cells post-inflammatory response

Contribute to the blood brain barrier

Repair damaged tissues

Maintain electrolyte levels in the brain

1160. The ability of the AV node to generate its ownimpulses when the sinus node is “sick” is due to:

a constant phase 4 membrane potential

slow calcium entry during phase zero

spontaneous diastolic depolarization

the absence of prepotentials

1161. Alveolar dead space:

Is less than physiological dead space


Is decreased with mechanical ventilation

Is increased with hypotension

1162. Which of the following structures not involved in visual processing?

Medial Geniculate nucleus

Visual Cortex

Lateral geniculate nucleus

Meyer's Loop

1163. Which of the following refers to minute ventilation?

the minimum volume that can be expired at rest

the volume of air breathed per minute

the volume of air that remains in collapsed lungs

all of the above

1164. The GABA-A receptor is a:

Na channel

Cl ion channel

Ca channel

cation channel

1165. Which of the following is an example of primary active transport?

Ca sequestration in sarcoplasmic reticulum by Ca-ATPase

Glucose entry into cells through glucose transporter 2 (GLUT-2)

H2O flux across cell membranes through aquaporins

Glucose uptake into intestinal epithelial cells by Na-glucose cotransporter 1 (SGLT-1)

1166. Micelles in bile are formed by:

bile salts and phospholipids

bile acids and bile salts

cholesterol and bile salts

cholesterol and phospholipids

1167. The sequence of contractions first atrial and then ventricles is due to:
the work of semilunar valves;

the pacemaker function of the sinoatrial node;

pressure gradient between the atria and ventricles;

atrioventricular delay.

1168. In the context of blood transfusions, ABO compatibility is important because:

There are 3 antigens in this system

The A and B antigens are present in all cells

When an individual’s RBC lacks the A or B antigen, the corresponding antibody is invariably present in serum.

O is a strong antigen

1169. Water excretion by the kidney is due to:

Osmosis

Active transport into the lumen

Passive secretion in the collecting tubules

Solvent drag

1170. Insulin dependent glucose uptake into skeletal muscle and adipose tissue is mainly mediated by:

GLUT 1

GLUT 2

GLUT 3

GLUT 4

1171. Auerbach's Plexus is also known as what?

Enteric plexus.

Interstital cells of Cajal.

Myenteric plexus.

Submucosal plexus.

1172. Homeostasis is:

Destruction of red blood cells;

The ratio of blood plasma and blood cells;

Blood clot formation;


The constancy of indicators of the internal environment.

1173. Which of the following best characterizes the secretions of the small intestine?

Hypotonic and slightly acidic

Hypotonic and slightly alkaline

Isotonic and slightly acidic

Isotonic and slightly alkaline

1174. The PQ interval on the ECG shows:

the spread of excitation in the ventricles;

the spread of excitation in the atria and through the atrioventricular node;

a general pause of the heart;

the duration of the atrial cycle.

1175. Alveoli in healthy people do not stick together when falling, because:

they have a water film;

there are elastic fibers in the wall of the alveoli;

in the alveoli there is a surfactant that reduces surface tension;

pleural sheets have the ability to absorb water.

1176. External breathing is:

the exchange of gases between the lungs and blood;

transport of gases by blood;

the exchange of gases between atmospheric and alveolar air;

the exchange of gases between the blood and tissues.

1177. In violation of the connection between the spinal cord and the medulla oblongata:

breathing stops completely;

abdominal breathing persists;

the smoothness of breathing is disturbed;

breathing does not change.

1178. Indicate which of these mechanisms narrows the vessels:

axon reflex;
a decrease of the sympathetic nervous system;

improving of the sympathetic nervous system;

increase the tone of the parasympathetic nervous system.

1179. In an Rh-negative mother not previously sensitized by the Rh antigen, Rh incompatibility does not usually have a serious
consequence during the first pregnancy because:

Antibodies are not able to cross placenta

The titer of IgG is low during the primary immune response

IgG is ineffective against fetal red cells

Massive hemolysis in the fetus is compensated by increased erythropoiesis

1180. When sitting or standing, blood flow is highest in the ____ of the lung.

apex

base

lingula

middle lobe

1181. A neonate scheduled for abdominal surgery has a serum calcium of 6 mg/dL (normal: 8.5-10.5 mg/dL). Before
supplementing calcium, which of the following should be checked?

Blood glucose

Serum albumin

Serum bilirubin

Oxygen saturation

1182. Cell volume and pressure is mainly dependent upon activity of:

Na glucose cotransporter

Na-K pump

Glucose transporter

Na-Ca exchanger

1183. Which of the following cell types found in the intestinal tract secretes hydrochloric acid?

Acinar cells

Mucous neck cells

Parietal cells
Peptic cells

1184. The pressure in the pleural cavity with deep expiration is:

6 - 9 mm Hg. v .;

3 –4 mm Hg. v .;

15 - 20 mm Hg. v .;

1, 0 mm Hg. v .;

1185. IPSPs due to chloride influx are produced by:

acetylcholine

GABA

glutamate

substance P

1186. In excitation-contraction of skeletal muscle, calcium binds to:

Myosin

Tropomyosin

Troponin C

Troponin I

1187. The highest number of voltage gated Na+ channels are found in:

Cell body

Initial segment of axon

Terminal buttons

Myelin sheath

1188. The lung:

Removes/inactivates serotonin (5HT)

Activates bradykinin

Converts angiotensin II to I

Inactivates aldosterone

1189. Adaptation to temperature changes occurs at:

0-20°C
20-40°C

40-45oC

45+oC

1190. Heme is converted to bilirubin mainly in the:

Kidneys

Liver

Spleen

Bone marrow

1191. Adrenaline, noradrenaline and dopamine act upon membrane receptors that span the membrane:

2 times

5 times

7 times

10 times

1192. The biologically most active androgen is:

Testosterone

Dehydropepiandrosterone

Androstenedione

Dihydrotestosterone

1193. Which of the following composes the majority of salivary volume?

Parotid gland

Small salivary glands

Sublingual gland

Submandibular gland

1194. What form of anaemia is associated with Gastritis?

Haemolytic

Iron deficiency

Megaloblastic

Pernicious
1195. The reason for the diffusion of gases from the alveolar air into the blood and back is:

the difference in partial pressure and tension between the alveolar air and blood;

close adherence of alveoli and capillaries;

active transport of O2 and CO2;

1196. Which of the following statements regarding sensory nervous system is INCORRECT?

10 sensory nerve fibers for fine touch & proprioception travels up the ipsilateral dorsal column of spinal cord to reach the
medulla

10 sensory nerve fibers for temperature & pain travel up the ipsilateral antero-lateral column of spinal cord to reach the
thalamus.

Fibers in the dorsal column do not decussate in spinal cord

Antero-lateral column also give sensory input to the reticular formation

1197. What is a syncytium?

A cluster of cells within the pericardium

A cluster of pacemaker cells in the right atrium, present in <20% of the population

A group of cardiac myocytes that function as a single unit

A rare extra blood vessel connecting the left and right coronary arteries

1198. Significant tubular reabsorption occurs with:

Phosphate

Creatinine

Urea

Sulphate

All of the above

1199. The volumetric rate of blood flow is:

the force with which blood pushes against the walls of blood vessels;

the volume of blood ejected by the heart into the aorta in one systole;

the volume of blood that passes through the vessel per unit of time;

the distance that a particle of blood travels per unit of time.

1200. Which of the following activates trypsin in the small intestine?

Cl⁻
Enterokinase

Kazal

NaHCO3

1201. Which of the following markers is incorrectly matched with its volume of distribution?

Antipyrine - total body water

Inulin - ECF volume

Evans blue - plasma volume

I125albumin - blood volume

1202. Which segment of the nephron is permeable to urea?

Descending limb of loop of Henle

Thin ascending limb of loop of Henle

Thick ascending limb of loop of Henle

1203. The oral rehydratation solution is helpful in rehydration in diarrheas because:

Na-glucose symporter is unaffected in toxigenic diarrheas

cAMP regulated Cl secretion is reduced

toxins inhibit facilitated glucose transport

all of the above

1204. The rate of diffusion of a substance across the cell membrane is inversely proportional to:

Concentration gradient for the substance

Diffusion coefficient

Surface area available for diffusion

Thickness of the membrane

1205. Hyperthyroidism following administration of large amounts of iodine (iodine induced hyperthyroidism; Jod-Basedow
mechanism) is least likely in an individual with:

Pan hypopituitarism

Endemic (iodine deficiency) goiter

Hashimoto’s thyroiditis

Graves’ disease
1206. Which of the following regulates body temperature?

cerebrum

cerebellum

thalamus

hypothalamus

1207. ECF surrounding the SA node has a notably high concentration of:

Calcium

Chloride

Potassium

Sodium

1208. Hans Selye defined “stressors” specifically as stimuli that increase the secretion of ______ in healthy humans or
laboratory animals.

CRH

ACTH

TSH

Cortisol

1209. Relative, dynamic constancy of internal environment is called:

Hemolysis;

Hemostasis;

Hemeostasis;

Blood transfusion.

1210. Which of the following statements about the retina is FALSE?

Macula lutea is the point of greatest visual acuity.

Retinal blood vessels supply the rods & cones in retina.

There are no blood vessels in the macula lutea

There are no rods in the macula lutea

1211. Which of the following statements regarding visual pathways is INCORRECT?

Objects in the temporal visual field is sensed by the nasal hemiretina


Interruption of optic nerve leads to blindness in the ipsilateral eye only

Fibers from temporal hemiretina decussate at the optic chiasm

Lateral geniculate body receives fibers from the ipsilateral temporal hemiretina & contralateral nasal hemiretina.

1212. Presynaptic inhibition occurs at:

axoaxonal synapses

axosomatic synapses

axodendritic synapses

all of the above

1213. Passive exhalation begins with:

expiratory muscle contraction;

relaxation of the inspiratory muscles;

air outlet from the lungs;

contraction of the inspiratory muscles.

1214. Which of the following components of bile is critical for fat digestion?

Bicarbonate

Bilirubin

Calcium salts

Lecithin

1215. Figures A and B given below depict the relationship between transport rates (in Y-axis, inarbitrary units) and
concentration gradients (in X-axis, in arbitrary units) of two substances A and Brespectively. Fig A Fig B Which of the following
statement

B is transported by facilitated diffusion

Transport of B is ATP dependent

There is a ‘transport maximum’ for B

Substance A would move in one direction only

1216. Which of the following organelles is a major site of steroidogenesis?

Peroxisomes

Ribosomes

Smooth endoplasmic reticulum


Rough endoplasmic reticulum

1217. What does the Frank-Starling mechanism represent?

cardiac output and strong volume

end-diastolic ventricular volume and cardiac output

end-diastolic ventricular volume and stroke volume

stroke volume and end-systolic ventricular volume

1218. Stomach emptying is controlled mostly by which of the following?

Activity if the small intestine

Local neurohumoral mechanisms originating in the duodenum.

Local neurohumoral mechanisms originating in the stomach itself

Vasovagal reflexes that reduce the tone in the muscle wall of the body of the stomach

1219. Increased physiological dead space with:

Age

Anaesthesia

Supine position

All of the above

1220. Increasing the tone of the vagus nerves during sleep:

strengthens and improves the functioning of the heart;

weakens and slows down the heart;

does not have a noticeable effect on the work of the heart;

has a phase amplifying - weakening effect.

1221. Which of the following is not true about saliva?

It is initially produced in Acinar glands

It is produced by both parasympathetic and sympathetic stimulation

The rate of its flow through the duct affects its final composition

When secreted into the mouth, it is both an aklaline and a hypotonic solution

1222. The phase of relative refractoriness in the heart coincides with:

AP plateau;
rapid initial depolarization;

fast final repolarization;

1223. Which term refers to the volume remaining in the lungs after maximum expiration?

expiratory reserve volume

inspiratory reserve volume

minimal volume

residual volume

1224. What takes place during initiation?

Depolarization

Hyperpolarization

None of the above

Repolarization

1225. Heart rate is slowed by:

deep inspiration

Bainbridge reflex

increased intracranial tension

carotid massage

1226. Which of the following is a potent stimulator of insulin secretion?

Somatostatin

VIP

Glucagon like polypeptide-1 (GLP-1)

Leptin

1227. Which of the following is a bile acid synthesized by the hepatocyte?

sodium taurocholate

chenodeoxycholic acid

deoxyholic acid

lithocholic acid
1228. A lab technician determines RBC count by manual hemocytometry, blood hemoglobin concentration by Sahli’s acid
hematin method, and hematocrit using a microcentrifuge. He follows all procedures correctly. Which of the following RBC
indices calculated from

Mean corpuscular volume

Mean corpuscular hemoglobin

Mean corpuscular hemoglobin concentration

Mean cell diameter

1229. Excitatory amino acids in the brain are:

GABA & glycine

Glutamate & aspartate

Glutamate & GABA

Glutamate & glycine

1230. Which of the following hormones is critical for acute regulation of a falling serum ionized calcium?

Calbindin

Calcitonin

Calcitriol

Parathyroid hormone

1231. Primary hemostasis refers to cessation of bleeding due to:

Formation of a definitive clot

Clot retraction

Formation of a temporary platelet plug.

none of the above

1232. Which of the following is not typically found in saliva?

Bicarbonate

HCl

IgA

Lysozyme

1233. As go from the top of the erect lung to the bottom:


Water vapour pressure remains constant

pN2 remains constant

pCO2 at apex is higher than at the base

pO2 at base is lower than at the apex

1234. The ‘diluting segment’ of the nephron is:

proximal tubule

descending limb of loop of Henle

ascending limb of loop of Henle

cortical collecting duct

1235. Excitatory postsynaptic potentials (EPSP) are not produced by:

opening of Na channels

closure of K channels

opening of ca channels

opening of Cl channels

1236. Across which site in the circulation is the pressure drop maximum?

Arterioles

Venules

Capillaries

Aortic valve

1237. Which of the following statements regarding aqueous humor is FALSE?

Aqueous humor is produced by ciliary body by active transport

Aqueous humor is reabsorbed through the trabecular meshwork into the canal of Schlemn.

Canal of Schlemn is found at the anterior chamber angle.

Acute angle closure glaucoma is due to the obstruction of anterior chamber angle.

1238. Which of the following transporters is present only in sinusoidal membrane of hepatocytes?

bile salt export pump

LDL-cholesterol receptors

MDR 3

multidrug resistance associated protein 2 (MRP 2)


multidrug resistance associated protein-2 (MRP-2)

1239. Which of the following increases insulin / glucagon molar ratio the most?

A large carbohydrate meal

Intravenous glucose

Overnight fast

A small protein meal

1240. The volume of RBC placed in a NaCl solution with an osmolality of 280mOsm/Kg H2O is 100 femtoliters. What will be the
steady state volume when they are placed in a NaCl solution with an osmolality of 350mOsm/Kg H2O?

120 fL

80 fL

50 fL

Data inadequate

1241. Histamine released from mast cells in the stomach stimulates the secretion of HCl by parietal cells. What type of
signaling is this?

Endocrine

Paracrine

Autocrine

Juxtacrine

1242. What is the correct order of the reflex arc?

Sensory organ - afferent neuron - synapse - efferent neuron - effector response

sensory organ - efferent neuron - afferent neuron - synapse - effector response

Sensory organ - synapse - afferent neuron - efferent neuron - effector response

Sensory organ - efferent neuron -synapse - afferent neuron - effector response

1243. Which of the following statements are true?

Potassium ions have a higher concentration outside the membrane

Potassium ions have a lower concentration inside the membrane

Potassium ions have a lower concentration outside the membrane

Sodium ions have a lower concentration inside the membrane

1244. The inhibitory amino acid neurotransmitters in the CNS are:


glutamate and glycine

glutamate and aspartate

GABA and glycine

aspartate and glycine

1245. Anemic hypoxia occurs when:

reducing the oxygen capacity of the blood;

impaired hemodynamics;

a decrease in pO2 in atmospheric air;

changes in the function of respiratory tissue enzymes.

1246. The enzyme that ultimately lyses fibrin is:

Plasminogen

TPA

Urokinase

Plasmin

1247. Which of the following muscles is subject to control by voluntary as well as reflex mechanisms?

external anal sphincter

lower esophageal sphincter

pyloric sphincter

all of the above

1248. Potassium secretion occurs mainly in:

PT

early distal tubule

late distal tubule

collecting ducts

loop of Henle

1249. Hypercapnia, hypoxemia and acidosis cause:

increase minute volume;

lowers minute volume;

respiratory arrest;
respiratory arrest;

do not change the minute volume.

1250. Indicate a conduction disturbance:

tachycardia;

extrasystole;

sinus arrhythmia;

atrioventricular block.

1251. The adhesion of platelets to sub endothelial collagen is impaired in the absence of:

Von Willebrand factor

Plasmin

Geparin

Antithrombin III

1252. Which of the following is true about the parotid gland?

It has a predominantly mucous secretion

It is innervated by the glossopharyngeal nerve (CN IX)

It is the smallest salivary gland

It produces the majority of saliva by volume

1253. Which heart sound is made by rapid ventricular filling following atrial contraction, usually only detected by
phonocardiography?

S1

S2

S3

S4

1254. Which of the following normally prevents the activation of pancreatic enzymes?

Secretion of trypsin inhibitor by the acinar cells

Storage at a neutral pH within the acinar cells

Storage at an acidic pH within the acinar cells

Tonic secretion of sodium bicarbonate

1255. The red blood cells of a person with the Bombay blood group do not have:
GLUT

H substance

Spectrin

Ankyrin

1256. What percentage of plasma is water?

89.5%

90%

91.5%

92%

1257. The involvement of the hypothalamus in the regulation of respiration is:

innervation of the respiratory muscles;

the integration of respiration with the vegetative functions of the body;

coordination of breathing with speech and singing;

ensuring rhythm and automatism of breathing.

1258. Regarding the nerve fibre action potential, which statement is incorrect?

After-depolarisation commences when repolarisation is 70% complete

Does not involve calcium ions

Spike potential peaks at sodium equilibrium potential

Total number of ions involved is minute compared to the total number present

1259. In which of the following does digestion occur?

Ascending colon

Duodenum

Ileum

all of the above

1260. Which of the following statement regarding sensory cortex is TRUE?

Cells for various sensory modality for a given body part is arranged in columns in the 10 sensory cortex.

20 sensory cortex is found in the sylvian fissure

10 sensory cortex is important for sensory processing regarding position sense & discrimination of size & shape of the
objects.
j

All of the above

1261. Which of the following is true about receptors?

All receptors are post-synaptic

Ligand-receptor binding can only cause effect through second messengers

Prolonged exposure to ligands causes decreased receptor response

There is only 1 receptor type for each ligand

1262. The test that screens the extrinsic pathway is:

Prothrombin time (PT)

Activated partial thromboplastin time (aPTT)

Thrombin time

Urea solubility test

1263. The principal estrogen secreted by the fetoplacental unit is:

Estriol (E3)

Estradiol (E2)

Estrone (E1)

1264. Absorption of water in the small intestine occurs by which of the following?

Active transport

Couple transport with glucose

Passive diffusion

Solvent drag

1265. Inhibition of the active transport of sodium through the basolateral membrane of the small intestinal epiithelial cells does
which of the following?

Attenuates glucose uptake from the small intestine

Attenuates the absorption of fructose

Facilitates glucose uptake from the small intestine

Facilitates the absorption of water

1266. The amount of hemoglobin present in 100 ml of red blood cells is defined as:

MCH
MCHC

Hemoglobin index

RDW

1267. The protein content of lymph draining from the _______ is highest.

Choroid plexus

Skeletal muscle

Liver

Gastrointestinal tract

1268. The blood-brain barrier

is formed by tight junctions.

is formed by oligodendrocytes.

tightly regulates the intracellular environment of the CNS.

uses chemical signals to communicate with the spinal cord.

1269. Red blood cells from a healthy individual were placed in each of the following solutions and observations were made after
1 hour. Cells would have most likely have lysed when placed in:

0.3% NaCl

0.9% NaCl

1.2% NaCl

all of the above

1270. Vagus nerve in the heart:

increases the excitability;

lowers contractility;

increases the heart rate;

increases contractility.

1271. The maximum pressure in the right ventricle is fine:

25 to 30 mm. Hg v .;

10 to 15 mm. Hg v .;

60 - 80 mm. Hg v .;

120-130 mm. Hg v.
1272. In a healthy euthyroid adult, the thyroid gland predominantly secretes:

Thyroxine

Triiodothyronine

Reverse T3

Reverse T4

1273. For a euthyroid woman who is neither pregnant nor nursing, and taking no medication, the recommended daily dietary
allowance of iodine is:

75 micrograms

150 micrograms

225 micrograms

300 micrograms

1274. Prolongation of prothrombin time does not occur when there is a deficiency of only:

Factor VIII

Factor IX

Factor X

Vitamin K

1275. If the intent is to replenish total body water in a dehydrated individual which of the following should be administered
intravenously?

0.9% NaCl

5% dextrose solution

Albumin

10% glucose solution

1276. Normally, the thyroid gland secretes:

CGRP

PTH related peptide

TSH

Calcitonin

1277. In an adult female who underwent pelvic surgery, internal genitalia were found to be male, and the karyotype was 46 XY.
The most likelyexplanation for this is:
Loss of function mutation in SRY gene

Deficiency of testosterone

Deficiency of Mullerian inhibiting peptide

Deficiency of testosterone 5α reductase

1278. Surfactant in the alveoli:

reduces the surface tension of the water film;

increases the permeability of the alveoli for gases;

creates an elastic craving of the lungs;

increases the surface tension of the water film.

1279. Regarding the resting membrane potential, which is true?

Implies that the inside of the cell is positive relative to the outside of the cell at rest

Is +70mV in mammalian cardiac cells

Is decreased by increasing the external K+ concentration

Is increased with increased external Na+ concentration

1280. Which is the commonest type of synapse in the CNS?

axosomatic synapse

axodendric synapse

anoaxoal synapse

none of the above

1281. Regarding nerve action potential, which is true?

During depolarisation, Na+ permeability is greater than K+ permeability

Hyperpolarisation is due to continuous opening of Na+ channels

Initial rapid depolarisation is due to rapid influx of Ca++ via fast channels

Plateau is formed by slow Ca++ channels

1282. Which of the following hormones has maximum glucocorticoid activity?

Cortisol

Corticosterone

Deoxy corticosterone
Cortisone

1283. The systemic circulation begins:

pulmonary trunk;

two pulmonary arteries;

the aorta;

two vena cava.

1284. When a serum sample is electrophoresed, which of the following bands is normally absent?

Albumin

α1 globulin

α2 globulin

Fibrinogen

1285. The stretch reflex in skeletal muscle:

Is a feedback reflex aimed at maintaining muscle length

Is a polysynaptic reflex

Is not elicited in the knee jerk which occurs after tapping

Maintains muscle strength at various levels of muscle strength

1286. Segmental contractions within the GIT cause:

atony.

churning.

mixing.

peristalsis.

1287. Regarding synaptic structure/function:

An IPSP can be produced by localised increase in membrane permeability to Cl-

Chemical mediators located in presynaptic fibres contain one-way conduction of impulses

Small clear vesicles in the presynaptic tunnel contain catecholamines

The EPSP is an all-or-none response

1288. When inhaling dust or pungent odor, reflexes occur:

protective (coughing, sneezing);


Goering-Breuer;

from chemoreceptors of the vascular zones;

from the baroreceptors of the vascular reflexogenic zones.

1289. The satiety-producing hormone is:

Orexin

Resistin

Adiponectin

Leptin

1290. The main organs of the blood depot are:

Bones, ligaments;

Liver, skin, spleen;

The heart, lymphatic system;

The central nervous system.

1291. Arrange the following steps in the order in which they occur in the thyroid gland.

Coupling reaction

Iodination

Secretion of T4

Oxidation of iodine

1292. Which of the following processes is ATP dependent? (Select all correct answers).

Acidification of lysosomes

Actin-myosin cross bridge cycling

Exocytosis

Micropinocytosis

All correct

1293. The participation of the cerebral cortex in the regulation of respiration consists in:

ensuring the rhythm and automatism of breathing;

the manifestation of conditioned respiratory reflexes and the coordination of breathing with speech and singing;

innervation of the respiratory muscles;

f f f
the integration of respiration with the vegetative functions of the body.

1294. The testis-determining gene (or the SRY gene) is located normally on/in:

Chromosome 6

Chromosome 12

Short arm of chromosome 22

Y chromosome

1295. The characteristics of the I tone include:

is heard only in young people, diastolic;

deaf, low, long, systolic;

tall, short, sonorous, diastolic;

not heard, registered only on PCG, diastolic.

1296. Glucagon, when present in high concentrations such as in a type I diabetic, facilitates all of the following except:

Lipolysis

Gluconeogenesis

Hepatic glycogenolysis

Muscle glycogenolysis

1297. Which of the following is incorrect about fetal hemoglobin (Hb F)?

In comparison to HbA, Hb F has greater affinity for 2,3–BPG

The oxygen dissociation curve of HbF is shifted to the left relative to HbA.

At low PO2, Hb F gives up more oxygen to tissues than Hb A.

none of the above

1298. The secretion of bile is important for the proper digestion of which of the following?

Complex carbohydrates

Lipids

Monosaccharides

Proteins

1299. The functional dead space includes:

the alveoli, which are ventilated but not perfused (no blood flow);
alveoli, which are ventilated and perfused;

the trachea;

bronchi.

1300. The most abundant androgen in the plasma of both males and females is:

Androstenedione

Dehydroepiandrosterone sulfate

Testosterone

Dihydrotestosterone

1301. The high pressure zone of the cardiovascular system includes:

the left ventricle, aorta, arteries and arterioles of the systemic circulation;

the right ventricle, arteries and arterioles of the pulmonary circulation;

the right atrium, venules and veins of the systemic circulation;

the right atrium, the left atrium and the pulmonary circulation.

1302. Echocardiography is registration:

heart tones;

reflected ultrasound waves from the structures of the heart;

the biopotentials of the heart;

mechanical vibrations of the body when the heart is working.

1303. In which zones is corticosterone converted to aldosterone?

Zona glomerulosa

Zona fasciculata

Zona reticularis

none of the above

1304. The pattern of intestinal motility that hastens transit in small intestine in the digestive state is:

peristaltis

segmentation contraction

tonic contraction

migrating motor complex


1305. Which of the following hormones is / are diabetogenic? (Check all correct answers)

Epinephrine

Cortisol

Growth hormone

all of the above

1306. In the absence of vasopressin, most of the filtered water is reabsorbed in the:

PT

loop of Henle

DT

collecting ducts

1307. Structural and functional unit of the kidneys is:

acinus;

micelle;

nephron;

neuron.

1308. Hypophysectomy does not immediately affect the secretion of:

Zona glomerulosa

Zona fasciculata

Zona reticularis

all of the above

1309. Osmotic pressure of crystalloids is not included in the Starling’s equation describing fluid exchange across capillaries
because:

They were discovered after Starling described the forces

Crystalloids carry electric charges

Crystalloids diffuse freely across the capillary endothelium

The quantity of crystalloids in plasma is negligible compared to plasma proteins

1310. Where are action potentials delayed to allow for atrial contraction prior to ventricular contraction?

AV node

Bundle branches
Bundle of His

Purkinje fibres

1311. The peripheral chemoreceptors:

Have a nonlinear response to paO2 changes

Have an intact response at 1MAC

Respond to a fall in PaCO2

Respond to alkalaemia

Are in the carotid sinus

1312. The microcirculation system is not involved in:

ensuring adequate metabolism in the tissues;

ensuring venous return of blood to the heart;

providing capacitive function;

determining the speed of propagation of the pulse wave.

1313. ECF volume is determined by:

Plasma [Na]

Plasma protein concentration

The amount of sodium in the ECF

all of the above

1314. Which of the following statements regarding somatostatin is incorrect?

It inhibits release of growth hormone

It inhibits release of insulin

It decreases blood flow to the intestine

It stimulates gastric acid secretion

1315. Renal blood flow is dependent on:

Juxtaglomerular apparatus

[Na+] at macula densa

Afferent vasodilatation

Arterial pressure
Efferent vasoconstriction

1316. Which of the following substances stimulates the motor activity of the stomach?

Cholecystokinin

Gastrin

Norepinephrine

Secretin

1317. Deficiency of which of the following hormones is not known to result in a clinically significant disorder of regulation of
ionized calcium in plasma?

Parathyroid hormone

Calcitriol

Calcitonin

Calcium sensing receptor

1318. The average half-life of neutrophils in the circulation is:

6 hours

5 days

2 weeks

1 month

1319. Which of the following are paired correctly?

Cholecystokinin-gallbladder relaxation

Cholecystokinin-relaxation of the sphincter of Oddi

Secretin- relaxation of the sphincter of Oddi

Secretin-stimulation of enzymatic release from the pancreatic acinar cells

1320. Anatomical dead space performs the following functions:

maintains the partial pressure of oxygen;

warms, moisturizes, cleans the air, includes protective reflexes;

regulates tidal volume;

regulates the composition of alveolar air.

1321. Which one of the following statements is false?

f f
Membrane potential is a combinatino of two equilibrium potentials guided by the permeability of two ions

There would be a chemical gradient present & energy present in the system if the concentration gradient of potassium ionsc.
Resting membrane potential (RMP) is constantly changing & isn't at rest as its terminlogy would suggest

Small local ion fluxes across the membrane mean the resting membrane potential fluctuates - it is not constant

1322. Atrioventricular delay:

regulates the strength of contractions of the heart;

provides blood flow to the heart;

provides consistent contractions of the atria and ventricles;

sets the rhythm of the heart.

1323. In the steady state, the value of which of the following variables is the same in ICF and ECF?

pH

Osmolality

Concentration of proteins

Number of osmoles

1324. The rate-limiting step in the biosynthesis of angiotensin II is the synthesis of:

renin

angiotensinogen

angiotensin I

angiotensin II

1325. Select all correct answers. TSH secretion is inhibited by:

Dopamine

Somatostatin

T3 and T4

all of the above

1326. Name the function of the heart:

blood depot;

organ hematopoiesis;

double action pump;

organ excretion.
1327. What percent of blood comes from the ventricles to the vessels in the phase of rapid ejection:

60%;

70%;

30%;

50%.

1328. Inhale is:

an active process of air flow into the lungs;

an active process of carbon dioxide in the lungs;

passive process of air flow into the lungs;

active process of removing carbon dioxide from the lungs.

1329. The major humoral mediator of pancreatic duct cell secretion of bicarbonate rich juice is:

secretin

CCK

somatostatin

all of the above

1330. Vasodilators of humoral factors include:

acetylcholine;

adrenaline;

vasopressin;

thyroxin.

1331. Pulse pressure is:

the minimum pressure in the arteries;

the true pressure in the arteries, if smooth pulsations;

the difference between systolic and diastolic pressure;

maximum pressure in the arteries.

1332. The juxtaglomerular apparatus of the kidney is:

a triangle formed by the walls of the bearing and carrying out arterioles and cells of the distal tubules;

a triangle formed by the walls of the bearing and carrying out arterioles and the cells of the proximal tubules;
a triangle formed by cells of the proximal and distal tubules;

the loop of Henle, collecting tubules and vessels that surround them.

1333. Which of the following contains regulatory centers for the respiratory and circulatory systems?

cerebrum

medulla oblongata

thalamus

hypothalamus

1334. The action potential of a neuron:

Declines in amplitude as it moves along te axon

Is initiated by efflux of Na

Is not associated with any net movement of Na+ or K+ across the cell membrane

Is terminated by efflux of K

1335. In glucocorticoid-remediable aldosteronism:

Angiotensin II has no effect on aldosterone synthesis

Angiotensin II stimulates whereas angiotensin III inhibits aldosterone synthesis

Aldosterone synthase is expressed in the zona fasciculata and exquisitely sensitive to stimulation by ACTH.

Administration of glucocorticoids increases ACTH levels

1336. The ascending limb of the Loop of Henle is: (thick)

Impermeable to Na

Involved in active transport of K+ into the lumen

Involved in active transport of Cl- out of lumen

Involved in active transport of Na+ into lumen

1337. Blood pressure in an adult (18 – 45 years) normally is:

100/50 mm Hg;

120/80 mm Hg.;

70/40 mm Hg.;

170/90 mm Hg.

1338. The Frank-Starling Act is:


decompensation of the heart;

violation of the heart;

increase in heart rate;

increase in heart rate in response to stretching of the heart chambers.

1339. Most of the calcium in the endoplasmic reticulum is sequestered by:

Calmodulin

Caldesmin

Calbindin

Calsequestrin

1340. Regarding synapses:

Acetylcholine is present in granulated vesicles in synaptic knob

The amount of transmitter released is proportionate to Ca++ efflux

The EPSP is caused by Na+ influx

Transmitters are released from synaptic knobs secondary to Na+ triggers

1341. Which G protein activates adenylyl cyclase?

Gs

Gq

Gi

Gl

1342. The QRST complex on an electrocardiogram is:

ventricular repolarization;

ventricular depolarization;

the spread of depolarization in the atria;

electrical ventricular systole.

1343. The systemic blood circulation ends with:

pulmonary trunk

pulmonary veins

vena cava
the aorta.

1344. New cortical cells are formed from reserve cells in the:

Zona glomerulosa

Zona fasciculata

Zona reticularis

none of the above

1345. The main function of anastomoses is:

ensuring adequate metabolism in organs and tissues;

discharge of blood from arterioles into venules, bypassing capillaries;

the transformation of intermittent blood flow into a continuous;

the transformation of laminar blood flow into turbulent.

1346. Which of the following hormones does not act via a G-protein coupled receptor in the cell membrane?

Thyrotropin releasing hormone

Angiotensin II

Antidiuretic hormone

Thyroxine

1347. Regarding noradrenergic stimulation:

Causes erection in males

Elevates free fatty acid levels

Increases blood flow to the skin

Increases thresholds in the reticular formation

1348. IV heart tone occurs during:

asynchronous reduction;

protodiastoles;

presystoles;

fast filling.

1349. Central chemoreceptors:

Bathed in CSF
Respond to increase in CSF pH

Bathed in ECF

In medullary respiratory centre

1350. III tone occurs in:

proto-diastole;

atrial systole phase;

the phase of rapid filling of the ventricles;

the phase of isometric contraction.

1351. In quiet breathing, exhalation is:

Passive due to elastic tissue alone

Passive due to surface tension in the alveoli and elastic tissue recoil

Active due to intercostal contraction

1352. Circulatory hypoxia occurs when:

reducing the oxygen capacity of the blood;

impaired hemodynamics;

reducing the partial pressure of O2 in the atmospheric

the air;

changes in the function of respiratory tissue enzymes.

1353. Which of the following is true regarding Poiseuille's law? (choose all that apply)

decreased hematocrit; decreased viscosity

decreased hematocrit; increased resistance

decreased hematocrit; increased viscosity

none of the above

1354. The most abundant anion in ECF is

Bicarbonate

Chloride

Phosphate

Protein anion
1355. Which is a false statement?

Highest amount of resistance in the upper airways

The lower airways have a small total area

The upper airways have a high velocity flow.

The upper airways have turbulent flow

1356. The main function of the capillaries is:

ensuring adequate metabolism in organs and tissues;

discharge of blood from arterioles into venules, bypassing capillaries;

the transformation of intermittent blood flow into a continuous;

the transformation of laminar blood flow into turbulent.

1357. The following values are obtained on a sample of serum from a child that has clinical evidence of dehydration. Serum [Na]
= 135 mmol/L

rum [glucose] = 540 mg/dL Serum [urea nitrogen] = 56 mg/dL

e osmolality of serum is expected to be close to:

290 mOsm/kg H2O

300 mOsm/kg H2O

310 mOsm/ kg H2O

320 mOsm/kg H2O

1358. A human growth hormone variant (hGH-V) is expressed primarily in:

Liver

Spleen

Kidney

Placenta

1359. The term which is defined as the volume remaining in the lungs after normal exhalation and is calculated by adding the
expiratory reserve volume and the residual volume

functional residual volume

residual volume

tidal volume

vital capacity
1360. When you cut both vagus nerves breathing:

becomes rare and deep;

becomes superficial and frequent;

does not change;

is terminated.

1361. In the presence of vasopressin, most of the water filtered by the kidneys is reabsorbed in the:

proximal tubule

thin descending limb

thick ascending limb

collecting ducts

1362. Which of the following hormones has the highest mineralocorticoid activity?

Cortisol

Corticosterone

Deoxy corticosterone

Cortisone

1363. Which of the following statements regarding muscle spindle is FALSE?

It contains nuclear bag & nuclear chain fibers, which are responsible for dynamic & static responses respectively.

It is important as a feedback sensor to give information about muscle length

It has no contractile elements at all

Increased Ay motor efferent discharge to muscle spindle will lead to increased muscle spindle sensitivity to stretch.

1364. Worldwide, the commonest preventable cause of mental retardation in the newborn is:

Iodine deficiency

Phenylalanine hydroxylase deficiency

Cystic fibrosis

Urea cycle defect

1365. Which of the following exerts the greatest osmotic effect across capillaries on a mole-mole basis?

Sodium

Chloride

Alb i
. +Albumin

Hydroxyethyl starch

1366. The composition of the internal environment includes the following liquids:

Blood, lymph, extracellular fluid;

Isotonic solution;

Blood plasma;

Digestive juices.

1367. The chronotropic effect is the effect on:

conductivity;

excitability;

automatism;

contractility.

1368. Which of the following is an amino acid that operates at inhibitory synapses in the brain?

acetylcholine

epinephrine

GABA

serotonin

1369. ICF volume does not change when dehydration is:

Isotonic

Hypertonic

Hypotonic

1370. The water content of lean body mass is about:

30 ml/100 g

50 ml/100 g

70 ml/100 g

highly variable

1371. Which hormone also upregulates its receptors in the adrenal cortex?

Epinephrine
Angiotensin II

Cortisol

Aldosterone

1372. The methods of recording sound phenomena in the heart include

electrocardiography;

echocardiography;

ballistocardiography;

phonocardiography.

1373. With shallow and frequent breathing, exhaled air:

approaching the composition to atmospheric air;

approaching the composition of the alveolar air;

does not change its composition;

the amount of O2 does not change, CO2 increases.

1374. The residual volume is:

the volume of air inhaled and exhaled with quiet breathing;

the volume of air that can be exhaled during maximum exhalation after a normal inhalation and exhalation;

the volume of air that remains in the lungs after the maximum expiration;

the amount of air that can be inhaled during maximum inspiration

1375. Insulin secretion from B-cells in pancreas is inhibited by:

Acetylcholine

Activation of beta-adrenergic receptors

Activation of alpha-adrenergic receptors and Somatostatin

Glucagon

1376. The amount of blood in the body:

6 - 8% of body weight;

1 - 2% of body weight;

8 to 10 liters;

1 to 2 liters.
1377. Macrocytes have a mean corpuscular volume greater than:

70 fL

80 fL

90 fL

100 fL

1378. Hypochlordria is a condition in which acid secretion by the stomach is greatly reduced. How does this condition affect the
digestion and absorption of food?

Digestion and absorption of all food substances are nearly normal

Digestion and absorption of all foods substances are markedly reduced

Digestion and absorption of carbohydrates are greatly affected by this condition

Digestion and absorption of fats are greatly affected by this condition

1379. Kidneys produce:

Erythropoietin

ADH

Angiotensin II

ANP

Cholecalciferol

1380. Which of the following is true of mass movements?

They are a modified type of peristalsis that occurs in the large intestine

They are strong peristaltic contractions of the small intestine in response to mucosal irritation

They normally move colonic contents from the cecum to the transverse colon

They normally occur approximately 9 to 12 times per minute

1381. Which of the following hormones lowers plasma level of ionized calcium?

Calcineurin

Calcitonin

Parathyroid hormone

PTH related peptide

1382. Hemolysis is called:

Destruction of red blood cells;


Destruction of leukocytes;

Blood clot formation;

The ratio of plasma and shaped elements.

1383. Summation of post synaptic potentials can occur at the:

axon hillock

axon terminal

cell body

dendritic arm of the neuron

1384. Insulin secretion from beta cells of pancreas in response to a glucose load is mediated by:

GLUT 1

GLUT 2

GLUT 3

GLUT 4

1385. How is the resting membrane potential maintained?

Potassium & sodium ions are moved against their concentration gradient, & this requires energy

The membrane at resting membrane potential is not permeable to any ions, thus no channels are open during this time.

The Na+/K+ ATPase pump ensures that no ions are moving in or out of the cell during resting membrane potential, to maintain
the concentration gradient

all of the above

1386. The most abundant cation in ICF is

Sodium

Potassium

Magnesium

Calcium

1387. In postnatal life, thyroid hormones do not increase oxygen consumption in the:

Heart

Skeletal muscle

Adult brain

Liver
Liver

1388. In the stretch reflex, the afferents are

I a fibers

I b fibers

A delta fibers

none of the above

1389. Which of the following is not excitory neurotransmitters?

acetylcholine

epinaphrine

GABA

glutamate

1390. Factor V Leiden:

Is a mutated form of factor IX

Is inactivated by protein C

Is present in a large subset of patients with venous thromboembolism

all of the above

1391. Which of the following does not occur as a consequence of adrenalectomy?

Reduced alertness

Increased vulnerability to hypotension

Decreased ability to excrete a water load

Glucose intolerance

1392. In a healthy adult, ECF volume constitutes what fraction of body weight?

10%

20%

30%

40%

1393. The prokinetic effects of erythromycin are due to agonist actions on:

histamine receptors
motilin receptors

CCk receptors

somatostatin receptors

1394. Check all correct statements.

The beta cell releases equimolar amounts of insulin and C-peptide.

Nonsuppressible insulin like activity (NSILA) is due to circulating C-peptide.

C-peptide has a shorter half-life compared to insulin.

none of the above

1395. ADH circulates in plasma primarily:

Bound to neurophysin I

Bound to neurophysin II

Bound to plasma albumin

Not bound to plasma proteins

1396. The time of a complete blood circulation in an adult person is on average equal to:

5 - 10 sec.

10 - 15 sec.

15 - 20 sec.

23 - 25 sec.

1397. Renal blood flow:

Greater per unit mass than cerebral blood flow

Is greater in the medulla compared to the cortex

Is closely related to tubular sodium reabsorption

Only sympathetically mediated

1398. How does gastrin stimulate hydrochloric acid secretion by the parietal cells of the gastric gland?

Directly by acting on a gastrin receptor located on the parietal cell

Indirectly through a cyclic adenosine monophosphate (cAMP)- mediated pathway

Indirectly through a cyclic guanosine monophosphate (cGMP).- mediated pathway

Indirectly via stimulation of histamine release from enterochromaffin cells


1399. A known amount of heavy water and inulin are injected intravenously into a healthy 35 year old male. The volume of
distribution of heavy water and inulin were 36 liters and 12 liters respectively. Which of the following estimations is most likely?

Plasma volume is 6 liters.

Interstitial fluid volume is 6 liters.

ICF volume is 28 liters.

ECF volume is 12 liters.

1400. Indicate the main reflexogenic zones of respiratory-regulating chemoreceptors:

respiratory muscles;

pleural sheets;

small vessels of the alveoli;

carotid sinus and bulbar respiratory center.

1401. The major stimulator of parathyroid hormone release from the parathyroid gland is:

Vitamin D

A fall in plasma ionized calcium

Calcitonin

A fall in plasma phosphate concentration

1402. The release of gastrin from G cells in the antrum of the stomach is inhibited by:

activation of vagal efferent fibers to the stomach

circulating epinephrine

blood-borne calcium

spmatostatin

1403. What determines resting membrane potential?

electrochemical equilibrium

ionic conductance

movement of K

movement of Na

1404. Bicuspid valve is localized:

in the right atrioventricular opening

in the left atrioventricular opening


at the mouth of the aorta

at the mouth of the pulmonary trunk.

1405. The greatest resistance to blood flow is created in:

the aorta;

arterioles;

capillaries;

venules.

1406. During exercise, a person has tachypnea, because:

mechanoreceptors of the lungs are irritated;

respiratory muscle proprioceptors are irritated;

chemoreceptors of brain vessels are excited;

decreases CO2 in the blood.

1407. The total circulating bile salt pool is approximately:

35 mg

3.5g

150 mg

30 g

1408. The blood brain barrier acts to completely isolate the central nervous system from the external environment, in order to
protect the brain against possible toxins and infection. There are a couple of areas where the BBB is porous, and these are
located in

I, II

I, II, III, IV

III, IV

I, IV

1409. In HCl formation, Cl⁻ enters through the basolateral membrane of parietal cells in exchange for the exit of:

CO2

HCO3⁻

K⁺

Na⁺
1410. Select all correct answers. Which of the following increase insulin resistance?

Leptin

Adiponectin

Resistin

none of the above

1411. The most frequent type of movement in the small intestine in the digestive state is:

peristaltis

antiperistaltis

slow wave

segmentation contraction

1412. Acidosis is:

Shift the blood reaction in the acidic side;

Shift of blood reaction to the alkaline side;

Change in osmotic pressure;

Change of oncotic pressure.

1413. Amongst plasma proteins, albumin makes the greatest contribution to the colloid osmotic pressure of plasma proteins
because, relative to most plasma proteins, albumin:

Is the most abundant plasma protein

Has the longest half-life

Has the lowest molar mass

Exits the capillary fastest

1414. Red cell antigens A and B are chemically:

Ahospholipids

Glycosphingolipids

Glycopeptides

Polypeptides

1415. Which of the following statement regarding withdrawal reflex is INCORRECT?

It is characterized by flexion of the ipsilateral limb & extension of the contralateral limb when strong noxious stimuli are
applied to the skin.
Flexing pattern is constant despite the site of stimuli varying in the limb.

Withdrawal reflex is prepotent & excludes other reflex activity from taking place.

Increased strength of stimuli will increase the number of motor units being recruited & increase the duration of response.

1416. Concentration and dilution of urine occurs in:

Bowman-Shimlyansky capsule;

loops of Henle and collecting tubules;

only in the collecting tubules of the nephrons;

only in the proximal tubules of the nephrons.

1417. Which of the following statements regarding referred pain is INCORRECT?

Superficial somatic pain is sometimes referred.

Referred pain usually occurs to a structure from the same dermatomal distribution.

Referred pain could be due to convergence of peripheral nerve fiber onto spinothalamic tract, leading to misinterpretation of
the site of pain by the brain.

Maxillary sinus pain is usually referred to the teeth.

1418. The low pressure zone of the cardiovascular system does not include:

the capillaries of the systemic blood circulation;

the capillaries of the pulmonary circulation;

right atrium, left atrium;

arteries of the systemic circulation.

1419. What stimulate the ileocecal sphincter to open?

Change in pH

Distention

Pelvic Nerve

Vagus nerve

1420. Which of the following is an example of monosynaptic reflex?

Stretch reflex

Abdominal reflex

Baroreceptor reflex

All of the above


1421. Glomerular filtration rate (GFR):

Is independent of the size of the capillary bed

Depends only on the hydrostatic and osmotic pressure differences across the capillary

Is determined by the same forces governing filtration across all other capillaries

Depends only on the permeability of the capillary

Requires active transport

1422. The term “brain of the gut” is used to refer to the:

autonomic ganglia

enteric nervous system

migratory motor complex

interstitial cells of Cajal

1423. Which of the following is inhibitory neurotransmitter?

Acetylcholine

Aspartate

Gallamine

Glycine

1424. In the nephrons of the kidneys are filtered:

blood cells;

large-scale plasma proteins;

only metabolites;

low molecular weight components of plasma and water.

1425. Which of the following is inactivated in the lung:

Angiotensin II

Angiotensin I

Bradykinin

Vasopressin

Noradrenaline

1426. Which of the following contributes least to the osmolality of plasma?


Glucose

Proteins

Sodium

Urea

1427. In the descending parts of the loop of Henle nephrons occurs:

dilution of urine;

urine concentration;

the formation of ultrafiltrate;

secretion of water and salts.

1428. Which of the following statement is false?

Decay is determined by the length constant

Electrical potential works if all ion channels open

ENa is the energy associated with the sodium gradient - there is a different polarity & direction

The concentration of a chemical gradient can be represented in terms of energy: delta miu = RT ln ( conc. inside / conc.
outside)

1429. Sympathetic cholinergic nerves innervate the vessels:

skeletal muscle and sweat glands;

the gastrointestinal tract;

the spinal cord and brain;

the heart and lungs.

1430. The group of chemical messengers whose actions are known to be mediated by receptor tyrosine kinases includes:

Angiotensin II, ANP and ET 1

EDRF, ANP and AVP

FSH, LH and GHRH

Insulin, EGF, IGF 1, PDGF

1431. Which of the following is the most potent stimulator of pancreatic secretion?

Cholecystokinin

Histamine

Secretin
Serotonin

1432. Injection of pharmacologic doses of ACTH in healthy experimental animals is least likely to produce a sustained increase
in:

Cortisol secretion

Lymphocyte count

Skin pigmentation

Aldosterone secretion

1433. Which of the following substances is released from the mucosa of the duodenum in response to acidic gastric juice?

Cholecystokinin

Gastric inhibitory peptide

Secretin

Substance P

1434. The following values are obtained on a sample of serum from a child that has clinical evidence of dehydration. Serum [Na]
= 130 mmol/L

rum [glucose] = 540 mg/dL Serum [urea nitrogen] = 56 mg/dL

suming there are no toxins in ECF, the effective serum osmolality is approximately:

290 mOsm/kg H2O

300 mOsm/kg H2O

310 mOsm/ kg H2O

320 mOsm/kg H2O

1435. The adrenal medulla does not normally secrete:

Epinephrine

Norepinephrine

Chromogranin B

Vasoactive intestinal polypeptide

1436. A high level of sympathetic stimulation is most likely to cause which of the following effects on gastrointestinal function?

Decrease in stomach Ph

Decrease motility

Increase in gastric acid secretion

I i i
Increase in mucous secretion

1437. Esophagus at rest is:

Open at the bottom

Open at the top and the bottom

Closed at the top and the bottom

1438. The mitotic spindle is made up of a protein called:

Tubulin

Caveolin

Connexin

Annexin

1439. Which of the following has the longest biologic half-life?

Insulin

Angiotensin II

Glucagon

Thyroxine

1440. In violation of the links between the medulla oblongata and the pons:

breathing stops completely;

abdominal breathing persists;

breathing does not change;

the smoothness and rhythm of breathing is disturbed.

1441. Which of the following retinal neurons generate 'all or none' action potentials?

Amacrine cell

Bipolar cell

Ganglion cell

Horizontal cell

1442. Transport maximum is:

the amount of a solute filtered in comparison to the amount excreted in urine

the amount of a specific solute entering bowman's capsule


the maximum rate of shuttling of a solute from filtrate into the capillaries

the plasma concentration at which a solute will appear in urine

1443. Indicate the factor that increases the affinity of hemoglobin for oxygen:

acidosis;

alkalosis;

hyperthermia;

hypercapnia.

1444. 100 mg of sucrose is injected intravenously into an adult male weighing 70 kg. The plasma concentration of sucrose after
mixing is 0.01 mg/ml. If 5 mg of sucrose has been metabolized during this period, the ECF volume in this individual is
approximately:

6 liters

9.5 liters

14 liters

17.5 liters

1445. Which of the following enzymes protects the fetus from hyperthyroidism when the mother is hyperthyroid?

Type I deiodinase (5’ deiodinase); outer ring deiodinase

Type 2 deiodinase (3’ deiodinase)

Type 3 deiodinase (3,3,5 - deiodinase); inner ring deiodinase

Iodotyrosine deiodinase

1446. Which of the following is not a function of bile?

Emulsify fat

Excrete waste product

Facilitates fat digestion and absorption

None of them

1447. Of the following substances, the lipid bilayer per se (i.e., without proteins) is most permeable to:

Sodium ions

Urea

Glucose

Water
1448. Which of the following produces a mixed serous/mucous saliva?

Intrinsic salivary glands

Parotid glands

Sublingual glands

Submandibular glands

1449. Specify a property that is not characteristic of the pleural cavity:

tightness;

the presence of serous fluid;

lack of air;

the presence of air.

1450. Water filtration by the kidney:

Is 180 l/hr

Is 125 ml/min

Up to 90% is reabsorbed

Most drugs have MW less than 600 and are freely filtered

1451. Pulmonary vascular resistance:

Is minimal at FRC

Increases/decreases with increase in lung volume

Increases with elevated CVP

1452. The respiratory minute volume is the amount of air that:

ventilated through the lungs for 1 minute;

enters the lungs for 1 respiratory cycle;

the maximum you can exhale after a deep breath;

the maximum you can exhale after the usual inhalation.

1453. Which of the following stimulates the release of aldosterone from the adrenal cortex?

Renin

Angiotensin III

↓ plasma [K+]
ANP

1454. Sertoli cells produce:

Androgen-binding protein

Inhibin B

Mullerian inhibiting polypeptide

all of the above

1455. Which of the following substances can inhibit small intestinal motility?

Cholecystokinin

Gastrin

Insulin

Secretin

1456. The osmolar concentration of sodium in normal human plasma is approximately:

275 – 295 mOsm/L

135 – 145 mOsm/L

240 – 250 mOsm/L

95 – 110 mOsm/L

1457. The maximum pressure in the left ventricle is normally:

25–30 mm Hg. v .;

120–130 mm Hg. v .;

20–40 mm Hg. v .;

160–180 mm Hg. Art.

1458. Resistance to renal blood flow is chiefly determined by:

Renal artery

Afferent & efferent arterioles

Interlobular & arcuate arteries

Peritubular capillaries

1459. The body fluid compartment that contains more osmotically active particles (in relation to other fluid compartments in
the same individual) is:

Intracellular fluid
Intracellular fluid

Plasma

Interstitial fluid

all of the above

1460. Which biologically active substances are partially degraded by the lung?

Surfactant

Histamine

Angiotensin

Noradrenaline

1461. I tone double valve is heard:

in the 2nd intercostal space at the left edge of the sternum;

in the 2nd intercostal space at the right edge of the sternum;

in the 5th intercostal space, 1.5 cm medially from the mid-clavicule line;

1462. The pattern of electrical and motor activity in the gastrointestinal tract during periods of fasting is called:

basic electrical rhythm

migrating motor complex

peristaltis

segmentation

1463. Which of the following is a ‘negative’ acute phase reactant?

Albumin

C-reactive protein

α2 macroglobulin

Transferrin

1464. Which of the following is false regarding the transmission of the action potential?

There are potassium channels located at each Node of Ranvier

Myelination increases the speed of transmission exponentially

Dendrites are important contributors to the action potential

The axon hillock is a Nissl free area


1465. The mitochondrial genome is absent from:

Sperm cells , mature red blood cells

Ovum

White blood cells

all of the above

1466. The term oncotic pressure is applied to osmotic pressure exerted by:

Albumin and other plasma proteins across the capillary wall

Crystalloids across the cell membrane

Hemoglobin across the capillary wall

Substances such as urea, lactate, glucose across the cell membrane

1467. Select all correct answers. Mutations in which of the following have been implicated in the pathogenesis of
hypercoagulable states?

Protein C

Protein S

Factor V

all of the above

1468. Blood viscosity is an interaction:

Erythrocytes with plasma salts;

Blood cells and protein between each other;

Cells of the vascular endothelium;

Acidity and bases in the blood plasma.

1469. In clinical practice, the urinary clearance of which substance is most frequently used to estimate GFR?

Inulin

Creatinine

Urea

Mannitol

1470. The normal basal metabolic rate in a euthyroid adult male is (BSA is body surface area):

10-15 kcal/m2BSA/hr

20-25 kcal/m2BSA/hr
35-40 kcal/m2BSA/hr

45-60 kcal/m2BSA/hr

1471. CFTR is found on the ____ surface of cells, allowing Cl⁻ to enter the ____.

apical, cytoplasm

apical, lumen

basolateral, cytoplasm

basolateral, ECF

1472. Which of the following is a normal characteristic of lung?

3,000,000 alveoli

Alveolar diameter 3 mm

External surface area: 10 m2

Alveolar surface area: 5 to 10 m2

None of the above

1473. Which type of sensation is most affected in the lesions in sensory cortex?

Position & fine touch

Temperature differentiation

Pain

Gross touch

1474. Inhibitory postsynaptic potentials are example of:

postsynaptic inhibition

presynaptic inhibition

indirect inhibition

none of the above

1475. Check all correct statements regarding oxytocin.

It is essential for milk ejection

It increases uterine contractility

It facilitates ejaculation of semen

All of the above


1476. The stopcocks of the circulation are:

arterioles

capillaries

valves

venules

1477. Select the correct options for Cholecystokinin (CCK).

It decreases stomach motility.

Its release is inhibited by unprocessed protein.

Its secreted by enterochromaffin cells.

all of the above

1478. A 55-year-old male accident victim in the ED urgently requires a transfusion. His blood group could not be determined as
his red cell group and plasma group did not match. Emergency transfusion should be done with:

RBC corresponding to his red cell group and colloids and crystalloids

Whole blood corresponding to his plasma group.

O positive RBC, colloids and crystalloids

AB negative blood

1479. Regarding nerve fibres, which statement is incorrect?

Action potentials are generated at the initial segment in spinal motor neurons

Membrane is more permeable to potassium than sodium at rest

Resting membrane potential is about -90mV

Sodium channels are highly concentrated at the nodes of ranvier

1480. Mandatory reabsorption in the kidneys is the process of transition of water and all substances valuable to the body, which
occur:

in the proximal tubule of the nephron;

from the blood to the renal tubules;

from the blood into the loop of Henle;

in the Bowman - Shumlyansky capsule.

1481. Which of the following markers administered intravenously distributes exclusively in intracellular fluid?

Evans blue dye


Heavy water

Sucrose

None of the above

1482. Which of the following statements about synaptic potentials is incorrect?

they are propagated down the postsynaptic neuron

they undergo spatiotemporal summation

they are analogous to generator potentials and end-plate potentials

IPSP hyperpolarize the postsynaptic neuron

1483. The cardiocycle begins with:

ventricular systole;

a general pause;

atrial systole;

atrial diastoles.

1484. Which structure not involved in control and processing of conscious movement?

Medial lemniscus

Basal Ganglia

Cerebellum

Primary Motor Cortex

1485. The processing of short term memory to long term memory is done in:

Prefrontal cortex

Hippocampus

Neocortex

Amygdala

1486. Hair cells in organ of Corti are

Mechanoreceptors

Chemoreceptors

Neither

Both A & B are correct


1487. Motor neurons in the myentric plexus that stimulate the contraction of smooth muscle in the gastrointestinal tract
release:

acetylcholine

vasoactive intestinal polypeptide

nitric oxide

somatostatin

1488. Bone resorption is induced by abnormally elevated levels of:

IGF-1

Insulin

Estrogens

Glucocorticoids

1489. The receptor of stretch reflex is the:

muscle spindle

Golgi tendon organ

Pacinian corpuscule

all of the above

1490. Breathing capacity is:

the volume of air inhaled and exhaled with quiet breathing;

the volume of air that can be exhaled during maximum exhalation after a normal inhalation and exhalation;

the volume of air that remains in the lungs after the maximum expiration;

the volume of air that can be inhaled with the maximum inhale after the usual inhalation.

1491. The pressure in the pleural cavity during normal inspiration is:

6, - 9 mm Hg. v .;

3, - 4 mm Hg. v .;

15, - 20 mm Hg. v .;

1, 0 mm Hg. v.

1492. Which of the following regarding touch sensation is INCORRECT?

Touch sensation in the spino-thalamic tract is more concerned with gross tactile localization.

Touch sensation in dorsal column is more concerned with localization, spatial & temporal pattern of touch.
Touch sensation is transmitted by C fibers.

All of the above

1493. Respiratory rate in an adult at rest equals:

5–10 / min;

18-20 / min;

25–30 / min;

30–40 / min.

1494. Which of the following inhibits gastric acid secretion by an action on the parietal cell?

acetylcholine

epinephrine

prostaglandine E

histamine

1495. What is the major / principle hormone that stimulates HCl release?

CCK

Gastrin

Intrinsic Factor

Secretin

1496. Which of the following is notsynthesized in the liver?

IgG

α2 macroglobulin

Albumin

Angiotensinogen

1497. Which of these is not a sensory receptor classification?

Adrenoreceptor

Mechanoreceptor

Nociceptor

Photoreceptor
1498. The proenzyme pepsinogen is secreted mainly from which of the following structures?

Motility of the gut

Secretion of the gut

Sphincter tone

Stomach pH

1499. What happens during hyperpolarization?

Membrane potential (Em) goes to the equilibrium potential for potassium ions (Ek)

Membrane potential (Em) goes to the equilibrium potential for sodium ions (ENa)

Sodium permeability increases

all of the above

1500. Intercalated disks are ____ found in _____.

desmosomes, cardiac muscle

desmosomes, skeletal muscle

gap junctions, cardiac muscle

gap junctions, skeletal muscle

1501. JAK-STAT pathways mediate the effects of:

Transducin

Aquaporins

Gusducins

Growth hormone

1502. Which of these statements is false?

Most of the total blood volume is contained in veins.

Exchanges between blood and tissue fluid occur across the walls of venules.

Capillaries have a greater total surface area than any other type of vessel.

Small arteries and arterioles present great resistance to blood flow.

1503. An increase in plasma osmolarity stimulates release of what hormone from the posterior pituitary?

Renin
Antidiuretic hormone (ADH)

Aldosterone

Angiotensin II

1504. Bulk flow - ?

related to concentration gradient

related to permeability coefficient

depends on hydrostatic and oncotic pressure

1505. The Fick principle states that

Oxygen uptake as gas is equal to the arterio-venous oxygen difference in flow through the lungs

Arterio-venous oxygen difference in the brain multiplied by flow equals oxygen uptake

Arterial baroreceptor activation produces bradycardia

None of the above

1506. The systemic circulation begins:

aorta

pulmonary trunk

pulmonary veins

SVC

1507. What is the definition of cardiac cycle?

The contraction of the atria

Circulation of the blood in the heart

The contraction and relaxation of the ventricles

It is a sequence of event that occurs during one complete heartbeat

1508. The action potential of the cardiac muscle is characterized by the presence of plateau which:

in the ventricular muscle it reached 100 m sec

in the artial muscle, it reached 300 m sec

it result from a balance between Ca++ influx and K+ efflux

it result from an inflow of Cl- and outflow of bicarbonate ions

1509. All cardiac valves are normally closed during which of the following phases of the cardiac cycle?
atrial contraction

systolic ejection

isovolumetric relaxation

ventricular filling

1510. The plateau phase of the cardiac action potential is due to the:

opening of voltage-gated Ca2+ channels

opening of fast voltage-gated Na+ channels

opening of voltage-gated K+ channels

1511. The longest phase in cardiac cycle is:

maximum ejection phase.

reduced filling phase.

maximum filling phase.

protodiastolic phase.

1512. Elevated circulating levels of aldosterone causes the kidneys to reabsorb more sodium and water, which increases blood
volume and arterial pressure.

True

False

1513. Blood pressure human is measured by the

Electro-cardiogram

Stethoscope

Sphygmomanometer

Barometer

1514. Which of the following statement(s) is(are) true

Laminar flow is characterized by concentric layers of blood moving in parallel down the length of a blood vessel.

Visceral pericardium and parietal pericardium secretes protein-rich fluid which fills the space between layers

Turbulent flow is characterized by concentric layers of blood moving in parallel down the length of a blood vessel.

AV node – the pacemaker of the heart, located in the posterior wall of the right atrium

1515. Parasympathetic stimulation of the heart has the following effect(s):


Negative chronotropic effect

Positive chronotropic effect

Negative inotropic effect

Positive inotropic effect

1516. Which types of blood is carried by the blood vessel marked 2?

deoxygenated

oxygenated

1517. Starling's law of the heart

Does not operate in the failing heart

Does not operate during exercise

Explains the increase in cardiac output that occurs when venous return is increased

Explains the increase in cardiac output when the sympathetic nerves supplying the heart are stimulated

1518. The left ventricle has a thicker wall than the right ventricle because :

it is richer in blood supply

it ejects blood through a narrower orifice

it ejects a greater cardiac output .

it ejects blood against a higher pressure

it contracts at a higher rate

1519. What is the function of the heart:

blood depot;

blood pumping

hematopoiesis

1520. The long-term mechanism for regulation of blood pressure involves regulating which of the following?

Vessel diameter

Blood volume

Heart rate

Contractility

1521. Which of the following is not true for ventricular systole?


The ventricles relax

The ventricles contract

The semi lunar valves close

The atrioventricular valves open

1522. What cells in the kidney monitor blood pressure, releasing renin when blood pressure is low?

Cortical cells

Angiotensinogen cells

Juxtaglomerular cells

All of the above

1523. What view of the heart do leads V1 and V2 represent?

Septal

Inferior

Anterior

Lateral

1524. The volume of blood each ventricle pumps out during a cardiac cycle is about ___________

70 ml

700 ml

156 ml

35 ml

1525. Autorhythmicity has:

myocardial contractile cell

pericardium

myocardium conductive cell

endocardium

1526. The absolute refractory period in the ventricle :

increases during sympathetic stimulation

coincides with the phase of rapid depolarization and the plateau

corresponds with the whole duration of action potential


it is shorter than the ARP of skeletal muscle

1527. The cardiac events that occur from the beginning of one heart beat to the beginning of the next heart beat are called the
cardiac cycle. During the cardiac cycle:

The atria contract before the ventricles.

The atria and the ventricles contract simultaneously.

The ventricles contract before the atria.

1528. Which of the following statement(s) is(are) correct

Diastolic blood pressure (SBP) represents the pressure in the blood vessels when the heart contracts (systole)

Diastolic blood pressure (DBP) represents the pressure in the blood vessels between heartbeats (diastole)

Pulse pressure is calculated by subtracting SBP from DBP

Systolic blood pressure (SBP) represents the pressure in the blood vessels when the heart contracts (systole)

1529. Minimum blood Pressure is in:

Aorta

Arteries

Capillaries

Venules

1530. The volume of blood pumped per minute by the left ventricle is

greater than the volume pumped by the right ventricle.

less than the volume pumped by the right ventricle.

is the stroke volume

1531. Semilunar valves are located in:

the mouth of the aorta

the mouth of the vena cava

the mouth of the pulmonary veins

between the atria and ventricles

1532. The pressure drop in the systemic circulation is greatest along which types of vessels?

aorta

arterioles

l di t ib ti l
large distributing vessels

capillaries

1533. Both atrioventricular valves , aortic pulnionary valve are all closed in

Atrial systole

Ventricular ejection

Atrial diastole

Isometric ventricular relaxation

1534. Which ONE of the following is true:

Right atrial systole and left atrial systole occur at same time

Pulmonary valve closes before aortic in inspiration

RV ejection precedes LV ejection

c wave of atrial pressure trace occurs at time of peak aortic pressure

1535. Which of the following receptor types are located on blood vessels and initiate smooth muscle contraction when its
associated autonomic neurotransmitter binds to the receptor?

alpha-1

beta-1

beta-2

muscarinic type 2

1536. The velocity of blood flow decreases if

vessel radius increases.

blood pressure increases.

viscosity increases.

afterload increases.

1537. Arterial systolic pressure occurs during left ventricular ejection, whereas arterial diastolic pressure occurs just before left
ventricular ejection.

True

False

1538. Resistance is determined by the ratio of perfusion pressure divided by blood flow.

true
false

1539. All arteries in the body contain oxygen-rich blood with the exception of

the aorta

the pulmonary artery

the renal artery

coronary sinus

1540. Which of the following statements is (are) false

The RAAS system starts with renin, a hormone released from granular cells in the hippocampus

The RAAS system starts with renin, a hormone released from granular cells in the juxtaglomerular apparatus

Angiotensin II causes vasoconstriction in the systemic circulation

Antidiuretic hormone is made by cell bodies located in the hypothalamus and released from the adjacent posterior pituitary.

1541. A change in blood pressure that decreases the impulses to the cardiovascular center results in increased impulses from
which branch of the autonomic nervous system? What would be the effect on blood pressure (BP)?

Sympathetic; increased BP

Parasympathetic; increased BP

Sympathetic; decreased BP

Parasympathetic; decreased BP

1542. Regarding the S-A node :

sympathetic stimulation decreases the slope of the pace - maker potential .

cells within the S-A node act as heart pace maker because their membrane depolarized to threshold and initiate an action
potential

acetyl choline increases the slope of the pace - maker potential .

the pace - maker cells within the S-A node are neurons rather than myocytes

1543. How does angiotensin II raise blood pressure?

Directly by increasing vasoconstriction and indirectly by stimulating aldosterone release

Only by increasing sodium (and, thus, water) reabsorption in the kidney

Only by causing vasoconstriction

1544. The firing rate of SA nodal pacemaker action potentials is decreased by

hyperthyroidism
hypokalemia which increases the slope of phase 4

increased circulating concentrations of epinephrine

vagus nerve activation which decreases the slope of phase 4

1545. The conduction velocity of AV nodal action potentials is increased by

cellular hypoxia

decreasing the slope of phase 4 of the AV nodal cells

drugs that activate the vagus nerves to the heart

stimulation of sympathetic adrenergic nerves

1546. The initial depolarization during a cardiac muscle action potential results from

a large decrease in intracellular potassium concentration

a large increase in intracellular sodium concentration

an increase in the relative conductance of potassium

opening of fast sodium channels

1547. Starling's law of heart states that the force of ventricular contraction is directly related to the end diastolic volume

True

False

1548. Which of the following, by decreasing blood viscosity, can increase blood flow at a given perfusion pressure?

decreased temperature

decreased concentration of red blood cells

low flow states

1549. What is true regarding vein

All the veins carry deoxygenated blood

All the veins carry Oxygenated blood

They carry blood from organs towards the heart

1550. Blood flow across a normal heart valve normally generates only a very small pressure gradient because valve resistance
is very low.

True

False
1551. The capillaries of skeletal muscles are of the structural type called __ _____

sinusoidal

continuous

discontinuous

fenestrated

all of the above

1552. The volume of blood is greatest in:

Systemic capillaries

Large veins

Small arteries

The lungs

1553. During a typical cardiac cycle under resting conditions, approximately __________ of the volume of blood present in the
ventricles just before ventricular contraction (referred to as enddiastolic volume), enters the ventricles as a result of atrial
contraction.

10%

20%

44%

32%

1554. The maximal rate of pressure development within the left ventricle occurs

during isovolumetric contraction

during rapid ventricular ejection

at peak left ventricular systolic pressure

during rapid ventricular filling

1555. The carotid sinus baroreceptors are located on the internal carotid arteries and are innervated by the vagus nerve.

True

False

1556. Starling's law of heart states that increase in force of contraction is directly related to cardiac output

True

False
1557. The action potential of cardiac muscle differs from that of skeletal muscles in:

it is propagated more slowly .

it is shorter in duration

it has a higher amplitude

it has no plateau

all of the above

1558. A change in blood pressure that increases the impulses to the cardiovascular center results in increased impulses from
which branch of the autonomic nervous system? What would be the effect on blood pressure (BP)?

Parasympathetic; increased BP

Sympathetic; decreased BP

Sympathetic; increased BP

Parasympathetic; decreased BP

1559. The largest total cross-sectional area of blood vessels is found in this region along the vasculature:

Large arteries

Small arteries

Capillaries

Venules

1560. The main function of the capillaries is:

ensuring metabolism in organs and tissues

discharge of blood from arterioles into venules, bypassing capillaries

the transformation of intermittent blood flow in continuous

the transformation of the laminar flow of blood into a turbulent

1561. All the following are correct except :

the left atrial wall is about the same thickness as the right atrium

the left ventricle is separated from the left atrium by a bicuspid valve

Purkinje system allows the excitation of all ventricular muscle fibers to occur at nearly the same time

the right heart receives oxygenated blood from the four pulmonary veins

1562. Isometric ventricular contraction

Begins when semilunar valves open


Begins when semilunar valves open

Lasts for about 0.5 s

Is a phase where all four cardiac values closed. Causes a sharp increase in atrial pressure

1563. What is the normal duration of a QRS complex?

0.16 seconds (4 small squares)

0.04 seconds (1 small square)

0.12 seconds (3 small squares)

0.08 seconds (2 small squares)

1564. The tendency for blood flow to be turbulent is increased by?

increased viscosity

increased hematocrit

partial occlusion of a blood vessel

decreased velocity of blood flow

1565. Long term control of tissue blood flow includes:

Nitric oxide

Baroreceptor reflex

Change in tissue vascularity

None of the above

1566. The vessels that carry oxygenated blood back to the heart are called

pulmonary arteries

pulmonary veins

aorta

superior and inferior venae cavae

1567. Aorta and arteries are:

vessels of elastic type

muscle vessels

muscle and elastic vessels

armless vessels
1568. Closure of the aortic and pulmonic valves produces which heart sound?

S1

S2

S3

S4

1569. During the cardiac cycle, opening of the atrioventricular valves following ventricular relaxation leads to:

an increase in the hydrostatic pressure within the ventricles

a decrease in the hydrostatic pressure within the atria

an increase in the hydrostatic pressure within the atria

1570. The role of the pericardium:

forms valves

softens friction

generates pulses

provides a reduction of chambers in the heart

1571. n a typical human cardiac cycle, the volume of blood remaining in the left ventricle at the end of ventricular systole is
approximately __________. This is referred to as the end-systolic volume.

10 mL

25 mL

100 mL

50 mL

1572. Heartbeat is accelerated by

Sympathetic nerves and acetylcholine

Cranial nerves and adrenaline

Cranial nerves and acetylcholine

Sympathetic nerves and epinephrine

1573. During a typical cardiac cycle under resting conditions, approximately __________ of the volume of blood present in the
ventricles just before ventricular contraction (referred to as enddiastolic volume), enters the ventricles as a result of atrial
contraction.

10%

20%
80%

50%

1574. The SA node is the normal pacemaker because of its :

rate of impulse discharge

location in the atrium

neural control

relative position to the A.V node

1575. A sudden fall in arterial blood pressure as occurs when a person rapidly stands up causes

increased firing of the carotid sinus baroreceptors

increased stimulation of the nucleus tractus solitarius

increased sympathetic adrenergic efferent activity

decreased firing of carotid body receptors

1576. In human being the duration of cardiac cycle is ___________

0,008 sec

0,8 sec

0,037 sec

3 sec

1577. The main cardiovascular (vital) center is located at:

medulla oblongata

hippocampus

spinal cord

cerebral cortex

1578. Cardiac muscle is different from skeletal muscle because:

Fast Na+ channels

Slow Ca++ channels

Presence of actin

Presence of myosin
1579. Compared to laminar flow, what hemodynamic changes occur in a blood vessel when turbulent flow is present?

a greater perfusion pressure is required to drive a given flow

flow is increased at given perfusion pressure

resistance to flow decreases

the slope of the relationship between flow and perfusion pressure increases

1580. As blood flows away from the heart, the mean blood pressure has the greatest decline across which class of vessels?

large distributing arteries

veins

capillaries

arterioles

1581. The mitral valve normally opens at the beginning of diastolic filling because

left atrial contraction forces open the valve

left ventricular pressure falls below the left atrial pressure

the papillary muscles contract

the papillary muscles relax

1582. Properties of cardiac muscle include

Excitability

Automaticity

Contractility

Conductivity

All of the answers are true

1583. Cardiac output is determined by _______

Heart rate

Pulse pressure

Heart rate and stroke volume

blood flow velocity

1584. During the isovolumic relaxation (also referred to as isovolumetric relaxation) phase of the cardiac cycle:
The hydrostatic pressure in the left ventricle is lower than the hydrostatic pressure in the left atrium.

The hydrostatic pressure in the left ventricle is higher than the hydrostatic pressure in the left atrium.

The hydrostatic pressure in the left ventricle is higher than the hydrostatic pressure in the aorta.

The hydrostatic pressure in the left ventricle is higher than the hydrostatic pressure in the PV.

1585. The cardiac events that occur from the beginning of one heart beat to the beginning of the next heart beat are called the
cardiac cycle. Which of the following is/are not true?

The left atrium and right atrium contract nearly simultaneously.

The left ventricle and right ventricle contract nearly simultaneously.

The atria contract before the ventricles.

The ventricles contract before the atria.

1586. Normal rhythm of the heart originates from

cells within the atrioventricular node

cells within the sinoatrial node

ectopic pacemakers

sympathetic nerve activity

1587. Increased sympathetic nerve activity at the heart would cause:

An increase in heart rate only

An increase in both heart rate and contractility

A decrease in heart rate and contractility

An increase in contractility only

1588. Of the conductive tissues of the heart, only these cells can directly stimulate cardiac myocytes to contract:

Cells of the sinoatrial (SA) node

Fibers of the bundle of His

Fibers of the left and right bundle branches

1589. Using the Fick Principle, cardiac output can be calculated by

dividing the arterial-venous pressure difference by the systemic vascular resistance

dividing the whole body oxygen consumption by the arterial-venous oxygen content difference

multiplying the stroke volume and heart rate

multiplying the whole body oxygen consumption and the arterial-venous oxygen content difference
1590. The aortic pulse pressure increases with a person’s age. The increased pulse pressure most commonly results from

decreased aortic compliance

increased left ventricular stroke volume

reduced left ventricular ejection velocity

1591. Cardiac action potentials have a much longer action potential than both nerve and skeletal muscle action potentials.

True

False

1592. Ventricular systole is defined as the time between the

closing of the aortic valve and the opening of the mitral valve

closing of the mitral valve and the closing of the aortic valve

opening and closing of the aortic valve

opening of the aortic valve and the opening of the mitral valve

1593. When the left ventricular stroke volume is 40 mL/beat and the heart rate is 80 beats/minute, the cardiac output is

2 liters/minute

3.2 liters/minute

5 liters/minute

1594. An increase in arteriolar resistance, without a change in any other component of the cardiovascular system, will produce:

A decrease in total peripheral resistance (TPR)

An increase in capillary filtration

An increase in arterial pressure

A decrease in afterload

1595. An increase in blood volume and arterial pressure will occur if the kidneys excrete excessive amounts sodium and water.

True

False

1596. Sympathetic stimulation of the heart has the following effect(s):

Negative chronotropic effect

Positive chronotropic effect

Negative inotropic effect


All of the above

1597. The value of blood pressure depends on:

strength, heart rate

peripheral resistance

volume and viscosity of circulating blood

all of the above

1598. Which reflex is represented in the following picture?

Danini-Ashner reflex

Baroreceptor reflex

Exercise Pressor Reflex

1599. Which one of these is the effect of angiotensin II ?

Vasoconstriction of afferent artery.

Vasodilation of efferent artery.

Vasoconstriction of efferent artery.

Vasoconstriction of both efferent and afferent.

1600. The basis of the autorhythmicity of the heart is:

potassium hyperpolarization

resistant sodium depolarization

interaction of acetylcholine with M- cholinergic receptors

spontaneous slow diastolic depolarization

1601. The cardiac conduction system includes all of the following except

the SA node

chordae tendineae

Purkinje fibers

the AV node

1602. The expiratory and inspiratory parts of the respiratory center are located in:

the medulla;

the midbrain;
the midbrain;

the hypothalamus;

the spinal cord.

1603. Which of the following prevents the alveoli from collapsing?

inspiratory reserve volume

expiratory reserve volume

tidal volume

residual volume

1604. The major reasons of anemic hypoxia are all except

Reduced erythropoiesis

Bloodloss

Synthesis of abnormal hemoglobin

Carbon monoxide poisoning

Cyanide poisoning

1605. The volume of air that can be exhaled after normal exhalation is the

tidal volume

residual volume

inspiratory reserve volume

expiratory reserve volume

1606. Which of the following activates trypsin in the small intestine?

Cl⁻

Enterokinase

Kazal

NaHCO3

1607. The submandibular gland secretes

Lingual lipase

Lysozyme

Proline rich proteins

Salivary amylase
1608. The nose serves all the following functions EXCEPT:

as the initiator of the cough reflex

warming and humidifying the air

cleansing the air

as a passageway for air movement

1609. Segmental contractions within the GIT cause:

atony.

churning.

mixing.

peristalsis.

1610. Which of the following is true about the parotid gland?

It has a predominantly mucous secretion

It is innervated by the glossopharyngeal nerve (CN IX)

It is the smallest salivary gland

It produces the majority of saliva by volume

1611. The surface of the airways is lined with:

ciliated epithelium;

endothelium;

alveolocytes;

stratified keratinized epithelium.

1612. Anatomical dead space performs the following functions:

maintains the partial pressure of oxygen;

warms, moisturizes, cleans the air, includes protective reflexes;

regulates tidal volume;

regulates the composition of alveolar air.

1613. When the diaphragm and external intercostals muscles contract, which of the following actions does NOT occur?

air moves into the lung

the intrapleural pressure increases


t e t ap eu a p essu e c eases

the diaphragm moves inferiorly

the intrapulmonary pressure decreases

1614. Normal breath begins with a reduction:

internal intercostal muscles and muscles of the shoulder girdle;

muscles of the chest and back;

external intercostal muscles and diaphragm;

neck muscles and internal intercostal muscles.

1615. What is indicated by number 16?

Descending colon

Ascending colon

Sigmoid colon

Caecum

1616. Which of the body systems listed below cooperate to supply O2 to cells and eliminate CO2? 1) digestive system 2)
cardiovascular system 3) urinary system 4) respiratory system 5) endocrine system

3, 5

2, 4

1, 2

1, 2, 4

1617. Quantitively, the most important enzyme in digestion of fat is:

lingual lipase

gastric lipase

pancreatic lipase

lipoprotein lipase

1618. What is hypoxia?

Reduced supply of O2 at cellular level

Reduced supply of O2 at tissue level

Reduced supply of O2 to brain

Reduced supply of O2 due to excess of CO2


1619. Normally, most of the water in the GI lumen is absorbed from:

stomach

duodenum

jejunum

colon

1620. Passive exhalation occurs due to:

contraction of the external intercostal muscles and diaphragm;

relaxation of the external intercostal muscles and diaphragm;

contraction of the abdominal muscles;

contraction of the internal intercostal muscles.

1621. Respiratory control chemoreceptors are sensitive to:

hypocapnia, alkalosis;

hypercapnia, acidosis, hypoxemia;

hyperoxia, alkalosis;

somatostatin, ADH.

1622. Select correct statement(s)

Carbon dioxide is 24 times more soluble in blood than oxygen is

About 10% of the CO2 is carried as plasma bicarbonate.

Bicarbonate is formed in the red blood cell but it is carried in the plasma compartment.

Expiratory center is located in cerebral cortex and is responsible primarily for expiration.

1623. Respiratory arrest occurs when:

damage to the medulla oblongata;

transection of the brain stem in the region of the diencephalon;

transection of the brain stem above the pons;

transection of the brain stem below the pons.

1624. The amount of air inhaled or exhaled in one breath during related, quiet breathing is:

functional residual capacity

residual volume
tidal volume

total lung capacity

1625. Which of the following formulas is correct for inspiratory capacity?

TV+IRV

IRV+RV

IRV+ERV+TV

ERV+TV

1626. The functions of the pleural cavity does not apply:

reduction of friction during movements of the lungs;

participation in the biomechanics of respiration;

protective, restrictive;

excretion of volatile substances.

1627. Acinus is:

upper airway;

the system of small bronchi;

respiratory bronchioles, alveolar passages and alveoli;

a collection of alveoli and all airways.

1628. The term “brain of the gut” is used to refer to the:

autonomic ganglia

enteric nervous system

migratory motor complex

interstitial cells of Cajal

1629. Physiologically, the most important choleretic is:

bile salts

CCK

secretin

gastrin

1630. Which of the following is not a function of bile?


Emulsify fat

Excrete waste product

Facilitates fat digestion and absorption

None of them

1631. Which of the following can cause stagnant hypoxia?

Shock or heart failure

anaemia

cyanide poisoning

carbon monoxide poisoning

1632. Select wrong statement(s)

Most of the muscles of chewing are innervated by the motor branch of the fifth cranial nerve

Mixed saliva contains 99.4-99.5% of water; the remaining part is a dry residue. PH of mixed saliva is 3-6.

Lingual lipase, an enzyme that is activated by stomach acid and digests fat after the food is swallowed

Daily secretion of saliva normally ranges between 800 and 1500 milliliters

1633. Which of the following characterizes carbohydrate digestion?

It begins when food comes in contact with gastric juice

It begins when the food comes in contact with saliva

It begins with when food comes in contact with pancreatic secretions

It ends when starch has been converted to maltose

1634. What does the arrow show?

Residual volume, the volume of air remaining in the lungs after maximum forceful inspiration. Reference values for residual
volume are 2 to 3 L

Residual volume, the volume of air remaining in the lungs after maximum forceful expiration. Reference values for residual
volume are 1 to 1.2 L

RV, reserve volume, the amount of air that we expire forcefully. Reference values for residual volume are 1 to 1.2 L

Residual volume, the volume of air remaining in the lungs after maximum forceful expiration. Reference values for residual
volume are about 500-600mL

1635. Which of the following best characterizes the secretions of the small intestine?

Hypotonic and slightly acidic

Hypotonic and slightly alkaline


Isotonic and slightly acidic

Isotonic and slightly alkaline

1636. Which of the following does not break down proteins?

Carboxypeptidase

Chymotrypsin

Pepsin

None of the above

1637. Which of the following contains regulatory centers for the respiratory and circulatory systems

cerebrum

medulla oblongata

hyppocampus

thalamus

1638. What is the structure indicated by C letter?

Duct of intestinal gland

Arteriole

Lacteal

Venule

1639. The functions of the pleural cavity does not apply:

reduction of friction during movements of the lungs;

participation in the biomechanics of respiration;

protective, restrictive;

excretion of volatile substances.

1640. The flow of air into the alveoli during inhalation occurs due to:

the difference between blood pressure and the force of the elastic thrust of the lungs;

elastic thrust of the lungs;

the difference between atmospheric and intraalveolar pressure;

the difference between the partial pressure of O2 and CO2 in the alveolar air.
1641. The secretion of bile is important for the proper digestion of which of the following?

Complex carbohydrates

Lipids

Monosaccharides

Proteins

1642. The exchange of gases between blood and cells is called

pulmonary ventilation

internal respiration

external respiration

cellular respiration

1643. During internal and external respiration, gases move by

osmosis

active transport

diffusion

endocytosis

1644. ​During exercise, a person has tachypnea, because:

mechanoreceptors of the lungs are irritated;

respiratory muscle proprioceptors are irritated;

chemoreceptors of brain vessels are excited;

decreases CO2 in the blood.

1645. The pattern of electrical and motor activity in the gastrointestinal tract during periods of fasting is called:

basic electrical rhythm

migrating motor complex

peristaltis

segmentation

1646. Surfactant in the alveoli:

reduces the surface tension of the water film;


increases the permeability of the alveoli for gases;

creates an elastic craving of the lungs;

increases the surface tension of the water film.

1647. A decrease in volume leads to a(n) ________ pressure.

decrease in

zero

equalization of

increase in

1648. All of the followings true about HCl except

It activates pepsinogen into pepsin

It has bacteriolytic action

It causes acidity of the chyme

It provides acid medium for the action of enzymes

Is secreted by chief cells

1649. Select the correct statement(s) about mucosa of GIT

It lines the lumen

The epithelium is simple columnar in most of the digestive tract

Is a thicker layer of loose connective tissue containing blood vessels

Is composed of a thin layer of areolar tissue topped by a simple squamous mesothelium

1650. The major humoral mediator of pancreatic duct cell secretion of bicarbonate rich juice is:

secretin

CCK

somatostatin

all of the above

1651. O2 and CO2 are transported by:

red blood cells;

leukocytes;

platelets;
plasma proteins.

1652. Digestion of which of the following occurs almost entirely in the small intestine?

Fat

Fruits and vegetables

Protein

Starch

1653. In HCl formation, Cl⁻ enters through the basolateral membrane of parietal cells in exchange for the exit of:

CO2

HCO3⁻

K⁺

Na⁺

1654. When we inhale

alveolar pressure decreases and intrapleural pressure increases

both alveolar pressure and intrapleural pressure increase

both alveolar pressure and intrapleural pressure decrease

alveolar pressure increases and intrapleural pressure decreases

1655. The major humoral mediator of meal-stimulated enzyme secretion is:

secretin

CCK

GIP

gastrin

1656. What is the structure indicated by A letter?

Epithelium

Lacteal

Arteriole

Venule

1657. Select wrong statement(s)

The hepatopancreatic sphincter (sphincter of Oddi) regulates the flow of both bile and pancreatic juice from the ampulla into
the duodenum
the duodenum.

Lacteal are the vessels which absorb lipids and transport them into the bloodstream by lymphatic system

There are an estimated 100 microvilli per square millimeter of small intestine

The ileum is the shortest part of the small intestine

1658. Anemic hypoxia occurs when:

reducing the oxygen capacity of the blood;

impaired hemodynamics;

a decrease in pO2 in atmospheric air;

changes in the function of respiratory tissue enzymes.

1659. The pressure difference between the intra-alveolar and intrapleural pressures is called ________.

negative pressure

pulmonary pressure

atmospheric pressure

transpulmonary pressure

1660. Which of the following is not secreted by parietal cells?

Gastroferritin

HCl

Intrinsic factor

Pepsinogen

1661. Select the incorrect statement

Minute Volume (MV): Is normally about 6 L

Expiratory reserve volume (ERV): additional volume that can be inspired after a passive inspiration

Vital capacity (VC): maximal volume that can be expired after a maximal inspiration

Transpulmonary pressure is the difference between the intrapleural and intra-alveolar pressures, and it determines the size of
the lungs

Atmospheric pressure is the amount of force that is exerted by gases in the air surrounding any given surface, such as the
body.

1662. Which of the following is not typically found in saliva?

Bicarbonate

HCl
IgA

Lysozyme

1663. Surface tension of the alveolar fluid is reduced by the presence of

mucus

sebum

surfactant

water

1664. Which muscles are involved in inspiration?

Diaphragm

Diaphragm and external intercostal muscles and abdominal muscles

Diaphragm and External intercostal muscles

Diaphragm and Internal intercostal muscles

1665. Circulatory hypoxia occurs when:

reducing the oxygen capacity of the blood;

impaired hemodynamics;

reducing the partial pressure of O2 in the atmospheric

the air;

changes in the function of respiratory tissue enzymes.

1666. Gas flow decreases as ________ increases.

resistance

pressure

airway diameter

friction

1667. The primary chemical stimulus for breathing is the concentration of

carbon monoxide in the blood

carbon dioxide in the blood

oxygen in the blood

carbonic acid in the blood


1668. Enterocytes are replenished by miotically active undifferentiated cells located in:

Brunner’s glands

Crypts of Lieberkuhn

Peyer’s patches

gut associated lymphoid tissue

1669. Most oxygen in the blood is transported

as gas dissolved in plasma

as oxyhemoglobin

as carboxyhemoglobin

as bicarbonate

1670. The reason for the diffusion of gases from the alveolar air into the blood and back is:

the difference in partial pressure and tension between the alveolar air and blood;

close adherence of alveoli and capillaries;

active transport of O2 and CO2;

1671. Which is the most important cholagogue?

secretin

CCK

gastrin

GIP

1672. Which of the following processes does atmospheric pressure play a role in?

pulmonary ventilation

resistance

surface tension

production of pulmonary surfactant

1673. Regions of stomach include all except

Cardiac region

Fundus

Pylorus
Body

None of the above

1674. Which of the following describes a correct order of structures in the respiratory passageways?

pharynx, trachea, larynx, bronchi, bronchioles b. c. d.

larynx, pharynx, trachea, bronchioles, bronchi

trachea, pharynx, larynx, bronchi, bronchioles

pharynx, larynx, trachea, bronchi, bronchioles

1675. Select wrong statement(s)

Type I, appear in the empty stomach when the tone of the stomach is low. The type I contractions are single contractions and
each one lasts for 20 seconds.

Type III contractions Otherwise called incomplete tetanus

Hunger contractions are the peristaltic waves superimposed over the contractions of gastric smooth muscle as a whole.

Type II hunger contractions occur when the tonus of stomach is stronger. These contractions last for 10 minutes

1676. External breathing is:

the exchange of gases between the lungs and blood;

transport of gases by blood;

the exchange of gases between atmospheric and alveolar air;

the exchange of gases between the blood and tissues.

1677. Which of the following is a normal characteristic of lung?

3,000,000 alveoli

Alveolar diameter 3 mm

External surface area: 10 m2

Alveolar surface area: 5 to 10 m2

None of the above

1678. In the lungs

PCO2 in the alveoli is the same as that in the capillaries

PO2 in the alveoli is the same as that in the capillaries

PCO2 in the alveoli is higher than that in the capillaries

PCO2 in the alveoli is lower than that in the capillaries


1679. Respiratory rate in an adult at rest equals:

5–10 / min;

18-20 / min;

25–30 / min;

30–40 / min.

1680. Which of the following composes the majority of salivary volume?

Parotid gland

Small salivary glands

Sublingual gland

Submandibular gland

1681. Specify a property that is not characteristic of the pleural cavity:

tightness;

the presence of serous fluid;

lack of air;

the presence of air.

1682. Which pattern of motility hastens the transit of chyme in the small intestine in the digestive state?

mass action contraction

migrating motor complex

peristaltis

segmentation contraction

1683. Which of the following glands does not secrete the mucous component of saliva?

Buccal

Parotid

Sublingual

Submandibular

1684. Which of the following refers to minute ventilation?

the minimum volume that can be expired at rest

the volume of air breathed per minute


the volume of air that remains in collapsed lungs

all of the above

1685. Gas exchange between alveoli and blood is carried out in:

arteries of the pulmonary circulation;

veins of the pulmonary circulation;

the capillaries of the great circle of blood circulation;

the capillaries of the pulmonary circulation.

1686. Inhale is:

an active process of air flow into the lungs;

passive process of air flow into the lungs;

an active process of carbon dioxide in the lungs;

active process of removing carbon dioxide from the lungs.

1687. The pressure in the pleural cavity with deep expiration is:

6 - 9 mm Hg. v .;

3 –4 mm Hg. v .;

15 - 20 mm Hg. v .;

1, 0 mm Hg. v .;

1688. When inhaling dust or pungent odor, reflexes occur:

protective (coughing, sneezing);

Goering-Breuer;

from chemoreceptors of the vascular zones;

from the baroreceptors of the vascular reflexogenic zones.

1689. Select the correct statemen(s)

The ileum is the longest part of the small intestine

The ileum is the shortest part of the small intestine

Duodenum has 3 parts

Brunner’s glands are inhibited by sympathetic stimulation

1690. Breathing capacity is:


the volume of air inhaled and exhaled with quiet breathing;

the volume of air that can be exhaled during maximum exhalation after a normal inhalation and exhalation;

the volume of air that remains in the lungs after the maximum expiration;

the volume of air that can be inhaled with the maximum inhale after the usual inhalation.

1691. NaHCO3⁻ in the duodenum comes from:

acini

pancreas

epithelial ducts

stomach

1692. The functional dead space includes:

the alveoli, which are ventilated but not perfused (no blood flow);

alveoli, which are ventilated and perfused;

the trachea;

bronchi.

1693. Which of the following is absorbed in the intestine independent of sodium?

glucose

amino acids

galactose

fructose

1694. Which of these processes occurs in the mouth?

mechanical digestion

ingestion

chemical digestion

all of the above

1695. During swallowing, the glottis is covered by

false vocal cord

true vocal cord

epiglottis
Adam’s apple

1696. What does this picture represent?

Intestinal villi

Intestinal microvilli

Intestinal Lieberkuhn gland

Major papilla of duodenum

1697. Which of the following hormone is normally released by the stomach into the systemic circulation?

Vasoactive intestinal polypeptide

Gastrin

Pepsinogen

Secretin

1698. What is the following structure?

Pylorus

Ileum

Jejunum

Duodenum

1699. Esophagus at rest is:

Open at the top

Open at the bottom

Open at the top and the bottom

Closed at the top and the bottom

1700. The residual volume is:

the volume of air inhaled and exhaled with quiet breathing;

the volume of air that can be exhaled during maximum exhalation after a normal inhalation and exhalation;

the volume of air that remains in the lungs after the maximum expiration;

the amount of air that can be inhaled during maximum inspiration

1701. The anatomic dead space:

varies with minute ventilation


is typically 150ml

is alveolar minus the pathological dead space

all of the above

1702. Select the correct statement(s)

Intrinsic salivary glands secrete relatively small amounts of saliva at a fairly constant rate whether we are eating or not

The act of deglutition is separated into two phases: oral and esophageal

Chyme is the solid material produced by the oral digestion of food

The cardiac region of stomach is subdivided into a funnel-like antrum and a narrower pyloric canal.

1703. As blood enters the systemic capillaries

PO2 in the blood is the same as PCO2 in the tissues

PO2 in the blood is higher than that in the tissues

PO2 in the blood is lower than that in the tissues

PO2 in the blood is the same as that in the tissues

1704. Alveoli in healthy people do not stick together when falling, because:

they have a water film;

there are elastic fibers in the wall of the alveoli;

in the alveoli there is a surfactant that reduces surface tension;

pleural sheets have the ability to absorb water.

1705. What is the maximum of volume that can be inspired at rest after a normal inspiration?

expiratory reserve volume

functional residual capacity

inspiratory reserve volume

residual volume

1706. Which statement(s) is(are) wrong about liver

The liver is the largest gland in the body

Heparin, Urea, Bile juice are secreted in liver

Bile juice and pancreatic juice are secreted into the small intestine by separate ducts

Unused glucose is stored in liver in the form of glycogen


1707. Which statement(s) is(are) true about histotoxic hypoxia

It is caused by a decrease in the amount of hemoglobin available for binding of O2 so that the O2 content of the arterial blood
is abnormally low

If the tissues are unable to use oxygen brought to them by blood, even that results in hypoxia.

Cyanide poisoning is one of reasons go histotoxic hypoxia

All of the above

1708. Contraction of the external intercostal muscles causes which of the following to occur?

The thoracic cavity volume decreases.

The ribs and sternum move upward.

The rib cage is compressed.

The diaphragm moves downward.

1709. Which of the following are paired correctly?

Cholecystokinin-gallbladder relaxation

Cholecystokinin-relaxation of the sphincter of Oddi

Secretin- relaxation of the sphincter of Oddi

Secretin-stimulation of enzymatic release from the pancreatic acinar cells

1710. Select the correct statement(s)

The small intestine receives not only chyme from the stomach but also secretions from the liver and pancreas

The duct of Santorini enters the duodenum more distally than the duct of Wirsung.

Pancreatic juice increases acidity in intestine

Bile is secreted by gallbladder

Bile salts prevent the formation of gallstone by keeping the cholesterol and lecithin in solution

1711. What is indicated by number 13?

Common bile duct

Main pancreatic duct

Pancreas

No correct answer

1712. Which of the following is not an effector of respiratory system?

blood circulatory system


trapezius

diaphragm

intercostals

1713. Intrinsic factor is essential for absorption of what substance in the terminal ileum?

Bile Salts

Ca²⁺

Vitamin B12

Vitamin C

1714. During gastric emptying, chyme is released into the duodenum through the ________.

esophageal hiatus

pyloric antrum

pyloric canal

pyloric sphincter

1715. In violation of the links between the medulla oblongata and the pons:

breathing stops completely;

abdominal breathing persists;

breathing does not change;

the smoothness and rhythm of breathing is disturbed.

1716. What do chief cells of stomach secrete?

HCl

Pepsinogen

Pepsin

Factor of Castle

1717. Hypoxic hypoxia occurs when:

reducing the oxygen capacity of the blood;

a decrease in pO2 in atmospheric air;

changes in the functions of respiratory tissue enzymes;

reducing the number of red blood cells.


1718. The volume of food in the stomach has which of the following effects on stomach emptying?

A the volume if food in the stomach increases, the rate of emptying decreases

A the volume of food in the stomach decreases, the rate of emptying decreases

A the volume of food in the stomach increases, the rate of emptying increases

The volume of food in the stomach has little if any effect of the rate if stomach emptying

1719. Motor neurons in the myentric plexus that stimulate the contraction of smooth muscle in the gastrointestinal tract release

acetylcholine

vasoactive intestinal polypeptide

nitric oxide

somatostatin

1720. The most frequent type of movement in the small intestine in the digestive state is:

peristaltis

antiperistaltis

slow wave

segmentation contraction

1721. In the mountains, the magnitude of the atmospheric pressure and the gas composition of the air change as follows:

atmospheric pressure and% gas content in the atmosphere decrease;

atmospheric pressure increases and% composition of gases does not change;

atmospheric pressure decreases and the% composition of gases does not change;

atmospheric pressure and% composition of gases increase.

1722. Which of the following best characterizes the secretions of the large intestine?

Consist mostly of trapped bacteria from the crypts of Lieberkuhn

They are mostly mucus

They contain enzymes for the final digestion of food

They contain large quantities of bicarbonate ions

1723. The term which is defined as the volume remaining in the lungs after normal exhalation and is calculated by adding the
expiratory reserve volume and the residual volume

functional residual volume

residual volume
tidal volume

vital capacity

1724. Select the statement, which is correct order of layers of GIT wall from inner to outer

mucosa,muscularis,submucosa,serosa

submucosa, mucosa,serosa,muscularis

mucosa, submucosa,muscularis,serosa

serosa,muscularis,submucosa,mucosa

1725. What is the following structure?

Ileum

Caecum

Duodenum

Jajunum

1726. Bile salts are essential for absorption of fat because they:

increase surface tension

solubilize dietary lipids in micelles

are hydrophobic

contain cholesterol

1727. During exercise, a person has tachypnea, because:

mechanoreceptors of the lungs are irritated;

respiratory muscle proprioceptors are irritated;

chemoreceptors of brain vessels are excited;

decreases CO2 in the blood.

1728. Acinus is

upper airway;

the system of small bronchi;

respiratory bronchioles, alveolar passages and alveoli;

a collection of alveoli and all airways.


1729. Which of the following is not a function of the lung?

Condensation of water vapor from expired air

Cool down air

Metabolism of substances

All are functions of the lung

1730. Actions of gastrin all are except

It inhibits the secretion of pepsinogen and hydrochloric acid by the gastric glands

It increases the motility of stomach

It promotes growth of gastric mucosa

It causes secretion of pancreatic juice, which is rich in enzymes

It stimulates production of hormones by pancreas

1731. Gas exchange occurs in:

large bronchi;

the alveoli;

medium bronchi;

trachea

1732. The heart muscle is a ________ effector.

somatic

sympathetic

visceral

afferent

higher-order

1733. Identify the branch of biological science that deals with the study of how living organisms perform their vital functions.

genetics

physiology

embryology

cytology

anatomy
1734. The most abundant proteins in blood plasma are .....

globulins.

transport proteins.

albumins.

lipoproteins.

fibrinogens.

1735. The white matter of the spinal cord is mainly .....

unmyelinated axons.

neuroglia.

Schwann cells.

myelinated axons.

nodes of Ranvier.

1736. Homeostatic regulation usually involves a(n) ________ that detects a particular stimulus, and a(n) ________ that responds to
the stimulus by communicating with a(n) ________ whose activity has an effect on the same stimulus.

control center; effector; receptor

receiver; communicator; effector

receptor; control center; effector

effector; receiver; communicator

control center; receiver; effector

1737. When body temperature rises, a center in the brain initiates physiological changes to decrease the body temperature. This
is an example of .....

negative feedback.

positive feedback.

nonhomeostatic regulation.

diagnostic regulation.

fever.

1738. Plasma composes about ________ percent of whole blood and water composes ________ percent of the plasma volume.

55; 92

92; 7
92; 55

45; 55

50; 50

1739. A person's blood type is determined by the .....

size of the RBCs.

volume of the RBCs.

chemical composition of the hemoglobin.

presence of specific glycoproteins on the cell membrane.

shape of the RBCs.

1740. Spinal nerves are .....

purely sensory.

purely motor.

both sensory and motor.

interneuronal.

involuntary.

1741. The part of the peripheral nervous system that carries sensory information to the CNS is designated .....

motor.

afferent.

efferent.

autonomic.

somatic.

1742. A plasma protein essential for blood clotting is .....

albumin alpha.

immunoglobulin A.

lipoprotein C.

fibrinogen.

metalloprotein D.

1743. Which of the following is arranged in correct order from the most complex to the simplest?

ll l ti l l t i
cellular, tissue, molecular, organ system, organ, organism

molecular, cellular, tissue, organ, organ system, organism

tissue, cellular, molecular, organ, organ system, organism

organ, organism, molecular, cellular, tissue, organ system

organism, organ system, organ, tissue, cellular, molecular

1744. Parasympathetic functions include all of the following, except .....

decrease in the rate of cardiac contraction.

constriction of the pupils.

dilation of the airways.

stimulation of urination.

stimulation of defecation.

1745. The clumping of red blood cells, when the specific antibody against the antigen on the cells is added, is called .....

coagulation.

hemostasis.

agglutination.

areolation.

vascularization.

1746. Blood volume represents about ________ percent of a person's body weight.

15

25

42

1747. The ________ nervous system is composed of the brain and spinal cord.

autonomic

peripheral

central

efferent

afferent
1748. Erythropoiesis is stimulated when .....

oxygen levels in the blood increase.

blood flow to the kidney declines.

coagulation begins.

carbon dioxide levels in the blood decrease.

blood pressure increases.

1749. Which white blood cell is most effective against parasitic infections?

neutrophils

eosinophils

basophils

monoctyes

lymphocytes

1750. Antigens of the surface of red blood cells are also called ________ and antibodies in the blood plasma are also called
________.

agglutinogens; agglutinins

agglutinins; agglutinogens

T-cells; B-cells

erythrogens; antibiotics

serum; plasma

1751. Substances that can carry electrical current across cell membranes are called .....

electrons.

electrolytes.

nonelectrolytes.

osmoregulators.

buffers.

1752. Interactions between actin and myosin filaments of the sarcomere are responsible for .....

muscle fatigue.

the conduction of neural stimulation to the muscle fiber.

muscle contraction.
muscle relaxation.

the striped appearance of skeletal muscle.

1753. The medulla oblongata, pons, and midbrain comprise the .....

basal ganglia.

brain stem.

telencephalon.

ventricles.

diencephalon.

1754. The basic functional unit of the nervous system is the .....

neuron.

nerve.

axon.

glial cell.

receptor.

1755. When you perform a physical activity that you have not done in a while, such as riding a bike, which area of your brain
coordinates your movements with learned experiences?

thalamus

pons

midbrain

cerebellum

medulla oblongata

1756. The average life span of a red blood cell is .....

24 hours.

1 month.

4 months.

about 1 year.

many years.

1757. A person whose platelet count is 40,000/µl is suffering from .....

thrombocytosis.
leukocytosis.

hemocytosis.

thrombocytopenia.

leukopenia.

1758. Which pH is closest to normal body pH?

pH 7

pH 8

pH 4

pH 3

pH 2

1759. The most abundant class of neuron in the central nervous system is .....

anaxonic.

bipolar.

multipolar.

pseudopolar.

unipolar.

1760. The ________ nervous system carries impulses to skeletal muscles.

somatic

sympathetic

parasympathetic

afferent

autonomic

1761. Water molecules move across cells by .....

osmosis.

primary active transport.

secondary active transport.

activity of the sodium-potassium pump.

phagocytosis.
1762. The ________ provides the principal link between the nervous and endocrine systems.

medulla oblongata

cerebellum

pons

cerebrum

hypothalamus

1763. ________ involves a cascade of reactions leading to the conversion of fibrinogen to fibrin.

Vascular spasm

The platelet phase

Retraction

Coagulation

Fibrinolysis

1764. Type AB blood has which of the following characteristics?

RBCs have the Rh positive antigens and the anti-D plasma antibodies.

RBCs have no surface antigens and both anti-A and anti-B antibodies in the plasma.

RBCs have both the A & B surface antigens and no ABO plasma antibodies.

RBCs have the A and the B surface antigens and the plasma has anti-A and anti-B antibodies.

RBCs have the A antigen and the plasma has the anti-B antibody.

1765. Plasma is closest in composition to .....

urine.

isotonic saline solution.

sterile water.

interstitial fluid.

1766. The ________ division of the autonomic nervous system is said to function during "rest and digest."

sympathetic

parasympathetic

thoracolumbar

visceral

somatomotor
somatomotor

1767. Which of the following best describes the term sarcomere?

protein that accounts for elasticity of resting muscle

repeating unit of striated myofibrils

storage site for calcium ions

thin filaments are anchored here

largely made of myosin molecules

1768. Each heme ring in hemoglobin encloses an atom of .....

magnesium.

calcium.

iron.

sodium.

copper.

1769. General sense receptors are typically the .....

dendrites of motor neurons.

dendrites of sensory neurons.

axons of motor neurons.

axons of sensory neurons.

cell bodies of sensory neurons.

1770. The most numerous white blood cells in peripheral circulation are the .....

eosinophils.

basophils.

lymphocytes.

neutrophils.

monocytes.

1771. Which of these descriptions best matches the term B lymphocytes?

adhere to collagen beneath endothelium

helper cells are one type

develop into plasma cells and produce antibodies in response to antigens


kill bacteria using hydrogen peroxide

often elevated in allergic individuals

1772. The principal cation in intracellular fluid is .....

sodium.

potassium.

calcium.

magnesium.

chloride.

1773. The largest and most numerous of the glial cells in the central nervous system are the ....

satellite cells.

astrocytes.

oligodendrocytes.

microglia.

ependymal cells.

1774. The ________ division of the nervous system carries motor commands to muscles and glands.

spinal

peripheral

autonomic

afferent

efferent

1775. Red blood cell production is regulated by the hormone .....

thymosin.

angiotensin.

erythropoietin.

thymopoietin.

renin.

1776. The plasma membrane of a muscle fiber is called the ....

sarcolemma.
sarcomere.

sarcosome.

sarcoplasmic reticulum.

sarcoplasm.

1777. Each thin filament consists of .....

two actin protein strands coiled helically around each other.

chains of myosin molecules.

six molecules coiled into a helical structure.

a rod-shaped structure with "heads" projecting from each end.

a double strand of myosin molecules.

1778. Excess iron is stored in the liver and spleen as .....

hemosiderin and ferritin.

transferrin.

bilirubin and biliverdin.

hemoglobin.

urobilin and stercobilin.

1779. ________ are large phagocytic white cells that spend most of their time outside the blood as fixed and free phagocytic
cells.

Neutrophils

Eosinophils

Basophils

Lymphocytes

Monocytes

1780. The function of platelets is to assist in the .....

destruction of bacteria.

process called hemostasis.

removal of worn out red blood cells.

immune response during an infection.

transport of blood gases such as oxygen.


1781. The percent fraction of formed elements relative to whole blood is the .....

viscosity.

specific gravity.

packed volume.

hematocrit.

differential cell count.

1782. The chemoreceptors in the medulla are directly stimulated by

CO2 from the blood.

H+ from the blood.

H+ in cerebrospinal fluid that is derived from blood CO2.

decreased arterial PO2

1783. Auerbach's Plexus is also known as what?

Enteric plexus

Interstital cells of Cajal.

Myenteric plexus.

Submucosal plexus.

1784. Which of the following is not a function of the respiratory system?

transports O2 to the tissues.

contributes to maintenance of normal acid- base balance.

provides a route for heat and water elimination.

enables speech, singing, and other vocalization.

1785. Absorption of water in the small intestine occurs by which of the following?

Active transport

Couple transport with glucose

Passive diffusion

Solvent drag

1786. Intrinsic factor is essential for absorption of what substance in the terminal ileum?

Bile Salts
Ca²⁺

Vitamin B12

Vitamin C

1787. Sodium transport through the brush border of the intestinal epithelial cells occurs by which of the following mechanisms?

Active transport

Co-transport with hydrogen ions

Facilitated diffusion

Passive diffusion

1788. Which of the following mechanisms allows complete absorption of glucose from the intestinal lumen into the cytosol of
the enterocyte?

simple diffusion through GLUT-5

Na-glucose cotransporter-1

NA channels in enterocyte membrane

paracellular uptake via gap junctions

1789. Which of the following substances stimulates the motor activity of the stomach?

Cholecystokinin

Gastrin

Norepinephrine

Secretin

1790. In which of the following food substances is chewing essential for digestion?

Cheese

Eggs

Fruits and vegetables

Meat

1791. The amount of air inhaled or exhaled in one breath during related, quiet breathing is:

functional residual capacity

residual volume

tidal volume

total lung capacity


1792. Which of the following is true of the gastroenteric reflex?

It increases peristaltic activity of the stomach

It leads to defecation

It leads to increased peristaltic activity in the small intestine

It stimulates contractions of the colon

1793. Normal breath begins with a reduction:

internal intercostal muscles and muscles of the shoulder girdle;

muscles of the chest and back;

external intercostal muscles and diaphragm;

neck muscles and internal intercostal muscles.

1794. Bile salts are essential for absorption of fat because they:

solubilize dietary lipids in micelles

increase surface tension

are hydrophobic

contain cholesterol

1795. Which of the following are paired correctly?

Cholecystokinin-gallbladder relaxation

Cholecystokinin-relaxation of the sphincter of Oddi

Secretin- relaxation of the sphincter of Oddi

Secretin-stimulation of enzymatic release from the pancreatic acinar cells

1796. Which of the following factors play a role in the oxygen–hemoglobin saturation/dissociation curve?

temperature

pH

BPG

all of the above

1797. The value of normal Functional Residual Capacity is:

3800 ml

2200 ml
4800 ml

1200 ml

1798. What phase of gastric regulation includes inhibition by the enterogastric reflex?

the intestinal phase

the gastric phase

the buccal phase

the cephalic phase

the pharyngo-esophageal phase

1799. After ventilation of the alveoli with fresh air, what is the next step in respiratory process?

Diffusion of O2 from alveoli to pulmonary blood

Diffusion of CO2 from pulmonary blood to alveoli

Diffusion of O2 from pulmonary blood to alveoli

Both A and C

1800. What is hypoxia?

Reduced supply of O2 at cellular level

Reduced supply of O2 at tissue level

Reduced supply of O2 to brain

Reduced supply of O2 due to excess of CO2

1801. Postganglionic parasympathetic neurons innervating the gastrointestinal smooth muscle are located in:

myentric plexus

submucosal plexus

paravertebral ganglia

prevertebral ganglia

1802. Which is the correct formula of Henry’s law:

Concentration of dissolved gas = Partial pressure / Solubility coefficient

Concentration of dissolved gas = Solubility coefficient / Partial pressure

Partial pressure =Concentration of dissolved gas / Solubility coefficient

Partial pressure= Solubility coefficient/ Concentration of dissolved gas


1803. Which of the following is not a function of bile?

Emulsify fat

Excrete waste product

Facilitates fat digestion and absorption

None of them

1804. The major humoral mediator of gall bladder contraction in response fo a fat meal is:

CCK

gastrin

secretin

somatostatin

1805. NaHCO3⁻ in the duodenum comes from:

acini.

epithelial ducts.

paneth cells.

the pancreas.

1806. Which Hypoxia is related to the Hemoglobin dysfunctioning?

Hypoxic Hypoxia

Anemic Hypoxia

Histotoxic Hypoxia

Ischemic Hypoxia

1807. Which of the following best characterizes the secretions of the large intestine?

Consist mostly of trapped bacteria from the crypts of Lieberkuhn

They are mostly mucus

They contain enzymes for the final digestion of food

They contain large quantities of bicarbonate ions

1808. Which of the following type of motor activity in the small intestine is stimulated by local irritation of the intestinal
mucosa?

Haustration

Peristaltic rush
Secondary peristalsis

Segmentation movement

1809. Micelles in bile are formed by:

bile salts and phospholipids

bile acids and bile salts

cholesterol and bile salts

cholesterol and phospholipids

1810. What is blood brain barrier?

a barrier between blood and brain parts and is highly effective for O2 as it is the need of brain

a barrier of very tightly placed endothelial cells in the brain through which the transfer of O2 becomes very difficult and CO2
passes very easily

a highly lipid unsoluble barrier between brain and blood

a lipid and protein unilayered barrier between brain and blood

1811. Which of these values would normally be the highest?

tidal volume

inspiratory reserve volume

expiratory reserve volume

vital capacity

1812. During gastric emptying, chyme is released into the duodenum through the ________.

esophageal hiatus

pyloric antrum

pyloric canal

pyloric sphincter

1813. Compare the partial pressures of the O2 ad CO2 gases both in the arterial and venous systems:

On arterial side O2 has 40mmHg and CO2 has 97mmHg; On venous side O2 has 46mmHg and CO2 has 40mmHg

On arterial side O2 has 60mmHg and CO2 has 90mmHg; On venous side O2 has 52mmHg and CO2 has 48mmHg

On arterial side O2 has 78.5mmHg and CO2 has 29.7mmHg; On venous side O2 has 35.4mmHg and CO2 has 82.3mmHg

On arterial side O2 has 97mmHg and CO2 has 40mmHg; On venous side O2 has 40mmHg and CO2 has 46mmHg
1814. Passive exhalation begins with:

expiratory muscle contraction;

relaxation of the inspiratory muscles;

air outlet from the lungs;

contraction of the inspiratory muscles.

1815. A high level of sympathetic stimulation is most likely to cause which of the following effects on gastrointestinal function?

Decrease in stomach Ph

Decrease motility

Increase in gastric acid secretion

Increase in mucous secretion

1816. Which of the following activates trypsin in the small intestine?

Cl⁻

Enterokinase

NaHCO3

gastrin

1817. Alveolar ventilation in a male with a respiratory rate of 10/min and tidal volume of 600 ml is

1750ml

3000ml

4500ml

6000ml

1818. The pressure difference between the intra-alveolar and intrapleural pressures is called ________.

atmospheric pressure

pulmonary pressure

negative pressure

transpulmonary pressure

1819. The stomach wall is made up of 2 mucous cells [surface + neck], parietal cells, chief cells, and G cells. Which one drives
stomach action via secretion and motility?

Chief

G
Mucous

Parietal

1820. Normally, most of the water in the GI lumen is absorbed from:

stomach

duodenum

jejunum

colon

1821. Which activity below is not a respiratory event?

Carbon dioxide is exchanged in the alveoli.

Cells produce nitrogen by their metabolism.

Gas exchange between tissues and the blood.

Oxygen is exchanged in the alveoli.

1822. All of the following contribute to the absorptive surface area of the small intestine except

villi.

circular folds.

haustra.

the brush border.

its length.

1823. Which of the following glands does not secrete the mucous component of saliva?

Buccal

Parotid

Sublingual

Submandibular

1824. Stomach emptying is controlled mostly by which of the following?

Activity if the small intestine

Local neurohumoral mechanisms originating in the duodenum.

Local neurohumoral mechanisms originating in the stomach itself

Vasovagal reflexes that reduce the tone in the muscle wall of the body of the stomach
1825. Which of the following is not an enzyme?

chymotrypsin

enterokinase

secretin

pepsin

1826. The pressure in the pleural cavity during normal inspiration is:

- 6 - 9 mm Hg

- 3 - 4 mm Hg

- 15 - 20 mm Hg

1,0 mm Hg

1827. Breathing capacity is:

the volume of air inhaled and exhaled with quiet breathing;

the volume of air that can be exhaled during maximum exhalation after a normal inhalation and exhalation;

the volume of air that remains in the lungs after the maximum expiration;

the volume of air that can be inhaled with the maximum inhale after the usual inhalation.

1828. What is diffusion?

A planned movement of molecules through the respiratory membrane and adjacent fluids

the random motion of molecules in all directions through the respiratory membrane and adjacent fluids

motion of the blood parts from higher to lower concentrated area through the respiratory membrane

motion of any fluid from lower to higher concentrated area through the respiratory membrane

1829. Esophagus at rest is:

Open at the bottom

Open at the top and the bottom

Closed at the top and the bottom

1830. The major humoral mediator of pancreatic duct cell secretion of bicarbonate rich juice is:

secretin

CCK

somatostatin
all of the above

1831. All of the following statements decrease the lung compliance except:

Lung fibrosis

Increased pulmonary venous pressure

long period of time where the lung is unventilated

Emphysema

1832. Which of the following is not an effector of respiratory system?

blood circulatory system

diaphragm

intercostals

trapezius

1833. Where does the majority of chemical digestion in the stomach occur?

fundus and body

cardia and fundus

body and pylorus

body

1834. The amount of air inhaled or exhaled in one breath during related, quiet breathing is:

functional residual capacity

residual volume

tidal volume

total lung capacity

1835. Which of the following substances can inhibit small intestinal motility?

Cholecystokinin

Gastrin

Insulin

Secretin

1836. Which of the following can cause stagnant hypoxia?

COPD
Shock or heart failure

Cyanide poisoning

Carbon monoxide poisoning

1837. Which of the following enzymes does not digest any nutrients?

chymotrypsin

lingual lipase

carboxypeptidase

enterokinase

1838. The reason for the diffusion of gases from the alveolar air into the blood and back is:

the difference in partial pressure and tension between the alveolar air and blood;

close adherence of alveoli and capillaries;

active transport of O2 and CO2;

concentration gradient

1839. What is the major / principle hormone that stimulates HCl release?

CCK

Gastrin

Intrinsic Factor

Secretin

1840. Which term refers to the volume remaining in the lungs after maximum expiration?

expiratory reserve volume

inspiratory reserve volume

minimal volume

residual volume

1841. Segmental contractions within the GIT cause:

atony.

churning.

mixing.

peristalsis.
1842. The types of contraction that normally occur only in colon is:

peristaltis

antiperistaltis

segmentation

all of the above

1843. Most of the carbon dioxide in the blood is carried in the form of

dissolved CO2.

carbaminohemoglobin.

bicarbonate.

carboxyhemoglobin

1844. What happens in COPD?

There is a chronic dysfunction of the respiratory centre

The passage ways are damaged

The air passage ways are more widened

The respiratory membrane is damaged

1845. Which of the following is absorbed in the intestine independent of sodium?

glucose

amino acids

galactose

fructose

1846. The functional dead space includes:

the alveoli, which are ventilated but not perfused (no blood flow);

alveoli, which are ventilated and perfused;

the trachea;

bronchi.

1847. Which of the following hormone is normally released by the stomach into the systemic circulation?

Vasoactive intestinal polypeptide

Gastrin
Pepsinogen

Secretin

1848. Which of these does not affect diffusion rate?

Surface area for diffusion

Concentration gradient

Solubility of gases

molecular weight of gases

none of the above

1849. Residual volume of the lung is:

1200 ml

1500 ml

4500 ml

500 ml

1850. Hypochloridria is a condition in which acid secretion by the stomach is greatly reduced. How does this condition affect
the digestion and absorption of food?

Digestion and absorption of all food substances are nearly normal

Digestion and absorption of all foods substances are markedly reduced

Digestion and absorption of carbohydrates are greatly affected by this condition

Digestion and absorption of fats are greatly affected by this condition

1851. Which of the following cell types found in the intestinal tract secretes hydrochloric acid?

Acinar cells

Mucous neck cells

Parietal cells

Peptic cells

1852. Inhale is:

an active process of air flow into the lungs;

an active process of carbon dioxide in the lungs;

passive process of air flow into the lungs;

active process of removing carbon dioxide from the lungs.


1853. The rhythmic control of breathing is produced by the activity of inspiratory and expiratory neurons in

the medulla oblongata.

the apneustic center of the pons.

the pneumotaxic center of the pons.

the cerebral cortex.

1854. Asthma is an example of:

hypoventilation

Histotoxic hypoxia

Hyperventilation

Normal ventilation but abnormal circulatory system

1855. The most potent stimulus for release of secretin from the duodenum is:

a reduction in duodenal lumen pH to <4.5

peptides

fatty acids with > 8 carbons

carbohydrates

1856. Which of the following does not break down proteins?

Carboxypeptidase

Chymotrypsin

Pepsin

None of them

1857. What stimulate the ileocecal sphincter to open?

Change in pH

Distention

Pelvic Nerve

Vagus nerve

1858. Which of the following substances is released from the mucosa of the duodenum in response to acidic gastric juice?

Cholecystokinin

Gastric inhibitory peptide


Secretin

Substance P

1859. According to _______ , the warming of air as it is inhaled helps to inflate the lungs.

Boyle’s law

Charles’ law

Dalton’s law

the Bohr effect

1860. The substance secreted from D cells is responsible for:

HCl activation.

protection against HCl.

stomach contraction.

stomach relaxation.

1861. Which of the following has the most powerful stimulatory effect on salivary secretion under normal conditions?

Gritty material

Sour taste stimulus

Sympathetic stimulation

Trigeminal stimulation

1862. When is salivary amylase most active?

Churning in Stomach

Lumen in Duodenum

Mastication in Mouth

Storage in Stomach

1863. Motor neurons in the myentric plexus that stimulate the contraction of smooth muscle in the gastrointestinal tract
release:

acetylcholine

vasoactive intestinal polypeptide

nitric oxide

somatostatin
1864. Specify a property that is not characteristic of the pleural cavity:

tightness;

the presence of serous fluid;

lack of air;

the presence of air.

1865. The anatomic dead space:

varies with minute ventilation

is typically 150ml

will increase COPD

is alveolar minus the pathological dead space

all of the above

1866. Stomach motility is increased in:

parasympathetic stimulation.

sympathetic stimulation.

during sleep

none of the above

1867. The submandibular gland secretes

Lingual lipase

Lysozyme

Proline rich proteins

Salivary amylase

1868. Physiologically, the most important choleretic is:

bile salts

CCK

secretin

gastrin

1869. The oxyhemoglobin dissociation curve shows the relationship between:


the amount of O2 and intrapleural pressure

tension O2 blood and the value of intrapulmonary pressure

the percentage of oxyhemoglobin and the oxygen tension in the blood

the percentage of oxyhemoglobin and the amount red blood cells

1870. Pulmonary ventilation is necessary for:

the approximation of the alveolar air in composition to the atmospheric

maintain constancy of alveolar air

reduce the amount of O2, increase the amount of CO2 in the alveolar air

increasing the amount of O2 and CO2 in the alveolar air

1871. Which of the following composes the majority of salivary volume?

Parotid gland

Small salivary glands

Sublingual gland

Submandibular gland

1872. Pulmonary surfactant a) b) c) d) e)

Prevents collapse of alveoli.

Prevents small alveoli from joining with larger alveoli.

Increases lung compliance.

Both a and c.

All of the above.

1873. Acidic histamine is secreted by:

Chief cells

Enterochromaffin cells

G cells

Parietal Cells

1874. Digestion of which of the following occurs almost entirely in the small intestine?

Fat

Fruits and vegetables

Protein
ote

Starch

1875. The pattern of electrical and motor activity in the gastrointestinal tract during periods of fasting is called:

basic electrical rhythm

migrating motor complex

peristaltis

segmentation

1876. Respiratory rate in an adult at rest equals:

5–10 / min;

14-20 / min;

25–30 / min;

30–40 / min.

1877. What form of anaemia is associated with Gastritis?

Haemolytic

Iron deficiency

Megaloblastic

Pernicious

1878. Which of the following enzymes acts in the stomach?

chymotrypsin

lingual lipase

carboxypeptidase

enterokinase

1879. All of the following shift the oxygen dissociation curve to the right except:

decreased pH

Increased temperature

Carbon Monoxide

Increased 2.3 DPG

1880. Which of these ingredients in saliva is responsible for activating salivary amylase?
mucus

phosphate ions

chloride ions

urea

1881. When all muscles of respiration are relaxed and alveolar pressure is zero, lung volume is equal to

Residual volume.

Vital capacity.

Functional residual capacity.

Total lung capacity.

1882. All of the following can bond with hemoglobin except

HCO3−

O2.

H+.

CO2.

1883. Which of the following components of bile is critical for fat digestion?

Bicarbonate

Bilirubin

Calcium salts

Lecithin

1884. Which of the following processes does atmospheric pressure play a role in?

pulmonary ventilation

production of pulmonary surfactant

resistance

surface tension

1885. Which of the following statements regarding the actions of gastrointestinal hormones is incorrect?

secretin inhibits gastric emptying

gastrin stimulates histamine release in the stomach

somatostatin reduces blood flow to the GIT

GIP inhibits the release of insulin from pancreas


p

1886. We need OXYGEN exactly for:

anaerobic respiration

aerobic respiration

to produce internal body H2O

All of the above

1887. Gas exchange between alveoli and blood is carried out in:

arteries of the pulmonary circulation;

veins of the pulmonary circulation;

the capillaries of the great circle of blood circulation;

the capillaries of the pulmonary circulation.

1888. Normally, there is a net secretion of which ion in the colon?

sodium

potassium

chloride

calcium

1889. The term “brain of the gut” is used to refer to the:

autonomic ganglia

enteric nervous system

migratory motor complex

interstitial cells of Cajal

1890. Tidal volume in man is:

1200 ml

1500 ml

4500 ml

500 ml

1891. Quantitively, the most important enzyme in digestion of fat is:

lingual lipase

gastric lipase
gastric lipase

pancreatic lipase

lipoprotein lipase

1892. Which muscles are involved in inspiration?

Diaphragm

Diaphragm and external intercostal muscles and abdomial muscles

Diaphragm and External intercostal muscles

Diaphragm and Internal intercostal muscles

1893. Spirometry can measure all except:

IC

ERV

FRC

VC

TV

1894. Where is the Pacemaker of Respiratory system located?

In the alveoli

In the lobes of Lungs

In the Medulla

In the Respiratory Membrane

1895. Inhibition of which of the following enzymes has the greatest effect on digaestion?

Chymotrypsin

Enterokinase

Pancreatic amylase

Ptyalin

1896. Which of the following normally stimulates the release of digestive enzymes from the ancinar cells of the pancreas?

Cholecystokinin

Gastrin

Histamine

Secretin
1897. The secretion of bile is important for the proper digestion of which of the following?

Complex carbohydrates

Lipids

Monosaccharides

Proteins

1898. Which of the following is not a function of the lung?

All are functions of the lung

Condensation of water vapor from expired air

Cool down air

Metabolism of substances

1899. Which of these processes occurs throughout most of the alimentary canal?

ingestion

propulsion

segmentation

absorption

1900. Which of the following characterizes carbohydrate digestion?

It begins when food comes in contact with gastric juice

It begins when the food comes in contact with saliva

It begins with when food comes in contact with pancreatic secretions

It ends when starch has been converted to maltose

1901. Which of the following describes how bicarbonate ions are absorbed from the small intestine?

Directly via facilitated diffusion

Directly via simple diffusion

Indirectly in the form of carbon dioxide and water

Indirectly via active transport

1902. Which of these processes occurs in the mouth?

ingestion

mechanical digestion
mechanical digestion

chemical digestion

all of the above

1903. The volume of food in the stomach has which of the following effects on stomach emptying?

A the volume if food in the stomach increases, the rate of emptying decreases

A the volume of food in the stomach decreases, the rate of emptying decreases

A the volume of food in the stomach increases, the rate of emptying increases

The volume of food in the stomach has little if any effect of the rate if stomach emptying

1904. Which of the following is not a function of the lung?

All are functions of the lung

Condensation of water vapor from expired air

Cool down air

Metabolism of substances

1905. Which of the following best characterizes the secretions of the small intestine?

Hypotonic and slightly acidic

Hypotonic and slightly alkaline

Isotonic and slightly acidic

Isotonic and slightly alkaline

1906. How does gastrin stimulate hydrochloric acid secretion by the parietal cells of the gastric gland?

Directly by acting on a gastrin receptor located on the parietal cell

Indirectly through a cyclic adenosine monophosphate (cAMP)- mediated pathway

Indirectly through a cyclic guanosine monophosphate (cGMP).- mediated pathway

Indirectly via stimulation of histamine release from enterochromaffin cells

1907. In the renal nephrons are filtered:

blood cells

plasma molecular proteins

metabolites

molecular components of plasma and water


1908. Basal metabolic rate in women compared to men

is the same

less by 10-15%

more by 10-15%

30-40% less

1909. Structural and functional unit of the kidneys is:

acinus

micelle

nephron

neuron

1910. Volume of urine is regulated by ___________

Aldosterone

ADH and Aldosterone

Aldosterone and testosterone

ANP

1911. The process of transition of water and substances dissolved in it from the tubules of the nephron into the blood is called:

synthesis

secretion

reabsorption

filtering

1912. Hypertension is likely to increase the secretion of

atrial natriuretic peptide.

antidiuretic hormone.

bicarbonate ions.

aldosterone.

1913. Juxtaglomerular apparatus is not formed by:

Macula densa

Extraglomerular mesangial cells

Juxtaglomerular cells
Juxtaglomerular cells

Podocytes

1914. What is the amount of energy consumption by persons engaged in particularly hard physical labor?

about 2200 kcal / day

about 3400 kcal / day

about 4300 kcal / day

about 7600 kcal / day

1915. Following are the hormones that regulate tubular reabsorption, except:

Aldosterone

Testosterone

Calcitonin

Parathormone

1916. Most sodium is reabsorbed from the glomerular filtrate by

the vasa recta.

the proximal convoluted tubule.

the distal convoluted tubule.

the nephron loop.

1917. Both formation and concentration of urine is mainly the function of:

Cortical nephrons

Juxtamedullary nephrons

Both types

None of the above

1918. What is the function of ADH?

Controls sugar level in blood

Increases water absorption

Decreases water absorption

Synthesis of salt

1919. The average daily basal metabolic rate in men is


3000 kcal

1000 kcal

2500 kcal

1700 kcal

1920. All the pressures involved in determination of filtration are collectively called:

Starling forces

Colloidal forced

1921. Tubuloglomerular feedback is the mechanism that regulates GFR through:

Renal tubule

Macula densa

Both

None

1922. . In the tubules of the nephrons are completely reabsorbed:

amino acids and glucose

urea and creatinine

dyes and radiopaque substances

high molecular substances

1923. Liquid which collects in Bowman’s capsule is ___________

Water and sulphates

Water and glycogen

Plasma minus blood proteins

Concentrated urine

1924. Filtration fraction is:

Portion of plasma that becomes filtrate

Portion of filtrate that becomes urine

Portion of urine that is reabsorbed

Portion is plasma, secreted into nephron

1925. Glomerular capillary pressure varies between:


20 to 30 mm Hg

45 to 70 mm Hg

80 to 100 mm Hg

10 to 15 mm Hg

1926. What way of heat transfer predominantly functions in humans at an ambient temperature of 40 ° C and normal humidity?

heat conduction

radiation

convection

evaporation

all answers are correct

1927. Tubular secretion is the process by which the substances are transported:

From blood into renal tubules

From renal tubules into blood

From Renal tubules into urine

From urine into blood

1928. Under what conditions does increased sweating not lead to an increase in heat transfer?

with the formation of a large amount of sweat

with the formation of highly concentrated sweat

at very low humidity

at very high humidity

1929. Low pressure capillary bed is formed by

Renal glomerular capillaries

Peritubular capillaries

Afferent arterioles

Efferent arterioles

1930. How much energy is released during the utilization of 1 g of glucose in the body?

9.3 kcal

4.1 kcal
5.0 kcal

3.75 kcal

1931. 85% of all nephrons are:

Cortical nephrons

Juxtamedullary nephrons

Collecting nephrons

Mixed nephrons

1932. Chemical thermoregulation provides ...

change in the rate of breakdown of carbohydrates

change in the intensity of fat hydrolysis

change in the intensity of protein breakdown

change in the intensity of heat production

1933. Which of these lies closest to the renal cortex?

the parietal peritoneum

the renal fascia

the renal capsule

the adipose capsule

1934. Tubuloglomerular feedback is the mechanism that regulates GFR through:

Renal tubule

Macula densa

Both

None

1935. The daily minimum volume of obligatory H2O loss that must accompany excretion of wastes is ml.

500

50

100

1000

1936. What are the daily energy consumption of people who are predominantly engaged in mental work?
2500-3000 kcal

2100-2450 kcal

3000-4000 kcal

1500-1700 kcal

1937. Increasing alveolar ventilation increases the blood pH because

it activates neural mechanisms that remove acid from the blood.

it makes hemoglobin a stronger acid.

it increases the PO2 of the blood.

it decreases the PCO2 in the alveoli.

1938. The amount of energy exchange after eating protein food

decreases by 10-20%

does not change

increases by 30-40%

increases by 10-20%

1939. In which of the following are microvilli most abundant?

Bowman’s capsule

Glomerular capillaries

Distal tubule

Proximal tubule

1940. . The juxtaglomerular apparatus of the kidney secretes into the blood:

aldosterone

sodium ureteric factor

angiotensin

renin

1941. Which one is an important constituent of rennin angiotensinogen aldosterone system?

Macular cells

Plasma cells

JGA cells
Erythropoietin

1942. The total calorie content of the daily diet (in%) is distributed between breakfast, lunch, dinner and the second dinner
approximately as follows:

10, 20, 45, 25

30, 40, 20, 10

25, 25, 35, 15

50, 25, 15, 10

1943. Reabsorption of the substances occur at the following sites:

Proximal convulated tubule

Distal convulated tubule

Loop of henle

All

1944. In the kidneys are secreted:

glucose and amino acids

vitamins and water

proteins and salts

Potassium ions and medicinal substances

1945. The juxtaglomerular apparatus of the kidney is:

a triangle formed by the walls of the carrying and removing arterioles and the cells of the distal convoluted tubule

a triangle formed by the walls of the carrying and removing arterioles and the cells of the proximal tubules

a triangle formed by proximal and distal convoluted tubule

loop of Henle, collecting tubules and vessels that surround them

1946. The lowest body temperature of a healthy person is observed at

7 am

13 pm

16 pm

4 am

19 pm
1947. What vitamins are fat-soluble?

C, B1, B2, B6

B6, H, B3, C

A, D, E, K

PP, B12, B6, B1

1948. The most abundant cation in the ICF is _______ .

Na+

K+

CL-

Ca+

1949. Tubular secretion of hydrogen is directly linked to

tubular secretion of potassium

tubular secretion of sodium

tubular reabsorption of potassium

tubular reabsorption of sodium

1950. The skin loses water by two processes, sweating and _______ .

evaporation

conduction

convection

radiation

1951. The daily fat requirement of a middle-aged person is

100-150 g

400-450 g

200-300 g

70-90 g

1952. In the distal convulated tubules, the sodium reabsorption is stimulated by hormone:

Angiotensin
Erythropoiten

Aldosterone

Parathormone

1953. Hypotonic hydration can result from

ADH hypersecretion.

ADH hyposecretion.

aldosterone hypersecretion.

aldosterone hyposecretion.

1954. How does the basal metabolic rate change after 35-40 years?

increases

decreases

does not change

1955. Which of the following filtered substances is normally not present in the urine?

Na+

H+

glucose

urea

1956. During the day in the body of an adult, ATP is formed in an amount of up to ...

200-250 kg

5-10 kg

1-3 kg

50-70 kg

1957. Each kidney has got about 8:

Minor calyx

Major calyx

Pyramids

Arcuate artries

1958. What are nutrients?


these are food components that maintain the body's water balance

food components that provide energy consumption and synthetic processes of the body

these are protective substances

1959. In micturition _____

Urethra relaxes

Ureters relax

Ureters contract

Urethra contracts

1960. Under normal conditions, the transfer of heat by the body can be carried out by ...

increase muscle tone and tremors

activation of non-contractile thermogenesis

heat radiation, convection, heat conduction, evaporation

only heat radiation, convection, heat conduction

heat radiation, convection, evaporation and thermogenesis

1961. Glomerular filtrate contains _________

Plasma without sugars

Blood without blood cells and proteins

Blood with proteins but without cells

Blood without urea

1962. Which of the following is not a part of renal pyramid?

Collecting ducts

Loops of Henle

Convoluted tubules

Peritubular capillaries

1963. The main factors affecting the filtration rate in nephrons are:

hydrostatic pressure, oncotic pressure of blood

osmotic pressure, blood pH

only intrarenal pressure and oncotic pressure of blood


partial pressure of gases

1964. In the descending parts of the loop of Henle nephrons occurs:

dilution of urine

urine concentration

the formation of ultrafiltrate

secretion of water and salts

1965. What is the most optimal ratio of proteins, fats and carbohydrates in the diet?

1:4:1

4:1:1

1:1:4

1:2:4

1966. Physical thermoregulation is a mechanism

increase sweating

changes in heat transfer

increasing heat transfer

reducing the intensity of metabolism

1967. How much energy is released when 1 g of body fat is utilized?

3.75 kcal

4.1 kcal

6.1 kcal

9.3 kcal

1968. The renal pyramids are separated from each other by extensions of the renal cortex called ________.

renal medulla

minor calyces

medullary cortices

renal columns

1969. Somatic motor neurons must be ________ to relax the external urethral sphincter to allow urination.

stimulated
inhibited

1970. Main function of Henle’s loop is ___________

Passage of urine

Filtration of blood

Formation of urine

Conservation of urine

1971. . Beavers have relatively little need to conserve water and could therefore be expected to have _______ than humans do.

fewer nephrons

longer nephron loops

shorter nephron loops

longer collecting ducts

1972. The highest body temperature in a healthy person is observed in 1) 24 hours 3) 7 hours 4) 10 hours

at 18 pm

at 4 am

at 7 am

at 10 am

1973. Angiotensin II produced under the effect of Renin, produces which of the following effects?

Increases blood pressure directly by noradrenaline

Increases blood pressure indirectly vasoconstriction

Increases blood pressure directly by vasoconstriction

Increases blood pressure by vasodilation

1974. Which part of the urinary system is not completely retroperitoneal?

kidneys

ureters

bladder

nephrons

1975. Micturition occurs in response to

Relaxation of the detrusor muscle.


e a at o o t e det uso usc e.

Contraction of the internal and external urethral sphincters.

Activation of parasympathetic neurons to the bladder.

Activation of somatic motor neurons to the bladder.

1976. The amount of energy exchange after eating carbohydrate food

decreases by 10-20%

does not change

increases by 30-40%

increases by 10-20%

1977. All are the structural components of kidney, except:

Inner medulla

Trigone

Renal sinus

Nephron

1978. Which of the indicated values is the upper lethal temperature of the human body (0C)?

42,5

18

45

41

1979. What are the names of the capillaries following the efferent arteriole?

arcuate and medullary

interlobar and interlobular

peritubular and vasa recta

peritubular and medullary

1980. Secretion, one of the processes of urination, occurs:

in Bowman capsules

from the blood to the renal tubules of the nephrons

from the blood in the loop of Henle nephrons

from distal tubules of nephrons into the blood


1981. Following are functions of the kidneys, except:

Uric acid production

Maintenance of water balance

Maintaince of acid base balance

Erythropietic production

1982. The primary structure found within the medulla is the ________.

loop of Henle

minor calyces

portal system

ureter

1983. Hyperchloremia is most likely to result in

alkalosis

acidosis

hypernatremia

hyperkalemia

1984. The kidneys help to maintain acid-base balance by

the secretion of H+ in the distal regions of the nephron.

the action of carbonic anhydrase within the apical cell membranes.

the action of carbonic anhydrase within the cytoplasm of the tubule cells.

the buffering action of phosphates and ammonia in the urine.

all of these means.

1985. The daily requirement of a middle-aged person for carbohydrates is

150-200 g

400-450 g

80-100 g

up to 800 g

1986. Micturition occurs when the _______contracts.

detrusor muscle
internal urethral sphincter

external urethral sphincter

muscularis of the ureter

all of the above

1987. _______ increases water reabsorption without increasing sodium reabsorption.

Antidiuretic hormone

Aldosterone

Atrial natriuretic peptide

Parathyroid hormone

1988. . Increased ADH secretion should cause the urine to have

a higher specific gravity.

a lighter color.

a higher pH.

a lower urea concentration.

1989. About 90% of kidney stones can pass out through urinary system by drinking _______

Juice

Water

Sugar drinks

Milk

1990. The most abundant cation in the ECF is _______ .

Cl-

K+

Na+

Ca+

1991. Kidney reabsorption is:

the process of transition of water and substances dissolved in it from blood into the Bowman capsule

the process of transition of water and substances dissolved in it from blood to the renal tubules

the process of transition of water and substances dissolved in it from blood in the loop
the process of transition of water and substances dissolved in it from tubules in the blood

1992. Juxtaglomerular hormone secretes which hormone:

Progesterone

Prostaglandin

Testosterone

None of the above

1993. How does the tone of skin vessels change under the influence of cold?

decreases

increases

does not change

1994. Assume that fluid enters the distal tubule with an osmolarity of 100 mOsm, and that the maximum osmolarity of
medullary interstitial fluid is 1100 mOsm. As plasma ADH levels rise, what happens to the osmolarity of the urine?

It approaches 100 mOsm as a lower limit.

It approaches 1100 mOsm as an upper limit.

It eventually exceeds 1100 mOsm.

It approaches 300 mOsm, the normal osmolarity of plasma.

1995. During the day, under normal conditions, how much water is excreted from the body through the skin and lungs?

900 ml

2000 ml

5000 ml

100 ml

1996. Glomerular filteration is which of the following process?

Microfiltration

Macrofiltration

Ultrafiltration

Autofiltration

1997. Which of the following does not stimulate aldosterone release?

Atrial natriuretic peptide


An increase in plasma potassium

An increase in renin secretion

An increase in angiotensin II production

1998. The protein content in meat products is about ...

20%

10%

50%

100%

1999. GFR and permeability of Glomerular capillary membrane has one of the following relationship:

Directly propotional

Indirectly proportional

Inversely proportional

Negative feedback

2000. Micturition occurs when the _______contracts.

detrusor muscle

internal urethral sphincter

external urethral sphincter

muscularis of the ureter

2001. GFR is directly proportional to:

Tempreature of body

Renal blood flow

Pulse rate

Respiratory rate

2002. What are the characteristics of homeothermic animals?

the constancy of body temperature

the dependence of body temperature on ambient temperature

the constancy of the metabolic rate

the constancy of body temperature, regardless of the ambient temperature


2003. How much energy is released during the utilization of 1 g of protein in the body?

3.75 kcal

4.1 kcal

9.3 kcal

5.1 kcal

2004. A person has the following symptoms: arterial pH = 7.48, PCO2= 44 mm Hg, plasma bicarbonate concentration = 27 mM
What is the diagnosis?

Respiratory acidosis

Respiratory alkalosis

Metabolic acidosis

Metabolic alkalosis

2005. A glomerulus and glomerular capsule make up one

renal capsule.

renal corpuscle.

kidney lobule.

kidney lobe.

2006. The comfort zone of a dressed person with an air humidity of 50% is the ambient temperature (in degrees Celsius)

16-18

22-24

26-28

18-20

2007. A calorie is ...

unit of measure for heat, equal to 0.239 Joule

the unit of measure for heat is 2.4 Joules

unit of measure for heat, equal to 4.2 Joules

unit of measurement of heat equal to 1 watt

2008. . If 180 liters of primary urine are produced per day, then in 1 minute of ultrafiltrate:

25 ml.

18 ml.
125 ml.

1.5 liters.

2009. Which of the following is the principal buffer in interstitial fluid?

hemoglobin

other proteins

carbonic acid

H2PO4

2010. A blood pH of 7.2 caused by inadequate pulmonary ventilation would be classified as _______ .

Respiratory alkalosis

Respiratory acidosis

Metabolic acidosis

Metabolic alkalosis

2011. In the presence of which hormone, distal convulated tubule and collecting duct of renal tubule become permeable to
water so that it is reabsorbed?

Aldosterone

Parathormone

Growth hormone

Anti diuretic hormone

2012. The process of the transition of water and the capillaries dissolved in it from the blood into the Bowman capsule is called:

synthesis

secretion

reabsorption

filtering

2013. Glomerular filtrate collected at Bowman's capsule contains all the substances of plasma except protiens which makes it:

Hypotonic to plasma

Hypertonic to plasma

Isotonic to plasma

None of the above


2014. The basal metabolic rate is ...

the amount of energy required to maintain life under normal conditions

the minimum energy required at resting state

the maximum amount of energy required for life

all the energy needed to maintain life

2015. The principal determinant of intracellular osmolarity and cellular volume is

protein

phosphate

potassium

sodium

2016. How much energy in the body is ultimately released from the body in the form of heat?

20%

100%

40%

60%

2017. The color of urine is determined mainly by ________.

diet

filtration rate

byproducts of red blood cell breakdown

filtration efficiency

2018. Production of less than 50 mL/day of urine is called ________.

normal

polyuria

oliguria

anuria

2019. Podocytes are found in ______________

Cortex of nephron

Outer wall of Bowman’s capsule

I ll f B ’ l
Inner wall of Bowman’s capsule

Wall of glomerular capillaries

2020. Yellow color of urine is due to ____________

Creatinine

Urea

Bilirubin

Uric acid

2021. What amount of heat is removed from the body under normal conditions at room temperature by heat radiation?

26%

30%

46%

66%

2022. Normal GFR is about:

125 ml/min

180 L/day

none of the above

both a and b

2023. Each kidney consists of:

1 to 1.3 million nephrons

3 to 5 million nephrons

10 billion nephrons

0.5 million nephrons

2024. The _______ sphincter is under involuntary control and relaxes during the micturition reflex.

ileocecal

Oddy's

Internal urethra

2025. The kidney has more _______ than any of the other structures listed.

arcuate arteries
minor calices

medullary pyramids

afferent arterioles

2026. Principal and intercalated cells form:

Ascending limb

Proximal convoluted tubules

Collecting duct

Glomerulus

2027. How will the basal metabolic rate change with a decrease in thyroid function?

will increase

decrease

will not change

2028. Which structure acts as the countercurrent exchanger?

Vasa recta

Macula densa

Glomerulus

Bowman's capsule

2029. In which area of the human body is the highest temperature?

in the liver

in the rectum

in the armpit

under the tongue

2030. What energy is not used in the body to do work?

chemical

mechanical

electric

thermal

2031. The right kidney is slightly lower because ________.


it is displaced by the liver

it is displace by the heart

it is slightly smaller

it needs protection of the lower ribs

2032. The greatest percentage of the body’s water is in

the blood plasma.

the lymph.

the intracellular fluid.

the interstitial fluid.

2033. Renal clearance (clearance) of inulin is a method of assessment:

renal blood flow

tubular secretion

glomerular filtration

tubular reabsorption

2034. The process of transition of substances from the blood of capillaries into the nephron canaliculi is called:

synthesis

secretion

reabsorption

filtering

2035. Water produced by the body’s chemical reactions is called _______ .

waste product

metabolic water

blood plasma

lymph

2036. In the tubules of the nephrons are completely reabsorbed:

amino acids and glucose

urea and creatinine

dyes and radiopaque substances


high molecular substances

2037. What are the characteristics of poikilothermic animals?

the constancy of body temperature, regardless of the ambient temperature

lack of constancy of body temperature

the dependence of body temperature on ambient temperature

the constancy of the metabolic rate

2038. Which of these is the most abundant nitrogenous waste in the blood?

uric acid

urea

ammonia

creatinine

2039. The renal clearance of _______ is normally zero.

sodium

potassium

uric acid

amino acids

2040. Which structure of the urinary system stores urine until it is excreted?

Kidneys

Bladder

Ureter

Urethra

2041. The functional unit of the kidney is called ________.

the renal hilus

the renal corpuscle

the nephron

Bowman’s capsule

2042. The most effective buffer in the intracellular fluid is

phosphate
protein

bicarbonate

carbonic acid

2043. The daily requirement for vitamin C is ...

50-100 mg

5-10 mg

20-30 mg

0.5-5 mg

2044. A substance of renal origin, which increases blood pressure:

ammonia

urokinase

renin

urea

2045. The compact ball of capillaries in a nephron is called

the nephron loop.

the peritubular plexus.

the renal corpuscle.

the glomerulus.

2046. Mark the mismatch pair.

Henle’s loop – concentration of urine

DCT – Absorption of glucose

PCT – Absorption of sodium and potassium ions

Bowman’s capsule – Glomerular filtration

2047. The fat content of vegetable oil is about ...

50-60%

10-20%

70-80%

100%
2048. In adults, during resting conditions both the kidneys receive 1,300 ml/min blood which is:

50% of cardiac output

10% of cardiac output

26% of cardiac output

70% of cardiac output

2049. Blood filtrate is captured in the lumen of the ________.

glomerulus

Bowman’s capsule

calyces

renal papillae

2050. Peristaltic contractions occur in the ________.

urethra

bladder

ureters

urethra, bladder, and ureters

2051. ADH makes the urine hypertonic with osmolarity of:

1,200 mOsm/L

2000 mOsm/L

5000 mOsm/L

500 mOsm/L

2052. The composition of the primary urine does not include:

urea

red blood cells

vitamins

amino acids

2053. Urea synthesis occurs in ______________

Kidney

Liver
Pancreas

Muscles

2054. Excretion is the process by which unwanted substances and metabolic wastes are:

Produced in the body

Utilized in the body

Eliminated from the body

Accumulated in the body

2055. _______ is the ability of a nephron to adjust its GFR independently of external nervous or hormonal influences.

renal autoregulation

nervous regulation

hormonal regulation

tubulaglomerular regulation

2056. Concentration and dilution of urine occurs in:

Bowman - Shimlyansky capsule

loops of Henle and collecting ducts

only in the collecting tubules of nephrons

only in the proximal tubules of the nephrons

2057. The countercurrent exchange in the vasa recta

removes Na+ from the extracellular fluid.

maintains high concentrations of NaCl in the extracellulnlar fluid.

raises the concentration of Na+ in the blood leaving the kidneys.

causes large quantities of Na+ to enter the filtrate.

does all of these.

2058. Vasopressin is called as ___________

Synovial fluid

Neurotransmitter

Antidiuretic hormone

Growth regulating substance


2059. Part of nephron impermeable to salt is ____________

DCT

Ascending limb of loop of Henle

Descending limb of loop of Henle

Collecting ducts

2060. Water, electrolytes and other substances are taken back into the blood from glomerular filterate, in a process called:

Tubular reabsorption

Tubular secretion

Glomerular extraction

Tubular transport

2061. What hormone directly opposes the actions of natriuretic hormones?

renin

nitric oxide

dopamine

aldosterone

2062. Pressure exerted by plasma protiens:

Colloidal osmotic pressure

Hydrostatic pressure

Net filtertion pressure

Glomerular capillary pressure

2063. 88% filtrate is reabsorbed in:

Distal convulated tubule

Proximal convulated tubule

Loop of Henle

Collecting duct

2064. The transfer of heat from a person in cold water is carried out mainly by

evaporation

radiation
all answers are correct

heat conduction

2065. Epithelial cells with pedicles on the basement membrane are called:

Podocytes

Astrocytes

Lymphocytes

Phagocytes

2066. Glomerular hydrostatic pressure is present in ________

Tubule of kidney

Bowman’s capsule

Malpighian tubule

Glomerulus

2067. Normal urinary out put is:

1 to 1.5 liters/day

5 to 10 liters/day

70 liters/day

20 liters/day

2068. Following substances are completely reabsorbed from the renal tubules, except:

Glucose

Vitamins

Amino acids

Minerals

2069. The main factors affecting the filtration rate in nephrons are:

hydrostatic pressure, oncotic pressure of blood, intrarenal pressure

osmotic pressure, blood pH

only intrarenal pressure and oncotic pressure of blood

partial pressure of gases

2070. The daily protein requirement of a middle-aged person is


120-150 g

150-200g

85-90 g

400-450 g

2071. The daily human need for water under normal conditions is ...

2,5 l

5l

1l

0,5 l

2072. Macula densa is the part of:

Renal hilum

Renal pelvis

Juxtaglomerular apparatus

Ureter

2073. Which of these is the most abundant nitrogenous waste in the blood?

uric acid

urea

ammonia

creatinine

2074. Assuming that arterial PCO2 is normal, metabolic acidosis promotes which of the following?

Increased hydrogen ion secretion by the renal tubule

Decreased alveolar ventilation

Increased bicarbonate reabsorption

Both a and c

2075. Glucose is mainly reabsorbed in _______

Henle’s loop

DCT

PCT

N h
Nephron

2076. In the lumen of the proximal tubule, secreted hydrogen ions are primarily buffered by

Bicarbonate.

Phosphates.

Proteins.

Sulfates.

2077. The condition where urea accumulates in blood is

Anemia

Glycosuria

Uremia

Acidosis

2078. Increased excretion of ammonium chloride in the urine most likely indicates

hypercalcemia

hyponatremia

hypochloremia

alkalosis

acidosis

2079. Substance which is not reabsorbed from loop of Henle:

Sodium

Phosphate

Chloride

None of the above

2080. Juxta glomerular cells of renal cortex synthesize a hormone called __________

ADH

Renin

Oxytocin

Urochrome

2081. The protein content in the edible part of the egg is about ...
12%

6%

50%

100%

2082. How much energy will enter the body when consuming 10 g of NaCl?

50 kcal

100 kcal

0 kcal

25 kcal

2083. Macula densa of the Juxtaglomerular apparatus is sensitive to:

Sodium chloride in the tubular fluid

Sodium Bicarbonate in tubular fluid

Protiens in tubular fluid

Pressure in glomerulus

2084. The osmolarity of tubular fluid increases as it flows through the descending limb of the loop of Henle because

Solutes are passively transported into the descending limb.

Solutes are actively transported into the descending limb.

Solutes are passively transported out of the ascending limb.

Water moves passively into the descending limb.

Water moves passively out of the descending limb.

2085. . Renal clearance (clearance) of inulin is a method of assessment:

renal blood flow

canalicular secretion

glomerular filtration

tubular reabsorption

2086. Which one of the following is the types of Sleep:

REM sleep

NREM sleep

REM sleep and NREM sleep


REM sleep and NREM sleep

REM, NREM & NTM Sleep

2087. The NREM sleep is divided into how many stages?

II

III

IV

2088. Electrical activity recorded by EEG may have:

synchronized waves

desynchronized waves

Both

None of the above

2089. It is the storage of information in brain for a longer period. The information could be recalled after hours, days, months or
years. It is:

Long term memory

Short term memory

Secondary memory

Primary memory

2090. Which functions are related to the higher nervous activity

Higher Intellectual functions

Higher Brain Functions

Higher Cortical Functions

All of these

2091. Which is not the stage of Non-Rapid Eye Movement Sleep

Stage of Drowsiness

Stage of Light Sleep

Stage of Medium Sleep

Stage of Deep Sleep

None of these
2092. Which one of the following is Skilled memory?

Non decorative memory

Explicit memory

Long term memory

Short term memory

2093. Types of conditioned reflex:

Classical

Instrumental

Permanent

Both a and b

Both b and c

2094. In the diagnosis of neuro logical disorders and sleep disorders, is useful:

Electroencephalogram

Electrocardiogram

Electrogram

Electroneurogram

Electrobrainogram

2095. Which of these is not a reflex reaction?

Salivation

Secretion of sweat

Flexion due to needle prick

Blinking of eyes due to strong light

2096. What forms the basis of higher mental functions?

congenital reflexes

Memory

Sleep

Conditional reflexes

2097. Sometimes which is also referred as negative memory:


Facilitation

Habituation

Short term memory

Long term memory

2098. An EEG:

rovides indication of intelligence

tends to show waves of smaller amplitude during deep sleep than alert state

show waves with a lower frequency during intense thought than during deep sleep

is bilaterally symmetrical

2099. Demonstration of conditioned reflex was first made by:

Pavlov

William Harve

Robert Brown

Karl von Frisch

2100. Which is the type of sleep associated with rapid conjugate movements of the eyeballs, which occurs frequently?

REM Sleep

NREM Sleep

Both NREM & REM Sleep

None of these

2101. During Sleep:

Formation of urine decreases

Specific gravity decreases

Systolic pressure falls to about 90-110mm Hg

All of these

2102. Bulbar or medullary reflexes are named because:

They have their centers in cerebral cortex

They are non protective

They have their centers in medulla oblongata

They are destructive in nature


They are destructive in nature

2103. Beta rhythm includes frequency waves:

15-28 per second

15-60 per second

1-5 per second

8-12 per second

2104. When atleast one part of reflex arc is formed by the autonomic nerve fiber, the reflex is termed:

Visceral reflex

Somatic reflex

Spinal reflex

Congenital reflex

2105. Alpha rhythm includes frequency waves:

15-60 per second

1-5 per second

8-12 per second

22-26 per second

2106. Long term memory, points to:

Weeks to months

Years

Life time

All of the above

2107. Learning is:

The process by which new information is acquired

The process by which old information is revised

Not helpful

Very higher in the age of above 90

2108. What are the types of Inhibition of Conditioned reflexes

External inhibition & Internal inhibition


Primary, Secondary & Tertiary Inhibition

Associative and Non- associative inhibition

None of the above

2109. Source of EEG:

A potential of pyramidal cells

A potential of ganglion cells

EPSP and IPSP of cortical cells which behave like dipoles

After potentials of parietalc cortex

2110. Recalling first day of schooling, birthday celebration of previous year, picnic enjoyed last week:

Short term Memory

Long term Memory

Moderate term Memory

Tertiary Memory

2111. Reflexes that protect body against gravitational force:

Flexor reflexes

Extensor reflexes

Aquired reflexes

Protective reflexes

2112. In vertebrates, simple reflex action is:

monosynaptic

polysynaptic

bisynaptic

trisynaptic

2113. Tone in all the muscles of body except ocular muscles decreases very much during sleep. It is called as:

Sleep Paralysis

Dehydrated Sleep

Tired Sleep

Normal Sleep
2114. Depending upon duration, memory is classified into all, except:

Sensory

Primary

Secondry

Tertiary

2115. On the basis of past experience, which alters the behaviour of a person?

Memory

Learning

Reading

Writing

2116. Stretch reflex is the best example of:

Polysynaptic reflex

Monosynaptic reflex

Both of the above

None of the above

2117. Hippocampus and Papez circuit (closed circuit between hippocampus, thalamus, hypothalamus and corpus striatum) are
the main sites for:

Memory encoding

Memory strengthening

Memory sensitization

None

2118. Acquired reflexes are developed mostly after:

Conditioning

Watching

Trauma

Disease

2119. When an electric shock is given to the leg of a pathed frog, it:

shows reflex response by contraction of leg muscles

blinks its eyes


shows no response

put out its tongue

2120. Activation of Raphe Nucleus results in:

Rapid Eye Movement Sleep

Non-Rapid Eye Movement Sleep

Both REM & NREM Sleep

None of these

2121. Ability to recall past experience or information is called:

Cognition

Memory

Coordination

Facilitation

2122. Which is not involved in Unconditioned reflex?

learning and memory

associative learning and associative memory

non associative learning and non-associative memory

None of above

2123. The EEG rhythm having the lowest frequency is:

Alpha

Beta

Delta

Theta

2124. Recognition memory is the other name of:

Short term Memory

Long term Memory

Explicit Memory

Implicit Memory
2125. - Threshold for most of the reflexes increases. Pupils are constricted. Light reflex is retained. Eyeballs move up and
down. These occur in:

Bad Learning

Good Learning

Sleep

Muscle Tone increasing

2126. Synchronized waves are:

regular and invariant waves

Irregular and Variant Waves

Regular and Variant Waves

None

2127. Inhibition of Conditioned reflexes is of how many types?

II

III

IV

2128. Skilled memory is the other name of:

Short term Memory

Long term Memory

Explicit Memory

Implicit Memory

2129. Short term memory, points to:

seconds to minutes

Months to years

Hours to days

Weeks to months

2130. Desynchronized waves are:


Regular and invariant waves

Irregular and Variant Waves

Regular and Variant Waves

None

2131. Which is not the type of Learning:

Associative learning

Non-associative learning

Habituation

Sensitization

None of these

2132. Reflex action is controlled by:

Autonomic nervous system

Peripheral Nervous system

Central Nervous System

None of these

2133. Which is the natural periodic state of rest for mind and body with closed eyes characterized by partial or complete loss of
consciousness:

Memorizing older information

Sleep

Learning

Body Rest

2134. Which is responsible for Higher nervous activity:

extensive outer layer of gray matter

inner layer of white matter

Outer layer of White matter

Extensive Inner layer of gray matter

2135. Example of Polysynaptic reflex is:

Extensor reflex

Intersegmental reflex
Withdrawal reflex

Acquired reflex

2136. There are how many types of Learning?

2137. Which of the following is controlled by autonomous nervous system?

Papillary reflex

Swallowing food

Knee-jerk response

peristalsis of intestine

2138. Recollection of past experience for a very short period, on the basis of which an action is executed. It is related to:

Short term Memory

Long term Memory

Moderate term Memory

Primary Memory

2139. Working memory is the second name of:

Short term Memory

Long term Memory

Medium term Memory

Primary Memory

2140. Non-declarative memory is the other name of:

Short term Memory

Long term Memory

Explicit Memory

Implicit Memory

2141. Nightmares occur due to all, except


Improper food intake

Drug or alcohol withdrawal

Digestive or Nervous disorders

Sleeplessness

2142. Negative conditioned reflex is of the following types:

External

Internal

Both a and b

None of the above

2143. In which disease, EEG patterns are not altered:

Subdural Hematoma

Disorders of midbrain

Epilepsy

Influenza

2144. Type of learning that involves response of a person to only one type of stimulus:

Associative learning

Non-associative learning

Both of above

None

2145. The two centers Raphe nucleus and Locus ceruleus of pons which induce the Sleep are located in:

Brainstem

Cerebellum

Left Hemisphere

Right Hemisphere

2146. Anatomical basis of Memory is:

Synapses

Basal ganglia

lateral ventricle
Corona radiata

2147. Unconditioned reflexes are:

Natural reflexes

Inborn reflexes

None of the above

Both a and b

2148. Followings are the components of Reflex arc, except:

Sensory nerve

End organ

Effector organ

Dura matter

2149. The main difference between REM sleep and wakefulness is:

EEG desynchronization

Rapid eye movements

Decreased muscle tone

Penile erection

2150. What type of memory can be recalled after hours, days, months or years?

working memory

Secondary memory

Sensory memory

remote memory

2151. In what memory storage is enhanced and involves increase in synaptic transmission and increased postsynaptic activity.

Habituation

Facilitation

Fast learning

None of these

2152. What association is formed in secondary conditioning?

Association between two unconditioned responses


Association between the first conditioned stimulus and a second conditioned stimulus in the absence of the unconditioned
stimulus

Association between unconditioned stimuli and conditioned responses

Association between two conditioned responses without conditioned stimuli

2153. Which type of reflexes excite the Conditioned reflexes?

Primary Conditioned Reflexes

Secondary Conditioned Reflexes

Positive Coditioned Reflexes

Negative Conditioned Reflexes

2154. Activation of Locus ceruleus of pons results in:

Rapid Eye Movement Sleep

Non-Rapid Eye Movement Sleep

Both REM & NREM Sleep

None of these

2155. Which one illustrates a reflex arc?

brain-Spinal cord- muscles

muscle-receptors-brain

muscles- spinal cord- brain

receptor-spinal cord-muscles

2156. What is Sensitization?

A process by which person gets used to something, to which a person is constantly exposed.

A process by which the body is made to become more sensitive to a stimulus

A process by which old information is revised

None of these

2157. Explicit memory, points to:

Forgetfullness

Subjective recalling

Conscious recollection of past experience

Objective recalling
2158. Retaining sensory signals in sensory areas of brain, for a very short period of few seconds after the actual sensory
experience, i.e. few hundred milliseconds:

Sensory memory

Secondary memory

Long term memory

Short term memory

2159. Which is the type of sleep without the movements of eyeballs?

REM or Paradoxical Sleep

NREM or Slow-wave Sleep

Both REM & NREM Sleep

None of these

2160. Recalling of events of weeks, months, years or sometimes lifetime. It is related to:

Special type of Long term memory

Long term Memory

Moderate term Memory

Primary Memory

2161. In normal person EEG has:

Three frequency bands

Five frequency bands

Nine frequency bands

Seven frequency bands

2162. What is punishment?

Any consequence where something pleasurable is added

Any consequence which makes behavior likely to reoccur in the future

Another form of negative reinforcement

Any consequence which makes behavior unlikely to reoccur in the future

2163. Sweat and Lacrimal secretion increases during:

Sleep
Remembering

learning

Associative Learning

2164. After searching and finding telephone number in the directory, we remember the number for a short while. It is an
example of:

Primary memory

Secondary memory

Long term memory

Short term memory

2165. Which learning involves relations between two or more stimuli at a time.

Non-associative learning

Associative learning

Sensitization

habituation

2166. Delta rhythm includes frequency waves:

15-60 per second

1-5 per second

8-12 per second

15-28 per second

2167. Which involves the conscious recollection of past experience?

Associative Memory

Short term

Long term memory

Explicit memory

2168. How many types there are of Sleep?

II

III

IV

V
2169. Which of these is an example of conditioned reflex?

Sneezing

Yawning

withdrawl of hand on touching a hot plate

watering of mouth at the smell of food

2170. During Sleep:

Plasma level reduces

Plasma level increases

Plasma level is not changed

None of these

2171. Memory depends on:

duration

non-associative learning

way of learning

associative learning

2172. During sleep following changes occur, except:

Rate and force of respiration is decreased

Salivary secretion is decreased

Formation of urine is decreased

Sweat secretion is decreased

2173. Implicit memory, points to:

Skilled memory

Non skilled memory

Crude memory

Raw memory

2174. The process involved in facilitation of memory is called:

Memory sensitization

Negative memory
g y

Positve Memory

Habituation

2175. Loss of memory is a dangerous phenomenon. Which one is loss of memory?

Amnesia

Tremor

non-fixed memory

viral encephalitis

2176. Delta waves in EEG are seen in:

Deep sleep

REM sleep

awake with eyes open

awake with eyes closed

2177. Theta waves are obtained generally in:

Children below age of 5 years

Teen agers

Adults

Aged women

2178. During Sleep:

Heart rate varies between 45 and 60 beats per minute

Heart rate varies between 90 and 120 beats per minute

Heart rate increases

Heart rate is not changed much

2179. Following are the types of spinal reflexes, except:

Segmental

Intersegmental

Suprasegmental

Extra segmental
2180. Buerger waves ( alpha waves) of EEG have the rhythm per second of:

0-4

4-7

8-13

13-30

2181. Epilepsy is divided into how many categories:

III

II

IV

2182. Which of these is an unconditioned stimulus?

Food

Loud noise

Shock

Pain

None of these

all of these

2183. Which is the category of the Epilepsy:

Generalized Epilepsy

Localized Epilepsy

Both

None of these

2184. Conditioned reflex forms the basis of

all higher intellectual functions

all higher nervous activity

all higher cortical functions

all higher brain functions

All of these
2185. Nightmares are seen in:

REM sleep

NREM stage II

NREM stage III

NREM stage IV

2186. Which instrument is used to record EEG?

Electroencephalograph

Electrocardiograph

Electrogram

Electromusclograph

2187. Which drugs are shown to improve learning and memory:

Caffeine, nicotine, strychnine, Metrazol

epinephrine, adrenaline, Caffeine

None

All of these

2188. Which type of reflex is developed with one unconditioned stimulus and one conditioned stimulus?

Secondary conditioned reflex

Primary conditioned reflex

Tertiary conditioned reflex

Negative conditioned reflexes

2189. Which type of reflexes inhibit the Conditioned reflexes?

Secondary Conditioned Reflexes

Tertiary Conditioned Reflexes

Negative Conditioned Reflexes

Positive Coditioned Reflexes

2190. Which of these is an unconditioned response?

Sweating

Blink
Salivation

Startle response

None of these

All of these

2191. Which type of reflex is developed with one unconditioned stimulus and two conditioned stimuli?

Secondary conditioned reflex

Tertiary conditioned reflex

Primary Conditioned reflex

Positive Coditioned Reflexes

2192. The greatest resistance in the systemic circulation is observed at the level ...

elastic arteries

muscular arteries

capillaries

arterioles

small veins

2193. The long-term mechanism for regulation of blood pressure involves regulating which of the following?

Vessel diameter

Blood volume

Contractility

Heart rate

2194. What is the velocity of blood flow in the aorta?

10-15 cm / sec

0.25-0.05 m / s

2.0-5.0 m / s

1.5-2.0 m / s

2195. The low pressure zone of the cardiovascular system does not include:

capillaries of the systemic circulation

capillaries of the pulmonary circulation

right atrium left atrium


right atrium, left atrium

arteries of the systemic circulation

2196. Average blood flow velocity through a given vessel ...

is directly proportional to the cross-sectional area of the vessel

is inversely proportional to the cross-sectional area of the vessel

is directly proportional to the square of the vessel radius

is directly proportional to the radius of the vessel

2197. The procedure for listening to the heart sounds is called cardiac __ ____.

pulpation

auscultation

inspection

percussion

2198. What humoral factor has a more pronounced vasoconstrictor effect?

adrenaline

acetylcholine

norepinephrine

adenosine

2199. What is the normal duration of a QRS complex?

0.16 seconds (4 small squares)

0.04 seconds (1 small square)

0.12 seconds (3 small squares)

0.08 seconds (2 small squares)

2200. In which example is the blood pressure increased, mainly due to vasoconstriction?

140/90 mm Hg

140/100 mm Hg

140/80 mm Hg

140/70 mm Hg

2201. The maximum pressure in the right ventricle is normal:


25 to 30 mmHg

10 to 15 mmHg

60 - 80 mmHg

120-130 mmHg

2202. Mean arterial pressure increases in all following cases, except ...

an increase in the cardiac output

an increase in the peripheral vascular resistance

a decrease in the cardiac output

increased venous return

increased blood viscosity

2203. With age, the size of the heart ...

decrease due to atrophy

remain relatively constant

increase due to hypertrophy

all answers are correct

2204. The cardiac conduction system includes all of the following except

the SA node.

the AV node.

the bundle branches.

the chordae tendineae.

the Purkinje fibers.

2205. What is the function of resistance vessels?

ensuring the exchange of oxygen and carbon dioxide between blood and tissues

ensuring the return of blood to the heart

are a blood reservoirs in the body

maintaining the proper value of systemic blood pressure, regulation of local blood flow

2206. What is the value of the blood pressure at the arterial end of the capillaries of the systemic circulation?

15-20 mm Hg

6-8 mm Hg
g

30-37 mm Hg

2-4 mm Hg

2207. Choose the correct statement ...

the blood flow velocity is constant in the vascular system

the blood flow rate and blood flow velocity are constant in the vascular system

the blood flow rate is constant in the vascular system,

the blood flow rate and blood flow velocity are not constant in the vascular system

2208. All arteries in the body contain oxygen-rich blood with the exception of

the aorta.

the pulmonary artery.

the renal artery.

the coronary arteries.

2209. What chamber of the heart performs a harder work?

right ventricle

left ventricle

both fulfill the same job

left atrium

2210. The role of the pericardium:

softens friction

forms valves

generates pulses

provides a reduction of chambers in the heart

2211. How will the lumen of the vessels of the skin and internal organs change at a high external temperature?

the vessels of the skin will narrow, the internal organs will expand

the vessels of the skin and internal organs will narrow

the vessels of the skin will expand, the internal organs will narrow

the vessels of the skin will not change, and the internal organs will expand
2212. If the radius of the blood vessel decreases 2 times, and the mean arterial and venous pressure remain unchanged, then ...

blood flow will decrease 4 times

blood flow will decrease 8 times

blood flow will decrease 16 times

blood flow will decrease 2 times

2213. Aorta and arteries are:

vessels of elastic type

muscle vessels

muscle and elastic vessels

armless vessels

2214. The depolarization phase of the action potential of the of contractile cardiomyocytes is due to ...

opening of Na+ channels of the cell membrane and inflow of Na+

opening of Na+ channels of the cell membrane and outflow of Na+

opening of K+ channels of the cell membrane and inflow of K+

opening of K+ channels of the cell membrane and outflow of K+

2215. What processes in the heart does the QRS complex of the electrocardiogram reflect?

depolarization of the atria

atrial repolarization

repolarization of the ventricles

depolarization of the ventricles

2216. The plateau in action potential of a contractile cell is due to:

the rapid entry of sodium into the cell;

sodium inactivation;

passive release of potassium ions;

slow entry of calcium ions in the background of the release of K + ions.

2217. The role of breathing in venous return is called the ___ ____ .

cardiac pump

muscle pump
thoracic pump

all of the above

2218. How does pulse pressure change in small arteries?

does not change

increases

decreases

all answers are correct

2219. Which mechanisms of these acts when blood flow to brain decreased?

Chemoreceptor reflexes.

Baroreceptor reflexes.

CNS ischemic response.

Atrial regulation.

2220. Slow diastolic depolarization in the pacemakers of the heart arises as a result ...

all the answers are correct

closure of K+ channels of the cell membrane

the gradual opening of the slow Ca+ channels of the cell membrane

a gradual increase in the permeability of the cell membrane for Na+

2221. The highest degree of autorhythmicity in:

sinoatrial node;

Bachmann's bundle;

atrioventricular node;

bundle of His.

2222. What is the basis of autorhythmicity of the heart?

Rhythmic oscillation of carbon dioxide and oxygen concentration in coronary vessels

Rhythmic spontaneous depolarization of the atrioventricular node

rhythmic spontaneous depolarization of the bundle of His

rhythmic spontaneous depolarization of the sinoatrial node

2223. The volume of blood each ventricle pumps out during a cardiac cycle is about ___________
70 ml

5000 ml

7l

1200 ml

2224. Stroke volume in a healthy young man at rest is equal ...

60 - 100 ml

200 - 1000 ml

1-3l

3.5 - 5.5 liters

20 - 50 ml

2225. Blood flow velocity is maximal in:

aorta;

arteries;

arterioles;

capillaries.

2226. What is the mechanism of genesis of the first heart sound ...

closing the atrioventricular valves

closing the semilunar valves

relaxation of the ventricles

atrial contraction

2227. Which of these statements is false?

Most of the total blood volume is contained in veins.

Capillaries have a greater total surface area than any other type of vessel.

Exchanges between blood and tissue fluid occur across the walls of venules.

Small arteries and arterioles present great resistance to blood flow.

2228. Which of these statements about tissue fluid is false?

It contains the same glucose and salt concentration as plasma.

It contains a lower protein concentration than plasma.


Its colloid osmotic pressure is greater than that of plasma.

Its hydrostatic pressure is lower than that of plasma.

2229. The action potential of a contractile ventricular cardiomyocyte lasts about ...

1 ms

300 ms

20 ms

3000 ms

2230. The systemic circulation begins:

pulmonary trunk

two pulmonary arteries

the aorta

two vena cava

2231. Cardiac muscle does not exhibit tetanus because it has

scanty sarcoplasmic reticulum.

a long absolute refractory period.

electrical synapses.

exclusively aerobic respiration.

fast Ca2 channels.

2232. What structures are transmitted AP from the atrioventricular node to the bundle of His?

by the Bachman pathways

by atrioventricular node

by Purkinje fibers

by internodal pathways

2233. Which of the following factors have a vasodilator effect?

vasopressin, angiotensin II

norepinephrine, angiotensin I

prostacyclin, nitric oxide

renin, thyroxine
2234. The maximum pressure in the left ventricle during their systole:

18 - 30 mm Hg.

125-130 mm. Hg

60 - 80 mm Hg.

18 - 30 mm Hg.

2235. The methods of recording sound phenomena in the heart include

electrocardiography

echocardiography

ballistocardiography

phonocardiography

2236. The highest arterial blood pressure attained during ventricular contraction is called ____ ___ pressure.

systolic pressure

diastolic pressure

pulse pressure

mean arterial pressure

2237. Blood pressure is lowest in

arteries.

arterioles.

capillaries.

venules.

veins.

2238. This famous physiologist discovered the circulation of blood:

Ernest Starling

William Harvey

Marcello Malpighi

Andreas Vesalius

2239. At what level of the vascular system does the most pronounced reduction in blood pressure occur?

arterioles
arteries

veins

capillaries

2240. The blood moves according to the conditions of cardiac hemodynamics:

against pressure gradient

from the ventricles to the atria

from the atrium to the veins

from the atria to the ventricles

2241. The ST interval of the electrocardiogram corresponds to ...

the spread of excitation through the myocardium of the ventricles

repolarization of the ventricles

slow spread of excitation in the atrioventricular node

atrial repolarization

complete depolarization of the ventricles

2242. At what level of the vascular system is the lowest blood pressure noted?

venules

capillaries

arteries

vena cava

arterioles

2243. In a typical human cardiac cycle, the volume of blood remaining in the left ventricle at the end of ventricular systole is
approximately __________. This is referred to as the end-systolic volume.

10 ml

25 ml

50 ml

100 ml

2244. When will the blood flow velocity increase?

with an increase in the total cross-sectional area of all vessels

with a decrease in the blood flow rate


with an increase in the blood flow rate

with a decrease in the total cross-sectional area of all vessels

2245. The greatest resistance to blood flow is created in:

arteries;

arterioles;

capillaries;

veins.

2246. With age, the maximum consumption of O2 by the myocardium ...

decreases

increases

remains unchanged

all answers are correct

2247. Which of these humoral factors stimulate heart work?

acetylcholine

potassium ions

adrenaline

endothelin

2248. Choose the correct sequence of action potential conduction along the conductive tissues of the heart:

SA node → AV node → Bundle of His → Purkinje fibers → Bundle branches

SA node → AV node → Bundle of His → Bundle branches → Purkinje fibers

SA node → AV node → Bundle branches → Bundle of His→ Purkinje fibers

SA node → AV node → Purkinje fibers → Bundle of His → Bundle branches

2249. The linear velocity of blood flow in the capillaries is equal to:

5 m / s.

0.5 m / s.

0,5mm / sec.

0.3 m / s

2250. At rest the hearth rate is equal ...


50-60 beats / min

60-90 beats / min

75-100 beats / min

80-110 beats / min

2251. Which answer correctly indicates effects of angiotensin II?

severe vasodilation, inhibition of the release of aldosterone from the adrenal glands

severe vasoconstriction, stimulation of the release of aldosterone from the adrenal glands

angiotensin II does not significantly affect vascular tone

angiotensin II is not a vasoactive substance

2252. Who first discovered blood circulation?

K. Galen

I.F. Zion

W. Harvey

I.P. Pavlov

2253. Cells of the sinoatrial node are the primary pacemakers of the heart. In the absence of any input from the autonomic
nervous system, how many action potentials do these cells generate every minute?

30 action potentials per minute

none

70 action potentials per minute

100 action potentials per minute

2254. What changes in heart function can an increase in the patient's blood content of Ca2 + lead to?

an increase in heart rate

to weakening of heart contractions

to increase heart rate

to weakening and shortening of heart contractions

2255. Atrial systole at heart rate = 75 beats / min lasts ...

0.3 s

0.1 s

0.2 s
0.8 s

2256. At what level of branching of the vascular system is the minimum blood flow velocity observed?

arteries

veins

capillaries

arterioles

aorta

2257. Semilunar valves are located in:

the mouth of the aorta

the mouth of the vena cava

the mouth of the pulmonary veins

between the atria and ventricles

2258. Ventricular pressure peaks during

the first heart sound.

the second heart sound.

the QRS complex.

the P–Q segment.

the S–T segment.

2259. Mean arterial pressure mainly depends on ...

contraction of large arteries and relaxation of large veins

the velocity of blood ejection by the ventricle

cardiac output and peripheral vascular resistance

the average blood flow velocity in the aorta

elasticity of the aorta

2260. Nitrogen monoxide (NO) is produced in ...

erythrocytes

macrophages

endothelium
neurons

all answers are correct

2261. During the cardiac cycle, opening of the atrioventricular valves following ventricular relaxation leads to:

an increase in the hydrostatic pressure within the ventricles

a decrease in the hydrostatic pressure within the atria

an increase in the hydrostatic pressure within the atria

an increase in the hydrostatic pressure within the atria

2262. What effect does nitrogen monoxide (NO) have on vascular tone?

vasoconstriction

no effect

vasodilation

2263. During the phase of isovolumetric relaxation of the ventricles, the pressure in the ventricles is

rising.

falling.

first rising, then falling.

constant.

2264. Elderly people have resting heart rate ...

decreases

increases

does not change significantly

all answers are correct

2265. Pacemaker potentials of the SA node result from:

slow Na+ influx

K+ influx

Cl− influx

fast Na+ influx

2266. The velocity of blood flow decreases if

vessel radius increases.


blood pressure increases.

viscosity increases.

afterload increases.

vasomotion decreases.

2267. What component of the microcirculation is missing among the above: arterioles, precapillaries, capillaries, venules,
lymphatic capillaries?

arteriolar-venular anastomoses

arteriovenous anastomoses

arteries

veins

aorta

2268. What processes in the heart does the T wave of the electrocardiogram reflect?

depolarization of the atria

atrial repolarization

repolarization of the ventricles

depolarization of the ventricles

2269. High excitation rate through Purkinje fibers provides:

consistent atrial and ventricular contractions

successive contractions of the right and left ventricles

asynchronous contraction of ventricular myocardium fibers

synchronous contraction of ventricular myocardium fibers

2270. The value of blood pressure depends on:

strength, heart rate

peripheral resistance

volume and viscosity of circulating blood

all of the above

2271. Which part of the central nervous system provides a change in heart rate during mental activity?

medulla oblongata

hypothalamus
hypothalamus

limbic system

cerebral cortex

spinal cord

2272. The blood pressure in the area of the confluence of the veins in the heart is:

120–130 mm Hg;

70–80 mm Hg;

12-15 mm Hg;

about O.

2273. Indicate which of the listed humoral factors has a positive effect on the myocardium:

acetylcholine

bradykinin

metabolites

epinephrine

2274. The Frank-Starling Law states that its ...

the force of contractions increases as the inflow of Ca2 + increases into the cell

the stroke volume is directly proportional to the strength developed by the ventricles in the isometric phase

the stroke volume increases as the myocardium is stretched by end-diastolic pressure

the force of contraction does not depend on the resistance to blood flow

2275. According to the functional classification resistance vessels include:

arteries

venules

capillaries

arterioles

2276. Atrioventricular valves open ...

at the beginning of the diastole of the ventricles

at the beginning of the phase of isovolumetric contraction of ventricles

at the end of the isoovolumetric relaxation of the ventricles

none of this phases and periods


2277. What is approximately the velocity of blood flow in the capillaries?

about 5 mm / s

about 20 mm / s

about 0.5 mm / s

about 10 mm / s

2278. In a healthy person in the right ventricle at the peak of systole and diastole, blood pressure is, respectively, approximately
...

125/80 mm Hg

120/0 mm Hg

25/0 mm Hg

0/25 mm Hg

2279. The blood contained in a ventricle during isovolumetric relaxation is

the end-systolic volume.

the end-diastolic volume.

the stroke volume.

the ejection fraction.

none of these; the ventricle is empty.

2280. First heart tone:

diastolic

is not heard with a stethoscope

deaf, long, deep

short, thin, ringing

2281. In an individual at rest, approximately this percentage of total blood volume can be found in the venous system:

5%

10%

35%

64%

2282. Which of the following substances with only a vasodilator effect?


acetylcholine, serotonin, vasopressin, renin

adrenaline, serotonin, histamine

acetylcholine, prostacyclin, bradykinin, histamine, acidic metabolic products

renin, acidic metabolic products, acetylcholine

2283. According to the Frank-Starling law of the heart, the strength of ventricular contraction is

directly proportional to the end-diastolic volume.

inversely proportional to the end-diastolic volume.

independent of the end-diastolic volume.

2284. Repolarization of the ventricles produces the ____ __ of the electrocardiogram.

QRS complex

P wave

T wave

PR interval

2285. Function of the ventricles:

the throw of blood in the circles of blood circulation;

suction of blood to the ventricles;

hydrodynamic shock for the atria;

unilateral movement of blood through the heart.

2286. Cardiac output during hard muscle exercises in a healthy man is equal ...

8-10 l

5-8 l

25-35 l

10-12 l

2287. The cardiac cycle begins with:

ventricular systole

a general pause

atrial systole

atrial diastoles
2288. What is the value of blood pressure at the venous end of the capillary of the systemic circulation?

30-35 mm Hg

6-8 mm Hg

2-4 mm Hg

15-20 mm Hg

2289. At the peak of the systole, the blood pressure in atria reaches ...

25-30 mm Hg

70-80 mm Hg

4-12 mm Hg.

40-50 mm Hg.

2290. If the hydrostatic pressure in the capillaries and tissues is 30 and 3 mm Hg, and oncotic blood pressure 25 and 2 mm Hg,
respectively, then what is the net filtration pressure?

4 mm Hg

0 mm Hg

2 mm Hg

6 mm Hg

2291. In which example is the highest pulse pressure?

120/80 mm Hg

110/60 mm Hg

130/90 mm Hg

140/95 mm Hg

2292. Where are the baro- and chemoreceptors involved in the regulation of the cardiovascular system located?

aortic arch

carotid sinus

the mouth of the hollow veins

the endocardium of the atria and ventricles

ubiquitous in the cardiovascular system

pulmonary trunk

2293. What is the name of the blood pressure, which reflects the difference between systolic and diastolic pressure?
mean arterial pressure

systolic pressure

diastolic pressure

pulse pressure

2294. The capillaries of skeletal muscles are of the structural type called __ _____

sinosoids

continuous capillaries

fenestrated capillaries

arterioles

2295. Atrioventricular valves separate:

the ventricles from the main arteries;

atria from the ventricles;

vena cava from the right atrium;

pulmonary veins from the left atrium.

2296. Which one of these is the effect of angiotensin II ?

Vasoconstriction of afferent artery.

Vasodilation of efferent artery.

Vasoconstriction of efferent artery.

Vasoconstriction of both efferent and afferent.

2297. Bradycardia is observed if the heart rate is less ...

60 beats / min

75 beats / min

80 beats / min

90 beats / min

2298. The volume of blood pumped per minute by the left ventricle is

greater than the volume pumped by the right ventricle.

less than the volume pumped by the right ventricle.

the same as the volume pumped by the right ventricle.


either less or greater than the volume pumped by the right ventricle, depending on the strength of contraction.

2299. What changes in heart function can result in a significant increase in the patient's blood content of K +?

to an increase and increase in heart rate

to increase heart rate

to weakening of heart contractions

an increase in heart rate

2300. How does irritation of the sympathetic nerve affect the excitability and conduction of the heart?

reduces

does not affect

increases

excitability - decreases, conductivity – increases

2301. What innervation does most of the vessels lack?

sympathetic

trophic

parasympathetic

peripheral

2302. What is the average mean arterial pressure?

45-50 mm Hg

80-90 mm Hg

120-125 mm Hg

90-100 mm Hg

2303. Which parts of the cardiovascular system contain up to 60-70% of the total blood volume?

arteries

capillaries

veins

aorta

2304. Which of the following substances with only a vasoconstrictor effect?

adrenaline, acetylcholine, renin, histamine, vasopressin


norepinephrine, serotonin, acidic metabolic products

bradykinin, prostacyclin, acetylcholine

norepinephrine, vasopressin, angiotensin

2305. If a patient has high systolic blood pressure with normal or low diastolic blood pressure, it can be assumed that ...

increased peripheral vascular resistance as a result of narrowing of arterioles

increased stroke volume and decreased aortic stiffness

decreased stroke volume and peripheral vascular resistance

increased cardiac output and slightly decreased peripheral vascular resistance

2306. An increase in arterial blood pressure occurs in all cases except ...

increasing the tone of smooth muscles of the arteries

an increase in the excitability of aortic baroreceptors

an increase in the cardiac output

reducing the excitability of aortic baroreceptors

2307. How does the velocity of blood flow from the aorta to the vena cava change?

remains constant at all levels of the vascular system

increases to the level of capillaries, then decreases

decreases to the level of capillaries, then increases

gradually decreases at all levels of the vascular system

2308. The duration of the cardiac cycle is considered by the ECG interval:

P–Q

Q–T

T–P

R–R

2309. In which of the vessels 30 mm long, the resistance to blood flow will have the greatest value if the radius of the vessel is
...

4 mm

6 mm

2 mm

5 mm
2310. The sequence of contractions first atrial and then ventricles is due to:

the work of semilunar valves

sineatrial node pacemaker function

pressure gradient between the atria and ventricles

atrioventricular delay

2311. If you damage the sympathetic and parasympathetic nerves going to the heart, then ...

heart rate will decrease

heart rate will increase

the rhythm of the atrioventricular node will appear

slow diastolic depolarization in the sinus node stops

the heart will stop beating

2312. Peristaltic waves of contraction move fluid within which of these vessels?

Arteries

Veins

Capillaries

Lymphatic vessels

All of these

2313. In which part of the central nervous system is the cardiovascular center located?

in the spinal cord

in the hypothalamus

in the medulla oblongata

in the thalamus

2314. How does the blood flow rate change in different types of the vascular system?

more in arteries and less in veins

the largest in the aorta and large arteries

does not change

the largest in the veins and the smallest in the arteries


2315. The Frank-Starling law is:

heart failure;

violation of the activity of the heart;

increase in heart rate;

an increase in the strength of heart contractions in response to the stretching of the heart chambers.

2316. What is the reason for the increase in the cardiac output during exercise in an untrained person?

predominantly an increase in heart rate

predominantly an increase in systolic blood volume

an increase in heart rate and a decrease in systolic volume

a decrease in heart rate and an increase in systolic volume

2317. The atria contract during

the first heart sound.

the second heart sound.

the QRS complex.

the P–Q segment.

the S–T segment.

2318. Pulse pressure is:

minimum pressure in the arteries

pressure in the arteries

the difference between systolic and diastolic pressure

maximum pressure in the arteries

2319. Atrial systole lasts:

0.1 s

0.8 s.

0.33 s

0.47 s

2320. What is the hemodynamic law calculated by the formula Q = ∆P / R?

blood viscosity
blood flow velocity

vessel length

blood flow rate

2321. Under normal conditions, the excitation of the myocardiac cells of the ventricles does not spread to the atrium, since ...

the duration of the plateau phase and the refractory period of the excitatory cells is greater than the contractile cells

excitation in an atrioventricular node usually goes only in one direction

atrial muscle fibers are at this time in the refractory period

ventricular muscles fibers are at this time in the refractory period

2322. In an individual at rest, approximately this percentage of total blood volume can be found in all capillaries:

5%

10%

35%

65%

2323. A large duration of the action potential of contractile cardiomyocytes depends on the plateau phase due to ...

extended sodium activation time

opening of K+ channels of the cell membrane and inflow of Ca2+

delayed sodium activation, delaying opening of K+ channels of the membrane and the delaying of repolarization

extended potassium activation time

2324. What is the mechanism of genesis of the fourth heart sound?

vibration of the walls of the ventricles when filling with blood

slamming the semilunar valves

turbulence of blood flow

vibration of the atrial walls during their contraction

2325. What does the duration of the PQ segment, equal to 0.25 seconds, indicate?

on the acceleration of the conduction of excitation from the atria to the ventricles

on the slowing down of the conduction of excitation in the musculature of the ventricles

about slowing down the conduction of excitation from the atria to the ventricles

on the acceleration of the conduction of excitation through the atria


2326. The spike phase of the action potential of the sinoatrial node pacemaker cells of the heart is caused by:

Opening of voltage-gated Na+ channels

Opening of voltage-gated Ca2+ channels

Closure of voltage-gated K+ channels

Opening of voltage-gated Cl− channels

2327. How will the blood pressure change if, as a result of slow blood loss, the amount of circulating blood has decreased by
10%?

will not change

decrease

will increase

all answers are correct

2328. Parasympathetic stimulation of the heart has the following effect(s):

Negative chronotropic effect

Positive chronotropic effect

Negative inotropic effect

Positive inotropic effect

2329. Of the conductive tissues of the heart, only these cells can directly stimulate cardiac myocytes to contract:

Cells of the sinoatrial (SA) node

Cells of the atrioventricular (AV) node

Fibers of the bundle of His

Fibers of the left and right bundle branches

2330. What factor does peripheral vascular resistance mainly depend on?

on the length of the vessel

from the radius of the vessel

from blood viscosity

from the area of the vessel

2331. How does the blood pressure change in the vascular system from the aorta to the vena cava?

remains constant throughout the entire vascular system

decreases throughout the entire vascular system


decreases to the level of capillaries, then rises

rises to the level of capillaries, then decreases

2332. Autoregulation of blood flow in the vessels means ...

maintaining the constancy of the blood flow rate

the effect of plasma accumulation and a decrease in hematocrit in small vessels

relatively constant blood flow rate despite fluctuations in blood pressure

maintaining a constant tone of the vasomotor nerves

2333. What is the name of the myocardial excitability phase, which correlates to the plateau phase of action potential?

relative refractory period

the phase of supernormal excitability

absolute refractory period

phase of subnormal excitability

2334. During the isovolumic relaxation (also referred to as isovolumetric relaxation) phase of the cardiac cycle:

The hydrostatic pressure in the left ventricle is higher than the hydrostatic pressure in the left atrium.

The hydrostatic pressure in the left ventricle is lower than the hydrostatic pressure in the left atrium.

The hydrostatic pressure in the left ventricle is higher than the hydrostatic pressure in the aorta.

The hydrostatic pressure in the left ventricle is lower than the hydrostatic pressure in the aorta.

2335. Indicate violation of myocardial contractility:

tachycardia

sinus arrhythmia

heart failure

premature contraction

2336. During a typical cardiac cycle under resting conditions, approximately __________ of the volume of blood present in the
ventricles just before ventricular contraction (referred to as enddiastolic volume), enters the ventricles as a result of passive
venous return of blood flowing through the atria and into the ventricles.

10%

20%

50%

80%
2337. The normal pulse pressure value is:

110-30 = 70 mmHg

140-60 = 80 mmHg

110-70 = 40 mmHg

100-90 = 10 mmHg

2338. At what type of blood pressure do Korotkoff sounds appear when measuring blood pressure?

with diastolic

with systolic

with pulse

with stroke

2339. Electrocardiography is a method of assessing ...

the work of the heart

the strength of the heart

excitement of the heart

heart contractions

2340. What is the function of the heart:

blood depot;

organ hematopoiesis;

double action pump;

organ excretion.

2341. The cardiac events that occur from the beginning of one heart beat to the beginning of the next heart beat are called the
cardiac cycle. Which of the following is/are not true?

The left atrium and right atrium contract nearly simultaneously.

The left ventricle and right ventricle contract nearly simultaneously.

The atria contract before the ventricles.

The ventricles contract before the atria.

2342. A properties of action potential of a contractile cell is:

the value of action potential;

depolarization phase;
the presence of a peak point in action potential;

slow repolarization - plateau.

2343. During the isovolumetric contraction phase of the cardiac cycle:

The atrioventricular valves and semilunar valves are closed.

The atrioventricular valves and semilunar valves are open.

The atrioventricular valves closed and semilunar valves are open.

The atrioventricular valves open and semilunar valves are closed.

2344. The pressure sensors in the major arteries near the head are called __ ___ __ .

chemorecetors

baroreceptors

mechanoreceptors

all of the above

2345. In a typical ventricular myocyte, the refractory period is approximately:

1 ms

5 ms

10 ms

100 ms

250 ms

2346. During the isovolumetric relaxation phase of the cardiac cycle:

The atrioventricular valves and semilunar valves are closed.

The atrioventricular valves and semilunar valves are open.

The atrioventricular valves closed and semilunar valves are open.

The atrioventricular valves open and semilunar valves are closed.

2347. Approximately how much pressure drops in the left ventricle during diastole?

100 mm Hg

0 mm Hg

80 mm Hg

40 mm Hg
120 mm Hg

2348. What structures are transmitted AP from a sinus node to an atrio-ventricular node?

by the Bakhman and internodal pathways

by the bundle of His

by Purkinje fibers

by the right branch of the bundle of His

2349. The main function of the capillaries is:

ensuring metabolism in organs and tissues

discharge of blood from arterioles into venules, bypassing capillaries

the transformation of intermittent blood flow in continuous

the transformation of the laminar flow of blood into a turbulent

2350. Which part of the central nervous system provides an increase in heart rate during emotional reactions?

medulla oblongata

the limbic system

cerebral cortex

spinal cord

2351. What structures are the excitement from the bundle of His to the myocardiac cells of ventricles?

by Purkinje fibers

by internodal pathways

by atrioventricular node

by the Bachman pathways

2352. The endings of the vagus nerve that innervates the heart are secreted ...

adrenaline

serotonin

histamine

acetylcholine

2353. The duration of the ventricular systole at heart rate = 75 beats / min is ...

0.44 s
0.23 s

0.8 s

0.33 s

0.52 s

2354. Tachycardia is observed if the heart rate is more ...

60 beats / min

75 beats / min

80 beats / min

90 beats / min

2355. All efferent fibers of the vasomotor center belong to the ___ ____ division of the autonomic nervous system.

parasympathetic

sympathetic

enteric

all of the above

2356. Factors contributing to the continuity of blood flow through the vessels:

elasticity of large arteries, resistivity of small vessels

cyclical activity of the heart

anastomoses work

the presence of sphincters and valves in the heart and veins

2357. Cardiac output in a healthy young man at rest is equal ...

1.5-2, 0 l

2.0-3.0 l

3.5-5.0 l

6-7 l

2358. What is the systolic / diastolic pressure in the pulmonary artery under normal conditions?

120/80 mm Hg

80/40 mm Hg

25/10 mm Hg
150/100 mm Hg

2359. Cardiac output is determined by _______

heart rate

stroke volume

heart rate and stroke volume

blood flow

2360. Which of the following is correct?

the blood flow velosity in large veins is greater than in capillaries

the total cross-sectional area of large veins is much smaller than that of capillaries

all answers are correct

the blood flow velocity in the aorta is greater than in the vena cava

2361. Where is the cardiovascular center located?

in the upper thoracic segments of the spinal cord

in the medulla oblongata

in the pons

in the hypothalamus

in the upper cervical segments of the spinal cord

2362. Indicate the pressure at which the semilunar valves of the pulmonary trunk open:

25 to 30 mmHg

10 to 15 mmHg

60-80 mmHg

120-130 mmHg

2363. Which of the indicated humoral factors inhibit the work of the heart?

norepinephrine

calcium ions

acetylcholine

gastrin

ions Mg2 +
2364. Common for cardiac and skeletal muscle fibers is ...

autorhythmicity of cells

the presence of intercalated disks

the dependence of action potentials from concentration gradients of K+ and Na+

the presence of the "plateau" phase

2365. What are Korotkoff sounds?

a sound phenomenon that occurs above the artery when it is incompletely pressed

the sound accompanying the work of the heart

the sound phenomenon that occurs when the veins are pressed

the sound phenomenon that occurs above the artery when it is fully pressed

2366. Blood flows faster in a venule than in a capillary because venules

have one-way valves.

exhibit vasomotion.

are closer to the heart.

have higher blood pressures.

have larger diameters

2367. Closing of the __ ____ valves produces turbulence in the bloodstream, which contributes to heart sound S2.

pulmonary

semilunar

aortic

mitral

2368. What neurotransmitters are involved in the transmission of excitation from nerves to the heart?

glycine, gamma-aminobutyric acid

norepinephrine, acetylcholine

dopamine

histamine, serotonin

2369. The main cardiovascular (vital) center is located at:

spinal cord
the midbrain

medulla oblongata

cerebral cortex

2370. Slow diastolic depolarization is characteristic to cells of ...

cardiac muscle fibers

skeletal muscle fibers

pacemakers

nervous fibers

2371. Where is the center of the sympathetic innervation of the heart?

in the upper cervical segments of the spinal cord

in the upper thoracic segments of the spinal cord

in the medulla oblongata

in the pons

2372. The sympathetic nerve fibers that innervate the heart secrete ...

acetylcholine

histamine

norepinephrine

serotonin

2373. What is extrasystole?

another contraction of the heart

increased heart contraction

extraordinary contraction of the heart

regular contraction of the atria

2374. Atrioventricular delay:

regulates the strength of contractions of the heart

provides blood flow to the heart

provides consistent atrial contractions and ventricles

sets the heart rhythm


2375. In a healthy adult in the left ventricle at the peak of systole and diastole, blood pressure is, respectively, about ...

150/100 mm Hg

120/60 mm Hg

120/0 mm Hg

120/80 mm Hg

2376. Sympathetic stimulation of the heart has the following effect(s):

Negative chronotropic effect

Positive chronotropic effect

Negative inotropic effect

Positive inotropic effect

2377. The contraction of myocardial cells is ...

isometric

isotonic

auxotonic

all the answers are correct

2378. The velocity of excitation in Purkinje fibers is:

0.05 m / s.

0.3 - 0.5 m / s.

0.02 - 0.04 m / s.

2 - 4 m / s.

2379. What is the reason for the increase in the cardiac output during physical activity in a trained person?

predominantly an increase in heart rate

predominantly an increase in systolic blood volume

an increase in heart rate and a decrease in systolic volume

a decrease in heart rate and an increase in systolic volume

2380. At the peak of systole in the right ventricle, blood pressure reaches about ...

70-80 mm Hg

120-130 mm Hg
50-60 mm Hg

25-30 mm Hg

2381. What is the name of the process of rhythmic spontaneous excitation of the heart?

Autorhythmicity

conductivity

Refractory period

contractility

2382. Increasing the tone of the vagus nerves during sleep:

strengthens and improves the functioning of the heart

weakens and slows down the heart

does not have a noticeable effect on the heart

has a phase reinforcing - weakening effect

2383. Blood pressure in the arterial part of the capillary is equal to:

70–80 mm Hg.

15–20 mm Hg.

35–40 mmHg

120–130 mm Hg

2384. Blood pressure in an adult (18–45 years old) normally amounts to:

100/50 mmHg

120/80 mm Hg.

70/40 mm Hg.

170/90 mm Hg.

2385. Which part of the central nervous system provides an increase in heart rate with an increase in body temperature?

cerebral cortex

hypothalamus

the limbic system

spinal cord

medulla oblongata
2386. Which of the following factors causes an increase in local blood flow?

an increase in the concentration of CO2 in the tissues

a decrease in the concentration of CO2 in the tissues

an increase in the concentration of O2 in the tissues

a decrease in the concentration of lactic acid in the tissues

2387. What hormone is of particular importance in regulating the activity of the heart under conditions of physical and
emotional stress?

vasopressin

adrenaline

histamine

serotonin

insulin

2388. The second heart sound immediately follows the occurrence of

the P wave.

the QRS wave.

the T wave.

2389. The aortic valves are opened at pressure in the left ventricle ...

more than 120-130 mm Hg

more than 25-30 mm Hg

more than 40-50 mm Hg

more than 80-90 mm Hg

2390. The pacemaker potential of the SA node cells results from the slow inflow of ___ ___.

Ca ions

K ions

Na ions

Cl ions

2391. The basis of the autorhythmicity of the heart is:

potassium hyperpolarization

resistant sodium depolarization


interaction of acetylcholine with M- cholinergic receptors

spontaneous slow diastolic depolarization

2392. In the right ventricle, the ejection of blood begins at a pressure ...

120-130 mm Hg

70-80 mm Hg

8-12 mm Hg

40-60 mm Hg

2393. The greatest resistance to blood flow occurs in

large arteries.

medium-size arteries.

arterioles.

capillaries.

2394. Which of the following is true of ventricular filling phase?

Atrioventricular valves are open

Semilunar valves are open

Atria contract all throughout this phase

All heart valves are close

2395. What is approximately the total exchange surface area of capillaries?

10 000 m2

100 m2

600 m2

300 m2

2396. What heart ventricle pumps blood under great pressure?

right

no differences

identical pressure

left or right depending on the tension

left
2397. The largest total cross-sectional area of blood vessels is found in this region along the vasculature:

Small arteries

Arterioles

Capillaries

Venules

Veins

2398. Autorhythmicity has:

myocardial contractile cell

pericardium

myocardium conductive cell

endocardium

2399. Blood pressure in the left ventricle of the heart is lowest during ...

isovolumetric contraction

slow ejection

the start of rapid filling phase

the of slow filling phase

2400. Endothelin is ...

analogue of prostacyclin

vasodilator peptide released by the endothelium

vasoconstrictor peptide released by the endothelium

vasoconstrictor peptide of the adrenal medulla

2401. In human being the duration of cardiac cycle is ___________

0.008 sec

0.5 sec

0.8 sec

8 sec

2402. The first heart sound, is produced by closing of

the aortic semilunar valve.


the pulmonary semilunar valve.

the tricuspid valve.

the bicuspid valve.

both AV valves.

2403. What processes in the heart does the P wave of the electrocardiogram reflect?

atrial repolarization

depolarization of the atria

repolarization of the ventricles

depolarization of the ventricles

2404. What is the mechanism of the genesis of the third heart sound?

vibration of the walls of the atria when they are filled with blood

vibrations of the atrial walls during their contraction

vibration of the walls of the ventricles when they are filled with blood

turbulence of blood flow

2405. What is the mechanism of genesis of the second heart sound ...

closing the AV valves

closing the semilunar valves

contraction of the ventricles

atrial contraction

2406. What ventricle of the heart pumps a larger blood volume per 1 minute?

Left

right

identical volumes

during exercise - right

2407. To get from the right atrium to the right ventricle, blood flows through

the pulmonary valve.

the tricuspid valve.

the bicuspid valve.

the aortic valve


the aortic valve.

the mitral valve.

2408. During the filling phase of the ventricles with blood atrioventricular valves ...

closed

open

left closed, right open

right closed, left open

2409. The high pressure zone of the cardiovascular system includes:

left ventricle, aorta, arteries and arterioles of the systemic circulation

right ventricle, arteries and arterioles of the pulmonary circulation

right atrium, venules and veins of the systemic circulation

right atrium, left atrium and pulmonary circulation

2410. Disturbances of excitability in the heart manifest themselves:

blockade;

premature contraction;

decompensation;

tachycardia.

2411. Nitrogen monoxide (NO) is produced in the greatest amount by ...

erythrocytes

macrophages

endothelium

neurons

all answers are correct

2412. Spontaneous impulses in the sinoatrial node occur with a frequency ...

20 imp / min

40-50 imp / min

30-40 imp / min

60-80 imp / min


2413. The circulatory route from aorta to the venae cavae is the _ _____ circuit.

systemic

pulmonary

coronary

enteric

2414. If the radius of the vessel has doubled, and the blood pressure remains unchanged, then ...

the average blood flow rate increases 4 times

blood flow will decrease 16 times

blood flow increases 16 times

2415. Arterial blood pressure increases under the influence of all following factors, except ...

an increase in the stroke volume of the heart

increasing rate of blood ejection by the left ventricle

an increase in the peripheral vascular resistance

a decrease in the peripheral vascular resistance

decrease in the elasticity of the aortic wall

2416. The RQ interval on the ECG shows:

the spread of excitation in the ventricles

the spread of excitation in the atria and through the atrioventricular node

general pause of the heart

atrial cycle duration

2417. How will the lumen of the vessels of internal organs change at low external temperatures?

decrease

will increase

will not change

all answers are correct

2418. At what blood pressure do Korotkoff sounds disappear when measuring blood pressure?

with diastolic

with systolic
with lateral

with pulse

2419. The lowest attained during ventricular relaxation is called ____ ___ pressure.

systolic pressure

diastolic pressure

pulse pressure

mean arterial pressure

2420. What hormone that reduces the reabsorption of Na + and Cl- in the nephron tubules is synthesized in atrial myocytes?

endothelin

vasopressin

natriuretic peptide

adrenaline

2421. When the volume of the ventricles of the heart is the smallest?

during the closure of atrioventricular valves

at the lowest pressure in the atria

during the first heart sound

in the period of common relaxation

during the closure of the semilunar valves

2422. The first heart sound is produced at

the beginning of systole.

the end of systole.

the beginning of diastole.

the end of diastole.

2423. What pressure can be measured by palpation?

diastolic

systolic and diastolic

systolic

pulse
2424. Blood normally flows into a capillary bed from

the distributing arteries.

the conducting arteries.

a metarteriole.

a thoroughfare channel.

the venules.

2425. Within the cardiovascular system, the highest hydrostatic blood pressure is found in the:

Left ventricle during ventricular systole

Left ventricle during ventricular diastole

Left ventricle during atrial systole

Left ventricle during atrial diastole

2426. What is the heart rate if the value of the R-R interval on the ECG is 1 sec?

100 bpm

90 bpm

60 beats / min

120 bpm

2427. The plateau phase of the cardiac action potential is due to the:

opening of voltage-gated Ca2+ channels

opening of fast voltage-gated Na+ channels

opening of voltage-gated K+ channels

opening of voltage-gated K+ and Ca2+ channels

2428. A compensatory pause occurs when:

atrial extrasystoles

ventricular extrasystoles

tachycardia

bradycardia

2429. In a healthy adult man at the peak of systole in the left ventricle, blood pressure reaches about ...

70-80 mm Hg
120-130 mm Hg

25-30 mm Hg

40-50 mm Hg

2430. The increase in the volume of circulating blood during physical activity is mainly provided by ...

the flow of interstitial fluid into the vessels

an increase in heart rate and strength

the flow of blood from the blood reservoirs

activation of erythropoiesis

2431. The P wave of the ECG represents _______________

ventricular depolarization

atrial depolarization

ventricular repolarization

atrial repolarization

2432. In which of the phases or periods of the heart cycle, all the heart valves are at the same time open ...

in the phase of isovolumetric contraction

during the period of rapid filling of ventricles

none of the phases of the cardiac cycle

at the beginning of isovolumetric relaxation

2433. Таламус является…

регулятором всех двигательных функций

регулятором мышечного тонуса

коллектором афферентных путей, высшим центром болевой чувствительности

регулятором гомеостаза

2434. Какую функцию выполняют вегетативные ганглии?

обеспечивают передачу возбуждения с преганглионарных волокон на постганглионарные

рефлекторную (в нем могут замыкаться периферические рефлекторные дуги)

все ответы правильные

транзиторную (через ганглий могут проходить транзитом афферентные и эфферентные волокна)

обладают свойствами нервных центров


обладают свойствами нервных центров

2435. Где располагаются вставочные нейроны соматической рефлекторной дуги спинномозгового рефлекса?

в передних рогах спинного мозга

в боковых рогах спинного мозга

в задних рогах спинного мозга

в спинномозговых узлах

2436. Голод и жажда определяются деятельностью

Гиппокампа

Гипоталамуса

Спинного мозга

Мозжечка

Полосатого тела

2437. Метод вызванных потенциалов - как метод исследования ЦНС применяется для изучения

Интенсивности биохимических процессов в структурах головного мозга

Суммарной электрической активности мозга

Состояния афферентных путей головного мозга

Химических основ работы нейрона

2438. Кто сформулировал основные принципы рефлекторной теории?

Р. Декарт

Я. Прохазка

И.М. Сеченов

И.П. Павлов

2439. По каким волокнам импульсы из спинного мозга поступают к мышечным веретенам?

Альфа-афферентным

Гамма-эфферентным

Гамма-афферентным

Волокнам группы С

Волокнам группы В
2440. Скорость распространения возбуждения в преганглионарных симпатических и парасимпатических нервных
волокнах равна :

70-120 м/с

20-50 м/с

30-70 м/с

3-15 м/с

0,5-2,0 м/с

2441. Эмоциональная реакция определяется деятельностью

Гиппокампа

Спинного мозга

Гипоталамуса

Мозжечка

Полосатого тела

2442. ГАМКергические нейроны …

способны вызывать пресинаптическое торможение

способны вызывать постсинаптическое торможение

блокируются стрихнином

блокируются столбнячным токсином

все ответы правильные

2443. Нервные центры обладают…

низкой чувствительностью к недостатку кислорода

отсутствием чувствительности к недостатку кислорода

высокой чувствительностью к недостатку кислорода

2444. Где «замыкается» дуга ахиллова рефлекса?

В крестцовом отделе спинного мозга

В продолговатом мозге

В шейном отделе спинного мозга

В среднем мозге
2445. В чем заключается роль синапсов ЦНС?

являются местом возникновения возбуждения в ЦНС

проводят токи покоя

передают возбуждение с одного нейрона на другой

формируют потенциал покоя нервной клетки

2446. Правое полушарие у правшей обеспечивает

Эмоциональные состояния

Интегративные функции

Образное мышление

Анализ сенсорной информации

Функции речи

2447. При поражении мозжечка имеет место:

Расстройство психики

Нарушение памяти

Атаксия

Расстройство зрения и слуха

2448. Какие из перечисленных структур являются центральными звеньями лимбической системы?

обонятельная луковица, передние ядра таламуса

таламус, свод, септальная область

гиппокамп, миндалина, сводчатая извилина

обонятельный тракт, крючок, поясная извилина

2449. Как называется торможение нейронов собственными импульсами, поступающими по коллатералям аксона к
тормозным клеткам?

вторичным

реципрокным

поступательным

возвратным

латеральным
2450. Эффекты повышения артериального давления и увеличения частоты сокращений сердца могут быть получены
при раздражении:

3адней группы ядер гипоталамуса.

Любой группы ядер гипоталамуса

Передней группы ядер гипоталамуса.

3адней группы ядер гипоталамуса.

В области воронки гипоталамуса

2451. Какие основные физиологические функции глии?

опорная, трофическая, изолирующая

интегративная

обеспечение процесса центрального торможения

оказание восходящего активирующего влияния на кору головного мозга

все ответы правильные

2452. По каким волокнам импульсы из мышечных веретен поступают в спинной мозг?

Альфа-афферентным

Волокнам группы С

Волокнам группы В

Гамма-афферентным

Гамма-эфферентным

2453. Чем обусловлено возникновение в нервном центре пространственной суммации?

увеличением частоты следования импульсов по афферентному входу

высокой утомляемостью

односторонностью проведения возбуждения

одновременным возбуждением нескольких близко расположенных синапсов

все ответы правильные

2454. При блокаде симпатической иннервации можно наблюдать

Расширение кровеносных сосудов кожи

Расширение зрачков

Снижение артериального давления


Уменьшение частоты сердечных сокращений

2455. Какой чувствительностью обладают нервные центры к действию различных химических веществ?

низкой

практически нечувствительны

вообще нечувствительны

высокой

2456. На каком уровне необходимо перерезать ствол мозга для получения у животного состояния децеребрационной
ригидности?

Выше ядер переднего двухолмия

Ниже уровня красных ядер

Выше уровня красных ядер

На уровне черной субстанции

2457. Какой из отделов вегетативной нервной системы может оказывать на один и тот же орган двоякое влияние?

симпатический

симпатический и парасимпатический

никакой

парасимпатический

2458. Как называется схождение различных путей проведения нервных импульсов к одной и той же нервной клетке?

дивергенция

трансформация

суммация

конвергенция

пролонгирование

2459. Торможение в ЦНС – это…

активный нервный процесс, ослабляющий или прекращающий возбуждение, а также препятствующий его
возникновению

пассивный процесс, связанный с развитием утомления

активный нервный процесс, сопровождающийся развитием утомления

все ответы правильные


2460. Чем обусловлено возникновение в нервном центре временной суммации?

одновременным возбуждением нескольких близко расположенных синапсов

увеличением частоты следования импульсов по афферентному входу

высокой утомляемостью

односторонним проведением возбуждения

все ответы правильные

2461. «Лифтные» рефлексы относятся к…

статокинетическим

позно-тоническим

выпрямительным

статическим

2462. Какая из указанных структур ЦНС осуществляет активирующее влияние на кору больших полушарий и
координирует рефлекторную деятельность спинного мозга?

стриопаллидарная система

гипоталамус

ретикулярная формация

подкорковые ядра

2463. Коленный рефлекс возникает при ударе молоточка по…

сухожилию четырехглавой мышцы бедра ниже надколенника

сухожилию четырехглавой мышцы бедра выше надколенника

надколеннику

четырехглавой мышце бедра

2464. Какие из ниже перечисленных функций обеспечивает большой круг Пейпеса?

агрессивно-оборонительное поведение

процессы обучения и памяти

пищевое поведение

сексуальные формы поведения

2465. Рефлекторные реакции какого отдела ЦНС имеют непосредственное отношение к поддержанию позы, жеванию,
глотанию пищи, секреции пищеварительных желез, дыханию, деятельности сердца, регуляции тонуса сосудов?
среднего мозга

таламуса

заднего мозга

спинного мозга

2466. Могут ли быть на теле одного нейрона и возбуждающие, и тормозные синапсы?

Нет, не могут

Только на нейронах спинного мозга

Могут быть на нейронах всех отделов мозга

Только на нейронах вегетативной нервной системы

Только на мотонейронах

2467. Какое количество нейронов включает эфферентная часть вегетативной рефлекторной дуги?

2 нейрона

1 нейрон

3 нейрона

4 нейрона

2468. Обозначьте вид торможения, представленного на рисунке.

Реципрокное

Возвратное торможение

Постсинаптическое

Пресинаптическое

2469. При двусторонней перерезке блуждающих нервов можно наблюдать

Расширение просвета бронхов

Увеличение кровяного давления

Увеличение частоты сердечных сокращений

Уменьшение слюноотделения

2470. Ядерно-магнитный резонанс как метод исследования ЦНС применяется для изучения

Интенсивности биохимических процессов в структурах головного мозга

Химических основ работы нейрона

Состояния афферентных путей головного мозга


Суммарной электрической активности мозга

2471. Кто впервые предложил термин «рефлекс» использовать в физиологии?

Р. Декарт

Я. Прохазка

И.М. Сеченов

И.П. Павлов

2472. Кто впервые выдвинул представление о рефлекторной деятельности?

Я. Прохазка

И.М. Сеченов

И.П. Павлов

Р. Декарт

2473. Какое влияние оказывает парасимпатический отдел вегетативной нервной системы на слюнные железы?

усиление секреции более концентрированной слюны

торможение секреции слюны

усиление секреции менее концентрированной слюны

все ответы правильные

2474. Как называется явление, при котором возбуждение одной мышцы сопровождается торможением центра
мышцы-антагониста?

отрицательной индукцией

реципрокным торможением

утомлением

облегчением

окклюзией

2475. Что такое рефлекс?

ответная реакция организма на действие раздражителей при обязательном участии высших отделов нервной
системы

ответная реакция организма на действие раздражителей без участия нервной системы

ответная реакция организма на действие раздражителей при обязательном участии нервной системы

ответная реакция организма на действие раздражителей при возможном участии нервной системы
2476. Где локализуются центры насыщения?

в медиальной группе ядер гипоталамуса

в латеральной группе ядер гипоталамуса

в передней группе ядер гипоталамуса

в задней группе ядер гипоталамуса

2477. Кто впервые предположил рефлекторный характер психических процессов?

Р. Декарт

Я. Прохазка

И.М. Сеченов

И.П. Павлов

2478. Какая из функций стриопаллидарной системы является наиболее важной?

регуляция тонуса мышц

регуляция реакций организма на восприятие раздражений (соматических, слуховых, зрительных)

регуляция вегетативных функций

участие в формировании, хранении и реализации сложных двигательных программ

2479. Быстрый сон определяется деятельностью

Гиппокампа

Мозолистого тела

Варолиева моста

Височной коры

2480. Как называются рефлексы, обеспечивающие равновесие при изменении скорости и направления движения?

статические

статокинетические

кинетические

вегетативные

2481. Где «замыкается» дуга коленного рефлекса?

В крестцовом отделе спинного мозга

В продолговатом мозге
В поясничном отделе спинного мозга

В среднем мозге

2482. Лимбические отделы коры обеспечивают

Эмоциональные состояния

Интегративные функции

Функции речи

Образное мышление

Анализ сенсорной информации

2483. В каких взаимоотношениях находятся между собой симпатический и парасимпатический отделы вегетативной
нервной системы?

полного синергизма

полного антагонизма

относительного антагонизма и синергизма

2484. Эмоциональная память определяется деятельностью

Гиппокампа

Мозолистого тела

Варолиева моста

Височной коры

2485. Нервная клетка выполняет

Интегративную функцию

Трофическую функцию

Функцию синтеза медиатора

Функции генерации потенциала действия

Все перечисленные функции

2486. Возбуждение в нервном центре распространяется…

от эфферентного нейрона через промежуточный к афферентному

от промежуточного нейрона через эфферентный к афферентному

от афферентного нейрона через промежуточный к эфферентному

от промежуточного нейрона через афферентный к эфферентному


от промежуточного нейрона через афферентный к эфферентному

2487. Как называется способность нейрона устанавливать многочисленные синаптические связи с различными
нервными клетками?

конвергенция

трансформация ритма

дивергенция

суммация

пролонгирование

2488. Где «замыкается» дуга глотательного рефлекса?

В крестцовом отделе спинного мозга

В продолговатом мозге

В шейном отделе спинного мозга

В среднем мозге

2489. Двустороннее поражение гиппокампа сопровождается нарушением:

Памяти

Движений

Сознания

Восприятия устной и письменной речи

2490. Обозначьте вид торможения, представленного на рисунке.

Реципрокное

Возвратное торможение

Постсинаптическое

Пресинаптическое

2491. Укажите скорость распространения возбуждения в постганглионарных симпатических нервных волокнах:

70-120 м/с

20-50 м/с

30-70 м/с

3-15 м/с

0,5-2,0 м/с
2492. Как называется способность нервного центра к восстановлению функций после повреждения части нейронов
центра?

трансформация ритма возбуждений

пластичность

лабильность

утомляемость

2493. Где расположены тела преганглионарных нейронов симпатического отдела нервной системы?

В среднем мозге, продолговатом мозге и крестцовом отделе спинного мозга.

В шейном и грудном отделах спинного мозга.

В продолговатом мозге и крестцовом отделе спинного мозга.

В боковых рогах грудного и поясничного отделов спинного мозга.

В передних рогах грудного и поясничного отделов спинного мозга.

2494. Кем было открыто явление центрального торможения?

А.А. Ухтомским

И.П. Павловым

И.М. Сеченовым

Ч. Шеррингтоном

2495. Какое влияние оказывают симпатические нервы на тонус сосудов сердца?

повышают

снижают

не влияют

2496. Что понимают под пролонгированием?

беспорядочное распространение возбуждения в ЦНС

направленное распространение возбуждения в ЦНС

увеличение времени импульсации на выходе из нервного центра по сравнению с длительностью раздражения

уменьшение числа импульсов на выходе из нервного центра по сравнению с длительностью раздражения

2497. Затылочные и височные отделы коры обеспечивают

Эмоциональные состояния

Интегративные функции
Функции речи

Образное мышление

Анализ сенсорной информации

2498. Тремор определяется деятельностью

Гиппокампа

Полосатого тела

Мозжечка

Гипоталамуса

Спинного мозга

2499. При блокаде симпатической иннервации можно наблюдать:

Расширение кровеносных сосудов кожи.

Расширение зрачков

Снижение артериального давления.

Уменьшение частоты сердечных сокращений.

2500. Где расположены тела преганглионарных нейронов парасимпатического отдела нервной системы?

В среднем мозге, продолговатом мозге и крестцовом отделе спинного мозга.

В шейном и грудном отделах спинного мозга.

В продолговатом мозге и грудном отделе спинного мозга.

В грудном и поясничном отделах спинного мозга.

В среднем мозге, в грудном и поясничном отделах спинного мозга.

2501. Возбуждение ядер передней группы гипоталамуса вызывает в иннервируемых органах…

симпатические эффекты

смешанные эффекты: симпатические и парасимпатические

парасимпатические эффекты

2502. Обозначьте вид торможения, представленного на рисунке.

Реципрокное

Латеральное

Постсинаптическое
Пресинаптическое

2503. Нервные центры являются…

быстро утомляемыми

медленно утомляемыми

практически неутомляемыми

вообще неутомляемыми

2504. Какие функциональные зоны выделяют в коре больших полушарий?

только сенсорные

только моторные

моторные, сенсорные, ассоциативные

моторные и сенсорные

2505. В коре мозжечка…

корзинчатые клетки - возбуждающие, все остальные тормозные

все клетки являются тормозными

клетки зерна - возбуждающие, все остальные тормозные

все клетки являются возбуждающими

грушевидные клетки - возбуждающие, все остальные тормозные

2506. При поражении базальных ядер нарушается:

Регуляция вегетативных реакций

Координация двигательной активности

Проведение афферентной импульсации от органов чувств

Все перечисленные функции

2507. Глицин увеличивает проницаемость постсинаптической мембраны для ионов…

калия

хлора

натрия

кальция

все ответы правильные


2508. При повреждении мозжечка не наблюдается…

нарушения координации движений

нарушения коленного рефлекса

потери сознания

изменения мышечного тонуса

вегетативных расстройств

2509. Левое полушарие у правшей обеспечивает

Эмоциональные состояния

Анализ сенсорной информации

Образное мышление

Функции речи

Интегративные функции

2510. Гипертермия определяется деятельностью

Гиппокампа

Спинного мозга

Гипоталамуса

Мозжечка

Полосатого тела

2511. Какие нейроны способны генерировать импульсы без воздействия раздражителей?

фоновоактивные

униполярные

полимодальные

моносенсорные

2512. Какая форма взаимодействия нейронов показана в основе механизмов распространения возбуждения в ЦНС?

Конвергенция

Пролонгирование

Окклюзия

Иррадиация
2513. Какой отдел ЦНС является ведущим в формировании статокинетических рефлексов?

Продолговатый мозг

Средний мозг

Промежуточный мозг

Кора мозга

2514. Лобные отделы коры обеспечивают

Эмоциональные состояния

Интегративные функции

Функции речи

Образное мышление

Анализ сенсорной информации

2515. Межполушарный перенос информации определяется деятельностью

Гиппокампа

Мозолистого тела

Варолиева моста

Височной коры

2516. В норме испытуемый при проведении пробы Ромберга сохраняет устойчивость, находясь в вертикальном
положении, когда…

пятки и носки слегка соприкасаются, руки скрещены на груди, глаза открыты

ноги расставлены на уровне плеч, руки скрещены на груди, глаза закрыты

пятки и носки слегка соприкасаются, руки вытянуты вперед, глаза закрыты

ноги расставлены на уровне плеч, руки вытянуты вперед, глаза закрыты

2517. Электроэнцефалография как метод исследования ЦНС применяется для изучения

Интенсивности биохимических процессов в структурах головного мозга

Химических основ работы нейрона

Состояния афферентных путей головного мозга

Суммарной электрической активности мозга

2518. Какой из перечисленных отделов ЦНС является ведущим в формировании произвольного движения?

Спинной мозг
Моторные области коры

Гипоталамус

Лимбическая система

2519. Возбуждение в нервных центрах проводится односторонне, что обусловлено…

синаптической задержкой

низкой лабильностью

все ответы правильные

свойством химических синапсов проводить возбуждение в одном направлении

2520. Какую роль играет торможение в работе нервных центров?

служит для замыкания рефлекторной дуги в ответ на раздражение

стимулирует работу нервных центров

выполняет охранительную, регулирующую и координирующую функции

обеспечивает объединение клеток ЦНС в нервные центры

2521. Какое явление возникает в нейроне, если в результате пространственной или временной суммации
возбуждающий постсинаптический потенциал достигает критического уровня деполяризации?

Гиперполяризация

Поляризация

Тормозной постсинаптический потенциал

Потенциал действия

Потенциал покоя

2522. Для развития торможения в ЦНС необходимо все, кроме…

медиатора

нарушения целостности нервного центра

открытия калиевых каналов

открытия хлорных каналов

энергии АТФ

2523. Замедление проведения возбуждения через синапсы нервного центра обусловлено…

синаптической задержкой

пролонгированием
пластичностью нервного центра

посттетанической потенциацией

2524. Слуховые ощущения определяется деятельностью

Гиппокампа

Мозолистого тела

Варолиева моста

Височной коры

2525. Где «замыкается» дуга локтевого сухожильного рефлекса?

В крестцовом отделе спинного мозга

В продолговатом мозге

В шейном отделе спинного мозга

В среднем мозге

2526. Какая форма взаимодействия нейронов показана в основе механизмов распространения возбуждения в ЦНС?

Конвергенция

Окклюзия

Пролонгирование

Иррадиация

2527. Мембранный потенциал нейрона в покое равен:

1-2 мВ

0,5 мВ

10 мВ

Около 60 мВ

Около 20 мВ

2528. Влияние инсулина на организм...

стимулирует гликолиз, катаболизм белка, угнетает липолиз

стимулирует гликолиз, липолиз и анаболизм белка

стимулирует гликолиз, анаболизм белка, угнетает липолиз

стимулирует гликолиз, липолиз и катаболизм белка


2529. В цветовом восприятии основную роль играют…

амакриновые клетки

пигментные клетки

палочки

колбочки

2530. Рецепторами какой части языка лучше всего воспринимается горький вкус?

края

верхушки

основания

боковой поверхности

спинки

2531. Пространство, видимое глазом при фиксации взгляда в одной точке называется

близорукость

поле зрения

аккомодация

острота зрения

астигматизм

2532. В организме в условиях сильного стресса уровень катехоламинов возрастает в…

2 раза

10 и более раз

100 и более раз

20 и более раз

2533. Какие из указанных образований слухового анализатора относятся к звукопроводящим?

преддверие и полукружные каналы

кортиев орган, полукружные каналы

евстахиева труба, преддверие

барабанная перепонка, слуховые косточки

2534.
2535. Проводниковый отдел сенсорной системы включает ...

специфические проводящие пути

неспецифические проводящие пути

специфические подкорковые образования

все ответы правильные

неспецифические подкорковые образования

2536. Какой гормон практически не имеет специального органа-мишени?

окситоцин

соматотропин

тиреотропин

антидиуретический гормон

2537. Общие принципы строения анализаторов

многоканальность

многослойность

все ответы правильные

дифференциация по вертикали и горизонтали

2538. Наибольшее поле зрения отмечается при восприятии…

синего цвета

красного цвета

зеленого цвета

черно-белого цвета

2539. Рецепторами слухового анализатора являются…

волосковые клетки

образования внутреннего уха

клетки спирального ганглия

опорные клетки

2540. Для какого из нижеперечисленных цветов границы поля зрения самые узкие?

желтого
синего

зеленого

красного

2541. Рецептивное поле нейрона -

совокупность рецепторов, сигналы от которых поступают на данный нейрон

совокупность высокопороговых рецепторов, сигналы от которых поступают на данный нейрон

совокупность низкопороговых рецепторов, сигналы от которых поступают на данный нейрон

совокупность нейронов мозга, сигналы от которых конвергируют на данный нейрон

2542. Приспособление глаза к ясному видению удаленных на разное расстояние предметов называется…

дальнозоркостью

аккомодацией

близорукостью

астигматизмом

2543. Влияние адреналина на зрачок

суживает

расширяет

не влияет

все ответы правильные

2544. Какая железа является местом интеграции иммунной и эндокринной систем организма?

эпифиз

аденогипофиз

нейрогипофиз

тимус

надпочечники

2545. Сенсорные системы, в отличие от анализаторов, включают …

специфические подкорковые образования

неспецифические проводящие пути

специфические проводящие пути

обратные связи
обратные связи

етикулярную формацию ствола мозга

2546. При паракринном типе действия вещество влияет на …

клетку, секретирующую данное вещество

клетки, окружающие секретирующую клетку

клетки, расположенные на значительном расстоянии от места выработки вещества

клетку, расположенную в тесном контакте с клеткой, секретирующей вещество

2547. Где в клетке расположены рецепторы для большинства пептидных гормонов?

в цитоплазме

на мембране

в оболочке ядра

на хромосомах

2548. Первичный мессенджер является фактором, который…

обеспечивает передачу сигнала от рецептора к конкретным внутриклеточным структурам

активирует комплекс гормон-рецептор

действует на рецепторы клеток в органах мишенях

вызывает диссоциацию молекулярного ингибитора

2549. Какие из нижеперечисленных эффектов присущи окситоцину?

стимуляция сокращения матки при родах

все перечисленные эффекты

сокращение гладких мышц протоков молочных желез

регуляция водно-солевого обмена и питьевого поведения

2550. Какие эффекты оказывает гормон клубочковой зоны надпочечников альдостерон?

стимулирует секрецию Na+ и реабсорбцию К+

стимулирует реабсорбцию воды и вторично Na+

стимулирует реабсорбцию Na+ и секрецию K+

стимулирует реабсорбцию воды и вторично К+

2551. Рефракция глаза -


преломление лучей в оптической системе глаза

способность видеть раздельно две точки

приспособление глаза к ясному видению разноудаленных предметов

искажение хода лучей в хрусталике

2552. Интеграция сенсорных сигналов наблюдается .....

в спинном мозге

в ретикулярной формации

в проекционных зонах коры

в таламусе

2553. Болевой анализатор располагается в ......

височной области коры

соматосенсорной области коры

лобной области коры

таламических ядрах

2554. К интерорецепторам относятся…

вестибуло-, обонятельные, вкусовые

тактильные, термо-, механо-

вестибуло-, проприо-, висцеро-

висцеро-, фото-, фоно-

2555. Рецепторный потенциал…

является постсинаптическим

не подчиняется закону «все или ничего»

подчиняется закону «все или ничего»

подчиняется закону «все или ничего»

2556. Чувствительность слухового анализатора человека максимальна при восприятии звуковых колебаний с
частотой…

5000-8000 Гц

100-400 Гц

1000-4000 Гц
600-800 Гц

2557. Гормоны циркулируют в крови…

в свободном виде

все ответы правильные

в виде комплекса со специфическими белками

2558. Отсутствие способности восприятия красного цвета называется

протанопией

дейтеранопией

тританопией

астигматизмом

аккомодацией

2559. Какая эндокринная железа принимает участие в регуляции циркадианных ритмов у млекопитающих и человека?

тимус

надпочечники

щитовидная железа

эпифиз

половые железы

2560. Адаптация в сенсорных системах

все ответы правильные

в снижении активности процессов возбуждения и торможения

в изменении чувствительности к длительно действующему раздражителю

в повышении активности процессов возбуждения и торможения

2561. Статины и либерины гипоталамуса имеют большое значение в регуляции функционирования…

нейрогипофиза

адено- и нейрогипофиза

аденогипофиза

гипоталамуса и некоторых зон таламуса

2562. Основным результатом действия вазопрессина является стимуляция…


реабсорбции воды

реабсорбции Na+ и вторично - воды

реабсорбции калия

реабсорбции ионов Н+

2563. Какие ощущения могут возникать при возбуждении висцерорецепторов?

позывы на мочеиспускание и дефекацию

голод и жажда

одышка

все ответы правильные

2564. Вторичный мессенджер является фактором, который…

действует на рецепторы клеток в органах мишенях

активирует комплекс гормон-рецептор

вызывает диссоциацию молекулярного ингибитора

обеспечивает передачу сигнала от рецептора к конкретным внутриклеточным структурам

2565. При увеличении содержания эстрогенов в плазме крови наблюдается увеличение секреции окситоцина
нейрогипофизом, что является примером …

положительной обратной связи

отрицательной обратной связи

пермиссивного эффекта

морфогенетического эффекта

2566. Какой из нижеперечисленных гормонов повышает активность остеокластов, вызывая резорбцию кости?

антидиуретический гормон

глюкагон

инсулин

паратгормон

2567. Какой из приведенных ниже пептидных факторов не участвует в регуляции секреции гипофиза?

кортиколиберин

тиреостатин

тиреолиберин
меланолиберин

2568. Рецепторами какой части языка лучше всего воспринимается кислый и соленый вкус?

спинки

верхушки

края

боковой поверхности

основания

2569. Аксоны каких клеток сетчатки образуют зрительный нерв?

амакриновых

ганглиозных

биполярных

горизонтальных

2570. Гормоны, приводящие к снижению уровня глюкозы крови

все перечисленные

тироксин, глюкокортикоиды

глюкагон, паратгормон

нет правильного ответа

адреналин, альдостерон

глюкагон, окситоцин

2571. Факторы, стимулирующие секреции и инсулина

повышение уровня глюкозы в крови

повышение уровня глюкозы и аминокислот в крови

повышение уровня аминокислот в крови

снижение всасывания глюкозы из кишечника в кровь

2572. Какое в среднем количество гормонов связано со специфическими белками-переносчиками в плазме крови?

20%

80%

60%
100%

2573. Область восприятия звуковых частот слуховым анализатором человека имеет границы…

6-10000 Гц

6-2000 Гц

16-20000 Гц

10-2000 Гц

2574. Максимальной остротой зрения обладает…

желтое пятно

слепое пятно

центральная часть сетчатки

периферическая часть сетчатки

2575. Какие пептиды относятся к стимуляторам антиноцицептивной системы?

вазопрессин, окситоцин

амилитин, скотофобин

эндорфины, энкефалины

соматолиберин, меланолиберин

2576. Разрушение ассоциативной коры головного мозга сопровождается грубыми нарушениями…

все ответы правильные

обучения и памяти

вегетативных функций

гомеостаза

2577. Возбуждение рецепторов полукружных каналов наблюдается…

при угловых ускорениях в начале движения и в момент его окончания

при линейных ускорениях постоянно

при угловых ускорениях только в начале движения

при угловых ускорениях постоянно

2578. Какие преимущественно влияния в сенсорных системах передаются по эфферентным путям?

тормозные
облегчающие

возбуждающие

тонические

2579. Если у испытуемого основной обмен повышен на 45%, то, вероятнее всего, имеет место гиперфункция…

эпифиза

аденогипофиза

-клеток островков Лангерганса

коры надпочечников

щитовидной железы

2580. При увеличении содержания глюкокортикоидов в кровеносном русле наблюдается снижение выделения
адренокортикотропного гормона аденогипофизом, что является примером…

положительной обратной связи

отрицательной обратной связи

кинетического действия

пускового действия

2581. Основная часть антидиуретического гормона (примерно 5/6 от общего количества) синтезируется в …

паравентрикулярном ядре гипоталамуса

вентромедиальном ядре гипоталамуса

супраоптическом ядре гипоталамуса

нейрогипофизе

2582. Эстрогены ....

подавляют лактацию и тормозят эритропоэз

подавляют лактацию и стимулируют эритропоэз

стимулируют лактацию и эритропоэз

стимулируют лактацию и подавляют эритропоэз

2583. Какой гормон снижает уровень Ca2+ крови?

паратгормон

тиреокальцитонин

тироксин
тироксин

адреналин

2584. Рецепторы гормонов находятся в…

гемоглобине крови

транспортном белке крови

клетках органов-мишеней

эндотелии сосудов

2585. Какой гормон является основным представителем глюкокортикоидов?

альдостерон

дезоксикортикостерон

кортизол

кортикостерон

2586. В каком из вариантов ответа неправильно указано влияние тиреоидных и стероидных гормонов на генетический
аппарат клетки?

включение-выключение определённых генов

репрессия некоторых генов

ускорение-замедление транскрипции функционирующих генов

стимуляция транскрипции определённых генов

2587. При аутокринном типе действия вещество влияет на …

клетку, секретирующую данное вещество

клетки, окружающие секретирующую клетку

клетки, расположенные на значительном расстоянии от места выработки вещества

клетку, расположенную в тесном контакте с клеткой, секретирующей вещество

2588. Для какого из нижеперечисленных цветов границы поля зрения самые широкие?

синего

желтого

зеленого

красного

2589. Благодаря наличию обратных связей от высших отделов сенсорной системы к низшим…
тормозится избыточная и облегчается проведение наиболее важной информации

облегчается проведение слуховой и зрительной информации

тормозится проведение всех видов информации

тормозится проведение болевой и облегчается проведение тактильной информации

2590. Какие гормоны обладают выраженным противовоспалительным действием?

минералокортикоиды

катехоламины

инсулин

глюкокортикоиды

тиреоидные гормоны

2591. Какие гормоны являются производными аминокислот?

тироксин, трийодтиронин

адреналин, норадреналин

дофамин

все перечисленное

2592. Рецепторный потенциал имеет характер…

иррадиирующего возбуждения

распространяющегося возбуждения

локального возбуждения

застойного возбуждения

2593. При гиперфункции щитовидной железы масса тела …

повышается

не изменяется

возникает ожирение

снижается

2594. Что является непосредственной причиной возбуждения рецепторов кортиева органа?

колебания текториальной мембраны

колебания основной мембраны


деформация волосков рецепторных клеток

деформация барабанной перепонки

2595. Что обеспечивает гипоталамо-гипофизарная портальная кровеносная система?

поступление статинов и либеринов в переднюю долю гипофиза

поступление окситоцина и вазопрессина в заднюю долю гипофиза

выделение окситоцина и вазопрессина из задней доли гипофиза в кровь

выделение гормонов аденогипофиза в кровь

2596. Какой гормон регулирует выделение глюкокортикоидов?

окситоцин

соматотропин

кортикотропин

пролактин

2597. Раздражитель, к действию которого рецептор приспособлен в процессе эволюции, называется…

биологическим

физическим

физиологическим

адекватным

2598. Отсутствие способности различать отдельные цвета называется

дальнозоркостью

дальтонизмом

астигматизмом

близорукостью

2599. Отсутствие способности к восприятию синего и фиолетового цветов называется…

ахромазией

тританопией

дейтеранопией

протанопией

2600. Какой из нижеперечисленных гормонов повышает активность остеобластов, увеличивая отложение Са2+ в
костях?

паратгормон

тиреокальцитонин

глюкагон

соматостатин

2601. Кортикотропин главным образом стимулирует выработку гормонов…

клубочковой зоны надпочечников

сетчатой зоны надпочечников

пучковой зоны надпочечников

мозгового вещества надпочечников

2602. Недостаток каких гормонов в детском возрасте вызывает задержку роста, непропорциональное телосложение,
задержку умственного развития?

тироксина, трийодтиронина

соматотропина

паратгормона

тестостерона

2603. Какой из катехоламинов в наибольшем количестве содержится в крови в покое?

дофамин

адреналин

норадреналин

диоксифенилаланин

2604. Какой гормон усиливает воспалительные явления?

альдостерон

тиреокальцитонин

адреналин

кортикотропин

2605. Уровень центральной нервной системы, где осуществляется взаимодействие сенсорных систем

таламический

ретикулярный
спинальный

корковый

все ответы правильные

2606. При близорукости изображение будет фокусироваться…

за сетчаткой

перед сетчаткой

в передней камере глаза

на сетчатке

2607. Где в клетке расположены рецепторы к стероидным гормонам?

на мембране

в оболочке ядра

на хромосомах

в цитоплазме

2608. Где вырабатывается натрий-уретический пептид?

в задней доле гипофиза

в правом предсердии

в гипоталамусе

в юкстагломерулярном аппарате почки

в плаценте

2609. Гормоны, оказывающие анаболическое действие, стимулирующие рост костей и закрытие зон роста,
увеличивающие эритропоэз

эстрогены

андрогены

гестагены

лейкопоэтины

2610. Какое из нижеперечисленных веществ относится ко вторичным посредникам (мессенджерам)?

ЦАМФ

ЦГМФ

ионизированный Ca2+ и кальмодулин


все перечисленные вещества

2611. Быстроадаптирующиеся рецепторы...

фоторецепторы

болевые (ноцицепторы)

давления (диски Меркеля)

ускорения, вибрации (тельца Паччини)

2612. Какие расстройства могут возникать при раздражении рецепторов вестибулярного аппарата?

неврозы

психозы

кинетозы

сумеречное состояние сознания

2613. Уровень секреции соматотропина во время глубокого сна …

уменьшается

увеличивается

не изменяется

все ответы правильные

2614. Каким минимальным количеством квантов света может быть возбуждена одна палочка?

40

2615. При гормональном (гемокринном) типе действия гормон влияет на …

эндокринную клетку, секретирующую гормон

клетки, окружающие эндокринную клетку

клетки, расположенные на значительном расстоянии от места выработки гормона

клетку, расположенную в тесном контакте с клеткой, секретирующей гормон

2616. Гормоны, приводящие к повышению уровня глюкозы крови

паратгормон, пролактин
р р , р

тироксин, кортизол, адреналин

альдостерон, инсулин, тиреокальцитонин

мелатонин, инсулин, эстрадиол

2617. Признаки боли включают изменения ...

ритма дыхания

лейкоцитарной формулы и гормонального спектра плазмы крови

кровяного давления

все ответы правильные

ритма сердечной деятельности

2618. При дальнозоркости изображение будет фокусироваться…

перед сетчаткой

в задней камере глаза

за сетчаткой

на сетчатке

2619. На какой участок нефрона преимущественно действует антидиуретический гормон?

капилляры клубочков

юкстагломерулярный аппарат

собирательные трубочки нефрона

проксимальный каналец нефрона

2620. Гормоны тормозящие секрецию пищеварительных соков

вазопрессин

адреналин

инсулин

альдостерон

2621. Абсолютная чувствительность сенсорной системы

минимальнй порог рецепторов

порог отдельного рецептора

максимальный порог рецепторов


порог реакции (ощущения)

2622. Удаление паращитовидных желез вызывает…

гиперкальциемию

не влияет на обмен кальция

гипокальциемию

гипокалиемию

2623. Расстройство сумеречного зрения возникает при недостатке витамина...

В6

В1

2624. Неодинаковое преломление лучей в разных плоскостях оптической системы глаза называется…

близорукостью

астигматизмом

аккомодацией

дальнозоркостью

2625. Каков суточный ритм выделения глюкокортикоидов?

уменьшаются утром и увеличиваются к вечеру

каждые 3-4 часа наблюдается их выброс с последующим снижением

каждые 60 мин наблюдается их выброс с последующим снижением

повышаются утром и уменьшаются к вечеру

2626. Где синтезируется основное количество эстрогенов в женском организме?

в клетках желтого тела

в сетчатой зоне надпочечников

в клетках фолликула

в плаценте
2627. Учёный, который создал учение об анализаторах

Н.Е. Введенский

И.П. Павлов

И.М. Сеченов

У. Гарвей

П.К. Анохин

2628. Какой из нижеперечисленных гормонов не относится к катехоламинам?

норадреналин

гистамин

адреналин

дофамин

2629. Гиперпродукция глюкокортикоидов характерна для…

базедовой болезни

болезни Иценко-Кушинга

микседемы

аддисоновой болезни

2630. Для полной характеристики положения тела в пространстве, кроме вестибулярной импульсации, необходима
дополнительная, от…

проприорецепторов шейных мышц

проприорецепторов мышц конечностей

проприорецепторов мышц туловища

проприорецепторов всех скелетных мышц

2631. Медленноадаптирующиеся рецепторы ...

вестибуло- и проприорецепторы

болевые, обонятельные, давления (диски Меркеля)

осязательные (тельца Мейснера)

ускорения, вибрации (тельца Паччини)


2632. Болевые рецепторы это

ноцицепторы

проприорецепторы

дистантные рецепторы

осморецепторы

2633. В состоянии покоя основной формой транспорта гормонов кровью к органам мишеням является…

в свободном виде

комплекс с белками-переносчиками

комплекс с форменными элементами крови

комплекс с катионами

Test System 2022

You might also like